You are on page 1of 147

Visit https://telegram.me/booksforcbse for more books.

Corporate
DISHA PUBLICATION
Office
45, 2nd Floor, Maharishi Dayanand Marg,
Corner Market, Malviya Nagar, New Delhi - 110017
Tel : 49842349 / 49842350

© Copyright
No part of this publication may be reproduced in
any form without prior permission of the publisher.

Disha
The author and the publisher do not take any legal
responsibility for any errors or misrepresentations
that might have crept in. We have tried and made
our best efforts to provide accurate up-to-date
information in this book.
All Right Reserved

Typeset by Disha DTP Team

www.dishapublication.com www.mylearninggraph.com
Books & ebooks Etests
for School &
Competitive for
Exams Competitive
Exams

Write to us at feedback_disha@aiets.co.in
Contents
MATHEMATICS
TERM - I

1. Real Numbers M-1-8

2. Polynomials M-9-17

3. Pair of Linear Equations in Two Variables M-18-27

4. Coordinate Geometry M-28-36

5. Triangles M-37-48

6. Introduction to Trigonometry M-49-56

7. Areas Related to Circles M-57-67

8. Probability M-68-75

TERM - Ii

9. Quadratic Equations M-76-86

10. Arithmetic Progressions M-87-94

11. Circles M-95-105

12. Constructions M-106-109

13. Some Applications of Trigonometry M-110-119

14. Surface Areas and Volumes M-120-131

15. Statistics M-132-142


1 Acids, Bases and
Real Numbers
Salts
7. The sum of exponents of prime factors in the prime-
Multiple Choice Questions (MCQs) factorisation of 196 is
(a) 3 (b) 4
DIRECTIONS : This section contains multiple choice (c) 5 (d) 2
questions. Each question has 4 choices (a), (b), (c) and (d) out
of which only one is correct. 8. When 2256 is divided by 17, then remainder would be
(a) 1 (b) 16
1. What is the largest number that divides 70 and 125, leaving (c) 14 (d) None of these
remainders 5 and 8 respectively?
(a) 13 (b) 9 9. The sum of three non-zero prime numbers is 100. One of
them exceeds the other by 36. Then, the largest number is
(c) 3 (d) 585
(a) 73 (b) 91
2. What is the largest number that divides 245 and 1029, (c) 67 (d) 57
leaving remainder 5 in each case?
p
(a) 15 (b) 16 10. The rational number of the form , q ≠ 0, p and q
q
(c) 9 (d) 5
are positive integers , which represents 0.134 i.e.,
3. A class of 20 boys and 15 girls is divided into n groups so (0.1343434....) is
that each group has x boys and y girls. Values of x, y and
134 134
n respectively are (a) (b)
999 990
(a) 3, 4 and 8 (b) 4, 3 and 6
(c) 4, 3 and 7 (d) 7, 4 and 3 133 133
(c) (d)
999 990
4. If p, q are two consecutive natural numbers, then H.C.F.
(p, q) is 11. The least number which is a perfect square and is divisible
(a) p (b) q by each of 16, 20 and 24 is
(c) 1 (d) pq (a) 240 (b) 1600
(c) 2400 (d) 3600
5. Given that L.C.M. (91, 26) = 182, then H.C.F. (91, 26) is
(a) 13 (b) 26 12. If n is an even natural number, then the largest natural
number by which n (n + 1) (n + 2) is divisible is
(c) 17 (d) 9
(a) 6 (b) 8
6. Which of the following statement is true? (c) 12 (d) 24
(a) Every point on the number line represents a rational
number. 13. The least number which when divided by 15, leaves a
remainder of 5, when divided by 25, leaves a remainder
(b) Irrational numbers cannot be represented by points on
of 15 and when divided by 35, leaves a remainder of 25,
the number line.
is
22 (a) 515 (b) 525
(c) is a rational number.
7 (c) 1040 (d) 1050
(d) None of these.
M-2 Mathematics
14. The number 313 – 310 is divisible by 22. On dividing a natural number by 13, the remainder is 3 and
(a) 2 and 3 on dividing the same number by 21, the remainder is 11. If
(b) 3 and 10 the number lies between 500 and 600, then the remainder
on dividing the number by 19 is
(c) 2, 3 and 10
(a) 4 (b) 6
(d) 2, 3 and 13
(c) 9 (d) 13
15. A number lies between 300 and 400. If the number is added
to the number formed by reversing the digits, the sum is 23. Let a1, a2,..., a100 be non-zero real numbers such that
888 and if the unit’s digit and the ten’s digit change places, a1 + a2 + ...+ a100 = 0
the new number exceeds the original number by 9. Then, Then,
the number is 100 100

(a) 339 (b) 341 ∑ ai 2ai > 0and


(a) ∑ ai 2−ai < 0
i =1 i =1
(c) 378 (d) 345 100 100
16. Which of the following will have a terminating decimal ∑ ai 2ai ≥ 0and
(b) ∑ ai 2−ai ≥ 0
i =1 i =1
expansion?
100 100
77 23
(a) (b) ∑ ai 2ai ≤ 0and
(c) ∑ ai 2−ai ≤ 0
210 30 i =1 i =1
125 23 100 100
(c)
441
(d)
8 (d) The sign of ∑ ai 2ai or ∑ ai 2−ai depends on the
i =1 i =1
17. I. The L.C.M. of x and 18 is 36. choice of ai’s
II. The H.C.F. of x and 18 is 2. 24. The value of 0.235 is :
What is the number x ? 233 233
(a) (b)
(a) 1 (b) 2 900 990
(c) 3 (d) 4 235 235
(c) (d)
999 990
18. If a = 23 × 3, b = 2 × 3 × 5, c = 3n × 5 and
25. Consider the following statements: For any integer n,
L.C.M. (a, b, c) = 23 × 32 × 5, then n =
(a) 1 (b) 2 n2 + 3 is never divisible by 17.
I.
(c) 3 (d) 4 n2 + 4 is never divisible by 17.
II.
19. If p 1 and p 2 are two odd prime numbers such that Then,
p1 > p2, then p12 – p22 is
(a) both I and II are true
(a) an even number
(b) both I and II are false
(b) an odd number
(c) I is false and II is true
(c) an odd prime number
(d) I is true and II is false
(d) a prime number
1
20. When a natural number x is divided by 5, the remainder 26. Given that = 0.142857 , which is a repeating decimal
7
is 2. When a natural number y is divided by 5, the
remainder is 4. The remainder is z when x + y is divided having six different digits. If x is the sum of such first three
2z − 5 1
by 5. The value of is positive integers n such that = 0.abcdef , where a, b, c,
3 n
(a) –1 (b) 1 d, e and f are different digits, then the value of x is
(c) –2 (d) 2 (a) 20 (b) 21
(c) 41 (d) 42
21. The largest non-negative integer k such that 24k divides
13! is 27. If m = n2 – n, where n is an integer, then m2 – 2m is divisible by
(a) 2 (b) 3 (a) 20 (b) 24
(c) 4 (d) 5 (c) 30 (d) 16
Real Numbers M-3
28. The unit digit in the expression 55725 + 735810 + 22853 is (c) Every real number is rational.
(a) 0 (b) 4 (d) Every point on a number line is associated with a real
(c) 5 (d) 6 number.

29. For some integer m, every even integer is of the form 37. Which of the following statement(s) is/are not correct?
(a) m (b) m+1 (a) There are infinitely many even primes.
(c) 2m (d) 2m + 1 (b) Let ‘a’ be a positive integer and p be a prime number
such that a2 is divisible by p, then a is divisible by p.
30. For some integer q, every odd integer is of the form (c) Every positive integer different from 1 can be
(a) q (b) q+1 expressed as a product of non-negative power of 2
(c) 2q (d) 2q + 1 and an odd number.
33 p is an irrational number.
31. The decimal expansion of the rational number 2 will (d) If ‘p’ is a positive prime, then
terminate after 2 .5
(a) one decimal place
(b) two decimal places
(c) three decimal places DIRECTIONS : Study the given Case/Passage and answer the
(d) more than 3 decimal places following questions.

32. Product of two co-prime numbers is 117. Their L.C.M. Case/Passage-I


should be To enhance the reading skills of grade X students, the school
(a) 1 nominates you and two of your friends to set up a class
(b) 117 library. There are two sections-section A and section B of
(c) equal to their H.C.F. grade X. There are 32 students in section A and 36 students
in section B.
(d) Lies between 1 to 117

33. Which of the following statement(s) is/are always true?


(a) The sum of two distinct irrational numbers is
rational.
(b) The rationalising factor of a number is unique.
(c) Every irrational number is a surd.
(d) None of these

34. Which of the following statement(s) is/are not correct?


73
(a) is a non-terminating repeating decimal. [From CBSE Question Bank-2021]
54
38. What is the minimum number of books you will acquire
(b) If a = 2 + 3 and b = 2 – 3 , then a + b is for the class library, so that they can be distributed equally
irrational. among students of Section A or Section B?
(c) If 19 divides a3, then 19 divides a, where a is a positive (a) 144 (b) 128 (c) 288 (d) 272
integer.
39. If the product of two positive integers is equal to the product
(d) Product of L.C.M. and H.C.F. of 25 and 625 is 15625.
of their HCF and LCM is true then, the HCF (32, 36) is
35. The product of unit digit in (795 – 358) and (795 + 358) is (a) 2 (b) 4 (c) 6 (d) 8
(a) 8 40. 36 can be expressed as a product of its primes as
(b) lies between 3 and 7 (a) 22 × 32 (b) 21 × 33
(c) 6 3
(c) 2 × 3 1 (d) 20 × 30
(d) lies between 3 and 6
41. 7× 11 × 13 × 15 + 15 is a
36. Which of the following statement(s) is/are not correct? (a) Prime number
(a) Every integer is a rational number. (b) Composite number
(b) The sum of a rational number and an irrational number (c) Neither prime nor composite
is an irrational number. (d) None of the above
M-4 Mathematics
42. If p and q are positive integers such that p = ab2 and Observe the following factor tree and answer the following:
q = a2b, where a, b are prime numbers, then the LCM 48. What will be the value of x?
(p, q) is (a) 15005 (b) 13915
(a) ab (b) a2b2 (c) a3b2 (d) a3b3 (c) 56920 (d) 17429
Case/Passage-II
49. What will be the value of y?
A seminar is being conducted by an Educational Organisation, (a) 23 (b) 22 (c) 11 (d) 19
where the participants will be educators of different subjects.
The number of participants in Hindi, English and Mathematics 50. What will be the value of z?
are 60, 84 and 108 respectively. (a) 22 (b) 23 (c) 17 (d) 19
51. According to Fundamental Theorem of Arithmetic 13915
is a
(a) Composite number
(b) Prime number
(c) Neither prime nor composite
(d) Even number
52. The prime factorisation of 13915 is
[From CBSE Question Bank-2021] (a) 5 × 113 × 132 (b) 5 × 113 × 232
2
(c) 5 × 11 × 23 (d) 5 × 112 × 132
43. In each room the same number of participants are to be
seated and all of them being in the same subject, hence
maximum number participants that can accommodated Assertion & Reason
in each room are
(a) 14 (b) 12 (c) 16 (d) 18 DIRECTIONS : Each of these questions contains an Assertion
followed by Reason. Read them carefully and answer the
44. What is the minimum number of rooms required during
the event? question on the basis of following options. You have to select
the one that best describes the two statements.
(a) 11 (b) 31 (c) 41 (d) 21
45. The LCM of 60, 84 and 108 is (a) If both Assertion and Reason are correct and Reason is
the correct explanation of Assertion.
(a) 3780 (b) 3680 (c) 4780 (d) 4680
(b) If both Assertion and Reason are correct, but Reason is
46. The product of HCF and LCM of 60,84 and 108 is not the correct explanation of Assertion.
(a) 55360 (b) 35360 (c) 45500 (d) 45360
(c) If Assertion is correct but Reason is incorrect.
47. 108 can be expressed as a product of its primes as (d) If Assertion is incorrect but Reason is correct.
(a) 23 × 32 (b) 23 × 33
13
(c) 22 × 32 (d) 22 × 33 53. Assertion : is a terminating decimal fraction.
3125
Case/Passage-III
Reason : If q = 2n.5m where n, m are non-negative integers,
A Mathematics exhibition is being conducted in your school p
and one of your friendsis making a model of a factor tree. He then is a terminating decimal fraction.
q
has some difficulty and asks for your help in completing a quiz
for the audience. 54. Assertion : Denominator of 34.12345. When expressed
p
x in the form , q ≠ 0 , is of the form 2m × 5n, where m, n
q
5 2783 are non-negative integers.
Reason : 34.12345 is a terminating decimal fraction.
253 y
55. Assertion : The H.C.F. of two numbers is 16 and their
product is 3072. Then, their L.C.M = 162.
11 z

Reason : If a, b are two positive integers, then
[From CBSE Question Bank-2021] H.C.F × L.C.M. = a × b.
Real Numbers M-5
56. Assertion : 2 is a rational number.
Reason : The square roots of all positive integers are Fill in the Blanks
irrationals.
DIRECTIONS : Complete the following statements with an
57. Assertion : If L.C.M. {p, q} = 30 and H.C.F {p, q} = 5, appropriate word / term to be filled in the blank space(s).
then p.q = 150.
Reason : L.C.M. of (a, b) × H.C.F of (a, b) = a.b. 63. 5 is a/ an ............... number.
58. Assertion : n2 – n is divisible by 2 for every positive integer. 1
64. is a/ an ............... number.
Reason : 2 is not a rational number. 2

59. Assertion : n2 + n is divisible by 2 for every positive 65. The exponent of 2 in the prime factorisation of 144,
integer n. is ...............

Reason : If x and y are odd positive integers, from 66. 7 5 is a/ an ............... number.
x2 + y2 is divisible by 4.
67. 6 + 2 is a/ an ............... number.
Match the Following 68. An .............. is a series of well defined steps which gives
a procedure for solving a type of problem.
DIRECTIONS : Each question contains statements given in 69. An........... is a proven statement used for proving another
two columns which have to be matched. Statements (A, B, C, D) statement.
in column-I have to be matched with statements (p, q, r, s) in column-II.
70. L.C.M. of 96 and 404 is ..................
60. Column-I Column-II 71. H.C.F. of 6, 72 and 120 is ..............
(A) Irrational number is (p) rational number
72. 156 as a product of its prime factors .............
always
(B) Rational number is (q) irrational number 35
73. is a ..................... decimal expansion.
always 50
(C) 3 6 is not a (r) non-terminating,
non-repeating True / False
(D) 2 2 is an (s) terminating decimal
61. Column-I Column-II DIRECTIONS : Read the following statements and write your
answer as true or false.
(A) H.C.F of the smallest (p)  6
composite number and 74. Given positive integers a and b, there exist whole numbers
the smallest prime q and r satisfying a = bq + r, 0 ≤ r < b.
number 75. Every composite number can be expressed (factorised) as
(B) H.C.F of 336 and 54 (q)  5 a product of primes and this factorisation is unique, apart
(C) H.C.F of 475 and 495 (r)  2 from the order in which the prime factors occur.
62. Column-I Column-II 76. 2 and 3 are irrational numbers.
551
(A) (p) a prime number 77. If x = p/q be a rational number, such that the prime factorisation
2 × 56 × 79
3
of q is of the form 2n5m, where n, m are non-negative integers.
(B) Product of ( )
5 – 3 (q) is an irrational number Then x has a decimal expansion which is terminating.
78. Any positive odd integer is of the form 6q + 1 or 6q + 3 or
and ( 5 + 3 is ) 6q + 5, where q is some integer.
(C) 5 – 4 (r) is a terminating decimal 79. The quotient of two integers is always a rational number.
representation 80. 1/0 is not rational.
422 81. The number of irrational numbers between 15 and 18 is
(D) 3 4 (s) is a non-terminating but
2
× 5 repeating decimal infinite.
representation 82. Every fraction is a rational number.
M-6 Mathematics

ANSWER KEY & SOLUTIONS


1. (a) Required number = H.C.F. {(70 – 5), (125 – 8)} 134 − 1 133
10. (d) 0.134 = =
= H.C.F. (65, 117) = 13. 990 990
2. (b) Required number = H.C.F.{(245 – 5), (1029 – 5)} 11. (d) The L.C.M. of 16, 20 and 24 is 240. The least multiple
of 240 that is a perfect square is 3600 and also we
= H.C.F. (240, 1024) = 16.
can easily eliminate choices (a) and (c) since they
3. (c) H.C.F. of 20 and 15 = 5 are not perfect number. Hence, the required least
So, 5 students are in each group. number which is also a perfect square is 3600 which
is divisible by each of 16, 20 and 24.
20 + 15 35
∴ n= = =7 12. (d) Out of n and n + 2, one is divisible by 2 and the other
5 5
by 4, hence n (n + 2) is divisible by 8. Also n, n + 1,
Hence, x = 4, y = 3 and n = 7 n + 2 are three consecutive numbers, hence one of them is
4. (c) 1 divisible by 3. Hence, n (n + 1) (n + 2) must be divisible
by 24. This will be true for any even number n.
91 × 126 91 × 126 13. (a) The number divisible by 15, 25 and 35 = L.C.M.
5. (a) H.C.F. (91, 126) = = = 13
L.C.M.(91, 126) 182 (15, 25, 35) = 525
6. (d) All the given statements are false. Since, the number is short by 10 for complete division
by 15, 25 and 35.
7. (b) 196 = 22 ⋅ 72, sum of exponents = 2 + 2 = 4
Hence, the required least number = 525 – 10 = 515.
2256 (24 )64 14. (d) 313 – 310 = 310 (33 – 1) = 310 (26) = 2 × 13 × 310
8. (a) When 2256 is divided by 17 then, =
24 + 1 (24 + 1) Hence, 313 – 310 is divisible by 2, 3 and 13.
By remainder theorem when f (x) is divided by x + a 15. (d) Sum is 888 ⇒ unit’s digit should add up to 8. This is
the remainder = f (– a) possible only for option (d) as “3” + “5” = “8”.
16. (d)
Here, f (a) = (24)64 and x = 24 and a = 1
17. (d) L.C.M × H.C.F = First number × second number
∴ Remainder = f (–1) = (–1)64 = 1
36 × 2
Hence, required number = =4.
9. (c) Since, the sum of all the three prime numbrs is 100. 18
18. (b) Value of n = 2.
Then, there are two cases
19. (a) Since, p1 and p2 are odd primes and sum of two odd
Case 1: All the three numbers should be even because number is an even number.
100 is an even number. But this case is not possible So, p1 + p2 is an even number.
as there is only one even prime. Since, multiple of even number is always even.
Case 2 : One prime is even and other two primes are Therefore, (p1 + p2) (p1 – p2) is even
odd. 2 2
Hence, p1 − p2 = ( p1 + p2 )( p1 − p2 ) is an even
Since, 2 is only even prime, so it must be one of three number.
primes. 20. (a) Since, x is divided by 5, the remainder is 2
Let p and p + 36 be the other two primes. therefore x = 5m + 2
Then, according to question similarly, y = 5n + 4 consider x + y = 5(m + n) + 6
2 + p + (p + 36) = 100; 2p + 38 = 100 = 5(m + n) + 5 + 1 = 5(m + n + 1) + 1
But given that when x + y is divided by 5, the remainder
62
2p = 100 – 38 = 62; p = = 31 is z
2
\ z = 1
So, all the three primes are 2, 31 and 67.
2z – 5 2 (1) – 5
Hence, largest prime number is 67. Now, = = –1
3 3
Real Numbers M-7
21. (b) We know that 1
26. (c) = 0.142857
13! = 2 × 3 × 4 × 5 × 6 × 7 × 8 × 9 × 10 × 11 × 12 × 13 7
= 210 × 35 × 52 × 7 × 11 × 13 ⇒ 24k = (23 × 3)k The second positive integer whose reciprocal have six
different repeating decimals is
where k is largest non-negative integer
1
When 13! is an divided by 24k, we get = 0.076923
13
210 ¥ 35 ¥ 52 ¥ 7 ¥ 11 ¥ 13
And the third positive integer whose reciprocal have
23k .3k
six different repeating decimals is
= 210– 3k . 35 – k . 52 × 7 × 11 × 13 1
= 0.047619
\ 10 – 3k is integer. 21
Then, maximum value of k = 3. Therefore, the values of x are 7, 13, 21
22. (a) Given: The natural number, when divided by 13 leaves Hence, the required sum is = 7 + 13 + 21 = 41
remainder 3
27. (b) ∵ m = n2 – n = n(n – 1)
The natural number, when divided by 21 leaves
Now, m2 – 2m = m(m – 2)
remainder 11
= n(n – 1)(n2 – n – 2) = n(n – 1)(n – 2)(n + 1)
So, 13 – 3 = 21 – 11 = 10 = k
Since we know that product of any four consecutive
Now, LCM (13, 21) = 273
integers is always divisible by 24.
But the number lies between 500 and 600
\ m2 – 2m is divisible by 24.
\ 2 LCM (13, 21) – k = 546 – 10 = 536
28. (d)
For given numbers,
536 = 19 × 8 + 4 \ remainder = 4
(55)725, unit digit = 5; (73)5810, unit digit = 9
23. (a) Let a1, a2, a3, ..., a100 be non-zero real number and
(22)853, unit digit = 2
a1 + a2 + a3 + ... + a100 = 0 Unit digit in the expression
ai − ai
ai ⋅ 2 > ai and ai ⋅ 2 < ai 55725 + 735810 + 22853 is 6
100 100 100 100 29. (c) 30. (d) 31. (b)
∴ ∑ a1 ⋅ 2ai > ∑ ai and ∑ a1 ⋅ 2−ai < ∑ ai 32. (b)
Since, H.C.F. of co-prime number is 1.
i =1 i =1 i =1 i =1
100 100 ∴ Product of two co-prime numbers is equal to their
⇒ ∑ a1 ⋅ 2ai > 0 and ∑ a1 ⋅ 2−ai < 0 L.C.M. So, L.C.M. = 117
i =1 i =1
33. (d)
Hence, option (a) is correct.
34. (a)
24. (c) Let x = 0.235 ...(i)
35. (a) Unit digit in (795) = Unit digit in [(74)23 × 73]
1000 x = 235.235 ...(ii) = Unit digit in 73 (as unit digit in 74 = 1)
235
Subtract (i) from (ii), 999 x = 235 ⇒ x = = Unit digit in 343
999
Unit digit in 358 = Unit digit in (34)4 × 32
25. (d) Let us consider that n2 + 3 is divisible by 17
[as unit digit 34 = 1]
∴ n2 + 3 = 17K [K ∈ N] = Unit digit is 9
⇒ n2 = 17K – 3 ⇒ n2 = 3 (17m – 1) [∵ K = 3m] So, unit digit in (795 – 358)
= Unit digit in (343 – 9) = Unit digit in 334 = 4
3(17m – 1) is a perfect square, which is not possible.
Unit digit in (795 + 358) = Unit digit in (343 + 9)
∴ n2 + 3 is never divisible by 17. = Unit digit in 352 = 2
In, n2 + 4, put n = 9 So, the product is 4 × 2 = 8
So, (9)2 + 4 = 81 + 4 = 85 which is divisible by 17. 36. (c)
37. (a)
∴ I is true and II is false.
M-8 Mathematics
38. (c) For getting least number of books, 53. (a) Reason is correct.
taking LCM of 32, 36 Since, the factors of the denominator 3125 is of the
432, 36 form 20 × 55.
88, 9 13
\ is a terminating decimal
91, 9 3125
1, 1 Since, assertion follows from reason.
⇒ 4 × 8 × 9 = 288 54. (a) Reason is clearly true.
39. (b)
HCF of 32, 36 is
3412345 682469 682469
Again, 34.12345 = = = 5 4
4 32, 36 100000 20000 2 ×5
8, 9
Its denominator is of the form 2m × 5n, where
=4
m = 5, n = 4 are non-negative integers
40. (a) 36 is expressed as prime
\ Assertion is true. Since, reason gives assertion
36 = 2 × 2 × 3 × 3 = 22 × 32
41. (b) 7 × 11 × 13 × 15 + 15 \ (a) holds.
⇒ 15 (7 × 11 × 13 + 1) 55. (d) Here, reason is true [standard result]
so given no. is a composite number.
3072
42. (b) Given a, b are prime number. So Assertion is false. ∵ = 192 ≠ 162
16
LCM of p, q, where p = ab2, q = a2b 56. (c) Here, reason is not true.
p = a × b × b
∵ 4 = ±2, which is not an irrational number.
q = a × b × a
a × b × b × a ⇒ a2b2 \ Reason does not hold. Clearly, assertion is true.
43. (b) For maximum number of participants, taking HCF of 57. (a) 58. (b) 59. (a)
60, 84 and 108
12 60, 84, 108 60. (A) → (r) [Q 12 = 3 × 4 Q it is a composite number]
5, 7, 9 (B) → (s) [Q g.c.d. between 2 and 7 = 1]
= 12 (C) → (p) [Q 2 is a prime number]
44. (d) Minimum number of rooms required are (D) → (q) [Q 2 is not a rational number]
5 + 7 + 9 = 21
61. (A) → (r); (B) → (p); (C) → (q)
45. (a) LCM of 60, 84, 108 is
62. (A) → (s); (B) → (p); (C) → (q); (D) → (r)
12 × 5 × 7 × 9 = 3780
46. (d) Product is = 12 × 3780 = 45360 63. irrational 64. irrational
47. (d) 108 = 2 × 2 × 3 × 3 × 3 = 22 × 33 65. 4 66. irrational
48. (b) x = 5 × 2783 = 13915 67. irrational 68. algorithm
49. (c) y = 253 ) 2783 ( = 11
69. lemma 70. 9696
50. (b) z = 11) 253 ( = 23
71. 6 72. 22 × 3 × 13
51. (a) Composite number having more than 2 factors.
52. (c) Prime factorisation of 13915 = 73. terminating
5 13915 74. True 75. True
11 2783 76. True 77. True
11 253 78. True 79. False
23 23 80. True 81. True
1
82. True
⇒ 5 × 11 × 11 × 23
⇒ 5 × 112 × 23
2 Acids, Bases and
Polynomials
Salts
6. If f (x) = 2x3 – 6x + 4x – 5 and g(x) = 3x2 – 9, then the
Multiple Choice Questions (MCQs) value of f (1) + g(–2) is
(a) –3 (b) –2
DIRECTIONS : This section contains multiple choice
(c) 3 (d) 2
questions. Each question has 4 choices (a), (b), (c) and (d) out
of which only one is correct. 7. Factor of the polynomial x3 – 3x2 – 10x + 24 are:
(a) (x – 2)(x + 3)(x – 4)
1. If the zeroes of the polynomial f (x) = k2x2 – 17x + k + 2,
(k > 0) are reciprocal of each other than value of k is (b) (x + 2)(x + 3)(x + 4)

(a) 2 (b) –1 (c) (x + 2)(x – 3)(x – 4)

(c) –2 (d) 1 (d) (x – 2)(x – 3)(x – 4)

2. If one zero of the quadratic polynomial 8. The zeroes of the polynomial are

5 p(x) = x2 –10x –75


2x2 – 8x – m is , then the other zero is
2 (a) 5, – 15 (b) 5, 15
2 2 (c) 15, – 5 (d) – 5, – 15
(a) (b) –
3 3
9. If a and b are zeroes of the polynomial
3 −15
(c) (d) 2t2 – 4t + 3, then the value of a2b + ab2 is :
2 2
3. Let p(y) = y4 – 3y2 + 2y + 5, then the remainder when p(y) 3
(a) (b) 2
is divided by (y – 1). 4
(a) 2 (b) 3 (c) 3 (d) 4

(c) –5 (d) 5 10. The zeroes of the polynomial x2 – 3x – m(m + 3) are


m, m + 3
(a) (b) –m, m +3
4. If the polynomials ax3 + 4x2 + 3x – 4 and x3 – 4x + a
leave same remainder when divided by (x – 3), find the m, –(m + 3)
(c) (d) –m, –(m + 3)
value of a. 11. The value of x, for which the polynomials
(a) –1 (b) 1 x2 – 1 and x2 – 2x + 1 vanish simultaneously, is
1 1 (a) 2 (b) –2
(c) −
(d)
2 2 (c) –1 (d) 1
5. Let f(x) = x2 – 27x + 196. If f (a) = a, then what is the 12. If x = 0.7 , then 2x is
value of a.
(a) 1.4 (b) 1.5
(a) 7 (b) 14
(c) 1.54 (d) 1.45
(c) 21 (d) 6
M-10 Mathematics
13. Lowest value of x2 + 4x + 2 is 21. The polynomial, f(x) = (x – 1)2 + (x – 2)2 + (x – 3)2 +
(a) 0 (b) –2 (x – 4)2 has minimum value, when x = ...................
(c) 2 (d) 4 (a) 40 (b) 20

14. If a3 – 3a2b + 3ab2 – b3 is divided by (a – b), then the (c) 10 (d) 2.5
remainder is 22. If one zero of the quadratic polynomial x2 + 3x + k is 2,
a2 – ab
(a) + b2 then the value of k is
(b) a2 + ab + b2 (a) 10 (b) –10
(c) 1 (c) 5 (d) –5
(d) 0 23. If one of the zeroes of the quadratic polynomial
(k –1) x2 + kx + 1 is –3, then the value of k is
15. A quadratic polynomial when divided by x + 2 leaves
a remainder of 1 and when divided by x – 1, leaves a 4 −4
(a) (b)
remainder of 4. What will be the remainder if it is divided 3 3
by (x + 2) (x – 1) ? 2 −2
(c) (d)
(a) 1 (b) 4 3 3

(c) x + 3 (d) x – 3 24. The zeroes of the quadratic polynomial x2 + 99x + 127
are
16. If the polynomials ax3 + 4x2 + 3x – 4 and x3 – 4x + a leave (a) both positive
the same remainder when divided by x – 3, then the value
(b) both negative
of a is
(c) one positive and one negative
(a) 1 (b) –1
(c) 19/14 (d) –5/14 (d) both equal
25. Which of the following given options is/are correct?
17. If the value of a quadratic polynomial p(x) is 0 only at
x = –1 and p(–2) = 2, then the value of p(2) is 2
(a) + 3 is a polynomial
(a) 18 (b) 9 x

(c) 6 (d) 3 (b) x + 5 is a polynomial

18. If x2 – 4 is the factor of 2x3 + k1x2 + k2x + 12, where k1, 2


(c) is a polynomial
k2 are constant, then the value of k1 + k2 is 3x – 4
(a) 11 (b) 5 1 3
(d) 5 x 2 + x + is a polynomial
2 7
(c) –11 (d) –5
26. Which of the following given options is/are correct?
19. If x = 3 + 32/3 + 31/3, then the value of (a) Degree of a zero polynomial is ‘0’.
x3 – 9x2 + 18x – 12 is (b) Degree of a zero polynomial is not defined.
(a) 1 (b) 0 (c) Degree of a constant polynomial is not defined.
(c) –1 (d) 2 (d) A polynomial of degree n must have n zeroes.
1 27. Which of the following is/are a polynomial?
20. Let P(x) be a polynomial of degree 3 and P(n) = for
2
n = 1, 2, 3, 4. Then the value of P(5) is 1
x2 +
(a)
1 x
(a) 0 (b)
5 2 x2 – 3 x + 1
(b)
2 3

(c) (d) x3 – 3x + 1
(c)
5 5
3
(d)
2 x 2 – 5x
Polynomials M-11
28. Which of the following is/are not graph of a quadratic 1
polynomial ? 31. If a and α
are the zeroes of the quadratic polynomial
Y
2x2 – x + 8k, then k is
1
A B (a) 4 (b)
(a)
X X (b) 4
O
–1
(c) (d) 2
Y 4
Y Y
32. The graph of x2 + 1 = 0
(a) Intersects x-axis at two distinct points.
A B X
(c)
X
O X (d) O
X
(b) Touches x-axis at a point.
Y Y (c) Neither touches nor intersects x-axis.
(d) Either touches or intersects x-axis.

33. If the sum of the roots is –p and product of the roots is


DIRECTIONS : Study the given Case/Passage and answer 1
– , then the quadratic polynomial is
the following questions. p

 x   x 
Case/Passage-I k  px 2 – –1
k  – px 2 + + 1 (b)
(a)
 p   p 
The below picture are few natural examples of parabolic shape
which is represented by a quadratic polynomial. A parabolic arch
 1  1
is an arch in the shape of a parabola. In structures, their curve k  x 2 + px –  (d)
(c) k  x 2 – px + 
 p  p
represents an efficient method of load, and so can be found in
bridges and in architecture in a variety of forms.
Case/Passage-II
An asana is a body posture, originally and still a general term
for a sitting meditation pose, and later extended in hatha yoga
and modern yoga as exercise, to any type of pose or position,
adding reclining, standing, inverted, twisting, and balancing
poses. In the figure, one can observe that poses can be related
  to representation of quadratic polynomial.

TRIKONASANA

[From CBSE Question Bank-2021]
29. In the standard form of quadratic polynomial, ax2 + bx + c,
a, b and c are
ADHOMUKHA SAVASANA   ADHO MUKHA SVANA
(a) All are real numbers.
(b) All are rational numbers. [From CBSE Question Bank-2021]
(c) ‘a’ is a non zero real number and b and c are any real
34. The shape of the poses shown is
numbers.
(a) Spiral (b) Ellipse
(d) All are integers.
(c) Linear (d) Parabola
30. If the roots of the quadratic polynomial are equal, where
the discriminant D = b2 – 4ac, then 35. The graph of parabola opens downwards, if__________.
(a) D > 0 (b) D < 0 (a) a ≥ 0 (b) a = 0
(c) D ≥ 0 (d) D = 0 (c) a < 0 (d) a > 0
M-12 Mathematics
36. In the graph, how many zeroes are there for the polynomial?

–2 4
3
1

2 4

–8

(a) 0 (b) 1
(c) 2 (d) 3 [From CBSE Question Bank-2021]

37. The two zeroes in the above shown graph are 39. The shape of the path traced shown is
(a) 2, 4 (b) –2, 4 (a) Spiral (b) Ellipse (c) Linear (d) Parabola
(c) –8, 4 (d) 2, –8 40. The graph of parabola opens upwards, if____________.
(a) a = 0 (b) a < 0 (c) a > 0 (d) a ≥ 0
38. The zeroes of the quadratic polynomial 4 3 x 2 + 5 x – 2 3
are 41. Observe the following graph and answer
2 3 2 3
(a) , –
(b) ,
3 4 3 4 6

2 3 2 3
(c) , – (d) – ,− 2
3 4 3 4 –4 –3 –2 –1 1 2 3 4
–2
Case/Passage-III
Basketball and soccer are played with a spherical ball. Even –6
though an athlete dribbles the ball in both sports, a basketball
player uses his hands and a soccer player uses his feet. Usually,
soccer is played outdoors on a large field and basketball is In the above graph, how many zeroes are there for the
played indoor on a court made out of wood. The projectile (path polynomial?
traced) of soccer ball and basketball are in the form of parabola
(a) 0 (b) 1 (c) 2 (d) 3
representing quadratic polynomial.
42. The three zeroes in the above shown graph are
(a) 2, 3, –1 (b) –2, 3, 1
(c) –3, –1, 2 (d) –2, –3, –1
v = 8.552 m/s 43. What will be the expression of the polynomial?
(a) x3 + 2x2 − 5x − 6 (b) x3 + 2x2 − 5x + 6
θ = 51.89°

R = 7.239 m x3 + 2x2 + 5x − 6
(c) (d) x3 + 2x2 + 5x + 6

h = 3.048 m Assertion & Reason

DIRECTIONS : Each of these questions contains an Assertion


followed by Reason. Read them carefully and answer the
question on the basis of following options. You have to select
the one that best describes the two statements.
Polynomials M-13
(a) If both Assertion and Reason are correct and Reason is
the correct explanation of Assertion. Fill in the Blanks
(b) If both Assertion and Reason are correct, but Reason is
not the correct explanation of Assertion. DIRECTIONS : Complete the following statements with an
(c) If Assertion is correct but Reason is incorrect. appropriate word / term to be filled in the blank space(s).
(d) If Assertion is incorrect but Reason is correct.
51. Polynomials of degrees 1, 2 and 3 are called ...............,
44. Assertion : If one zero of polynomial p(x) = (k 2 + 4) x2 + .................. and ............. polynomials respectively.
13x + 4k is reciprocal of other, then k = 2.
52. The zeroes of a polynomial p(x) are precisely the
Reason : If (x – α) is a factor of p(x), then p(α) = 0 i.e. α
x-coordinates of the points, where the graph of y = p(x)
is a zero of p(x).
intersects the ...............-axis.
45. Assertion : x3 + x has only one real zero.
53. A quadratic polynomial can have at most 2 zeroes and a
Reason : A polynomial of nth degree must have n real cubic polynomial can have at most ............. zeroes.
zeroes.
54. If α and β are the zeroes of the quadratic polynomial
46. Assertion : Degree of a zero polynomial is not defined. −b c
ax2 + bx + c, then α + β = & αβ =
Reason : Degree of a non-zero constant polynomial is ...... .....
‘0’. 55. Zero of a polynomial is always ...............

47. Assertion : Zeroes of f(x) = x2 – 4x – 5 are 5, – 1. 56. A polynomial of degree n has at the most ........... zeroes.

Reason : The polynomial whose zeroes are 2 + 3, 2 – 3


is x2 – 4x + 7. True / False
48. Assertion : x2 + 4x – 5 has two zeroes.
DIRECTIONS : Read the following statements and write your
Reason : A quadratic polynomial can have at the most answer as true or false.
two zeroes.
57. Sum of zeroes of quadratic polynomial
Match the Following (coefficient of x )
= −
(coefficient of x 2 )
1
DIRECTIONS : Each question contains statements given in 1
58. x 2 + 1 is a polynomial
two columns which have to be matched. Statements (A, B, C, D) 5
in column-I have to be matched with statements (p, q, r, s) in 3
column-II. 6 x + x2
59. is a polynomial, x ≠ 0
49. Column-I Column-II x
(Zeroes) (Quadratic 60. Product of zeroes of quadratic polynomial
polynomial) constant term
= −
(A) 3 and –5 (p) x2 – 25 (coefficient of x 2 )
(B) 5 + 2 and 5 − 2 (q) + 2x – 15 x2 61. A polynomial cannot have more than one zero.
(C) – 9 and 1/9 2
(r) x + (80/9)x – 1
62. The degree of the sum of two polynomials each of degree
(D) 5 and – 5 (s) x2 – 10x + 21 5 is always 5.
50. Column-I Column-II
63. 3, –1, 1/3 are the zeroes of the cubic polynomial
(Polynomials) (Zeroes)
(A) 4 – x2 (p) 7 p(x) = 3x3 – 5x2 – 11x – 3.
x3 –
(B) 2x2 (q) –2
64. Zeroes of quadratic polynomial x2 + 7x + 10 are 2 and –5
(C) 6x2 – 3 – 7x (r) 3/2
(D) –x + 7 (s)  0 65. Sum of zeroes of 2x2 – 8x + 6 is – 4
M-14 Mathematics

ANSWER KEY & SOLUTIONS


1. (a) Since zeroes are reciprocal of each other, so product So, p(x) = 0 when x = 15 or x = –5. Therefore
k+2 required zeroes are 15 and –5.
of the roots will be 1, so =1,
k2 9. (c) We have, sum of zeroes
k2 – k – 2 = 0 ⇒ (k – 2)(k + 1) = 0
( −4)
= a + b = − =2
k = 2, k = –1, Since k > 0 ∴ k = 2 2
3
5 Product of zeroes = ab =
2. (c) Let α, β be two zeroes of 2x2 – 8x – m, where a = . 2
2
3

( −Coefficient of x ) ∴ a2b + ab2 = ab (a + b) =
2
×2 = 3
a+b=
Coefficient of x 2
10. (b) x2 – (m +3)x + mx – m(m + 3) = 0
5 8 ⇒ x[x – (m + 3)] + m[x – (m + 3)] = 0
⇒ +b=
2 2
⇒ (x + m) [x – (m + 3)] = 0
8 5 3
⇒ b= − = . \ x + m = 0   x – (m + 3) = 0
2 2 2
3. (d) Substitute y = 1 x = –m x=m+3

4. (a) Substitute x = 3 in polynomial ax3 + 4x2 + 3x – 4 and 11. (d)


The expressions (x – 1) (x + 1) and (x – 1) (x – 1)
x3 – 4x + a to obtain remainder and equate. which vanish if x = 1.

a(3)3 + 4(3)2 + 3(3) – 4 = (3)3 – 4(3) + a ⇒ a = –1 12. (b) 10 x = 7.7 or x = 0.7

5. (b) Equate value of polynomial at x = a with a 7


Subtracting, 9x = 7  \  x =
9
a2 – 27a + 196 = a ⇒ a2 – 28a + 196 = 0 ⇒ a = 14
14
2x = = 1.555........ = 1.5
6. (b) Substitute x = 1 in f (x) and x = –2 in g(x), and add 9
f (1) = 2(1) – 6(1) + 4(1) – 5 = –5 ⇒ g(–2) = 3(4) – 9 x2 + 4x + 2 = (x2 + 4x + 2) – 2 = (x + 2)2 – 2
13. (b)
=3
Lowest value = – 2 when x + 2 = 0
f (1) + g(–2) = –2
14. (d) Put a = b in given polynomial. Remainder comes to be
7. (a) x3 – 3x2 – 10x + 24 0.
∵ Last term = (product of roots) 15. (c)
∴ Factorising 24 = 2 × 4 × 3 16. (b) P(x) = ax3 + 4x2 + 3x – 4
Also sum of roots must be “3” P(3) = 27a + 36 + 9 – 4 = 27a + 41
∴ Possible factors are (2, 4, –3) P(x) = x3 – 4x + a; P(3) = 27 – 12 + a = 15 + a
∴ Factorization of x3 – 3x2 – 10x + 24 \ 27a + 41 = 15 + a ⇒ a = –1

= (x – 2) (x + 3) (x – 4) 17. (a) x = – 1 is the root of the quadratic polynomial p(x)

8. (c) We have, p(x) = x2 –10x –75 = x2 – 15x + 5x – 75 So, quadratic polynomial p(x) = k(x + 1)2
p(–2) = k(–2 + 1)2 = 2 ⇒ k = 2 ∴ p(x) = 2(x + 1)2
= x (x – 15) + 5 (x –15) = (x – 15) (x + 5)
Also, p(2) = 2(2 + 1)2 = 2 × 3 × 3 = 18
∴ p(x) = (x –15) (x + 5)
Polynomials M-15
18. (c) x2 – 4 = (x – 2)(x + 2) are the factors 24. (b) Coefficient of all the terms are positive. So, both
roots will be negative.
∴ x = 2, –2 are roots of polynomial
25. (d) In (a) power of x is –1 i.e. negative
∴ at x = 2; P(2) = 2(2)3 + k1(2)2 + k2(2) + 12 = 0
\ (a) is not true.
⇒ 16 + 4k1 + 2k2 + 12 = 0 ⇒ 2k1 + k2 = –14...(i)
at x = 2; P(–2) = 2(–2)3 + k1(–2)2 + k2(–2) + 12 = 0 1
In (b) power of x = , not an integer. \ (b) is not
2
⇒ –16 + 4k1 – 2k2 + 12 = 0 true

⇒ 2k1 – k2 = 2 ...(ii) In (c) Here also power of x is not an integer \ (c) is


not true
From (i) & (ii), k1 = –3 ∴ k1 + k2 = –11
(d) holds [Q all the powers of x are non-negative
19. (b) x = 3 + 32/3 + 31/3 integers.]
3
 2 1 26. (b)
3
(x – 3) =  3 + 33 
3
  (a) is not true [By def.]
(b) holds
x3 – 27 – 9x2 + 27x = 32 + 3 + 3 × 32/3 × 31/3 (32/3 + [Q degree of a zero polynomial is not defined]
1/3
3 ) (c) is not true
x3 – 27 – 9x2 + 27x – 9 – 3 = 9(x – 3) [Q degree of a constant polynomial is ‘0’]
(d) is not true
x3 – 39 – 9x2 + 27x – 9x + 27 = 0
[Q a polynomial of degree n has at most n zeroes].
x3 – 9x2 + 18x – 12 = 0
27. (c)
20. (a) P(x) is a polynomial of degree 3.
1
x2 +
(a) = x 2 + x –1 is not a polynomial since the
1 x
and P(n) = ⇒ n P(n) – 1 = 0
n exponent of variable in 2nd term is negative
n(P(n)) is a polynomial of degree 4 1

\ n P(n) – 1 = k(n – 1)(n – 2)(n – 3)(n – 4) (b) 2 x 2 – 3 x + 1 = 2 x 2 – 3 x 2 + 1 is not a polynomial,


since the exponent of variable in 2nd term is a
−1 rational number.
For n = 0; –1 = 24 k ⇒ k =
24
x3 – 3x + 1 is a polynomial.
(c)
−1
For n = 5; 5 × P(5) – 1 = (4)(3)(2)(1) 3
24
2 x 2 – 5 x is also not a polynomial, since the
(d)
⇒ 5 ⋅ P(5) – 1 = – 1 ⇒ P(5) = 0 exponents of variable in 1st term is a rational
number
f(x) = (x – 1)2 + (x – 2)2 + (x – 3)2 + (x – 4)2
21. (d)
2 Hence, (a), (b) and (d) is not a polynomial.
 5
= 4  x −  + 5 28. (d)
 2
5 (a) It is quadratic polynomial
f(x) is minimum at x = = 2.5
2 [∵ the graph meets the x-axis in two points]
22. (b) Since, 2 is the zero of x2 + 3x + k, (b) It is a quadratic polynomial
[∵ the graph meets the x-axis in two points]
\ (2)2 + 3(2) + k = 0  ⇒  k + 10 = 0 ⇒ k = – 10
(c) It is a quadratic polynomial
23. (a) Since –3 is the zero of (k –1) x2 + kx + 1, [∵ the graph meets the x-axis in two points]
\ (k – 1) (–3)2 + k(–3) + 1 = 0 (d) It is a not quadratic polynomial
4 [∵ the graph meets the x-axis in one point]
⇒ 9k – 9 – 3k + 1 = 0 ⇒ 6k – 8 = 0 ⇒ k =
3
M-16 Mathematics
29. (c) a ≠ 0, a, b, c are real numbers x3 – (– 3 – 1 + 2) x2 + ((–3)(–1) + (–1)(2) + (2)(–3))x
30. (d) For roots are equal – (–3)(–1)(2)
b2 – 4ac = 0 x3 + 2x2 + (3 – 2 – 6)x – 6
or D = 0 x3 + 2x2 – 5x – 6
31. (b) For value of k,
44. (b) Reason is true.
1 c c
α. = (Product of roots = ) 1
α a a Let α, be the zeroes of p(x), then
α
8k
1=
2 1 4k 4k
α. = 2 ⇒1= 2
1 α k +4 k +4
k=
or
4
\ k2 – 4k + 4 = 0
32. (c) For x2 + 1 = 0
roots are not real. ⇒ (k – 2)2 = 0 \ k = 2

So, graph of x2 + 1 = 0, neither touches nor intersects \ Assertion is true.


x-axis.
Since, reason is not correct explanation for assertion.
33. (c) We know, for a quadratic polynomial
45. (c) Reason is false [\a polynomial of nth degree has at
k(x2 – (Sum of roots) x + Product of roots)
most x zeroes.]
k(x2 – (–p) + (–1/p))
k (x2 + p – 1/p) Again, x3 + x = x (x2 + 1)
34. (d) Parabola. which has only one real zero (x = 0)
35. (c) a < 0, Graphs look like
[Q x2 + 1 ≠ 0 for all x ∈ R]
open downwards
\ Assertion is true.
36. (c) According to graph, there are two zeros
one at (–2) and 2nd at 4, –2, 4 46. (b)
37. (b) –2, 4 47. (c)
−b ± b 2 − 4ac
38. (b) For zeros D = 48. (d)
2a
Here, a = 4 3, b = 5, c = −2 3 49. (A) → q; (B) → s; (C) → r; (D) → p
−5 ± 25 + 4 × 4 3 × 2 3 −5 ± 11 50. (A) → q ; (B) → s ; (C) → r ; (D) → p
= =
8 3 8 3 (A) 4 – x2 = 0 ⇒ x = ±2
−2 3 x3 – 2x2 = 0 ⇒ x2(x – 2) = 0
(B)
⇒ ,
3 4
x = 0 or x = 2
39. (d) Parabola
(C) 6x2 – 7x – 3 = 0 ⇒ 6x2 – 9x + 2x – 3 = 0 
40. (c) If a > 0, Graph of parabola looks like
41. (d) Here graph cuts x-axis at 3 points 3x(2x – 3) + 1(2x – 3) = 0 ⇒ (3x + 1) (2x – 3) = 0
so it has three zeros. x = 3/2 or x = – 1/3
42. (c) Observing the graph we find –3, –1, 2 as zeros.
x=7
(D)
43. (a) Given zeros are –3, –1, 2, then
Expression is (x – (–3)) (x – (–1)) (x – 2) 51. linear, quadratic, cubic

= (x + 3)(x + 1)(x – 2) 52. x


= x3 + 2x2 – 5x – 6 53. 3
x3 – (Sum of zeros)x2 + (Sum of zeros taking two at a
54. a, a
time)x – (Product of zeros)
Polynomials M-17
55. zero 61. False, a polynomial can have any number of zeroes. It
depends upon the degree of the polynomial.
56. n
62. False, x5 + 1 and – x5 + 2x + 3are two polynomials of
57. True
degree 5 but the degree of the sum of the two polynomials
58. False, because the exponent of the variable is not a whole is 1.
number.  1
63. True, p(3) = 0, p(–1) = 0, p   = 0
3 3
6 x + x2 64. False
59. True, because = 6 + x, which is a polynomial.
x Coefficient of x −( −8)
60. False 65. False, sum of zeroes = = =4
Coefficient of x 2 2
Pair of Linear
3 Acids, Bases and
Equations in Two
Salts
Variables
5. Which of the following is the other name for a pair of
Multiple Choice Questions (MCQs) linear equations in two variables?
(a) Consistent equations (b) Simultaneous equations
DIRECTIONS : This section contains multiple choice
(c) Inconsistent equations (d) Dependent equations
questions. Each question has 4 choices (a), (b), (c) and (d) out
of which only one is correct. 6. The difference between two numbers is 26 and one number
is three times the other. Find them.
1.
For which value of p, will the lines represented by the
(a) 39, 13 (b) 41, 67
following pair of linear equations be parallel
(c) 96, 70 (d) 52, 26
3x – y – 5 = 0
6x – 2y – p = 0 7. Sanjay starts his job with a certain monthly salary and
earns a fixed increment every year. If his salary was
(a) all real values except 10 ` 4500 after four years of service and ` 5400 after 10 years,
(b) 10 find his initial salary and annual increment.
(c) 5/2 (a) 4000, 200 (b) 3900, 150
(d) 1/2 (c) 4500, 100 (d) 3800, 250

2. For what values of k will the following pair of linear 8. The pair of linear equations x + 2y = 5 and 3x + 12y = 10 has
equations have infinitely many solutions? (a) unique solution
kx + 3y – (k – 3) = 0 (b) no solution
12x + ky – k = 0 (c) more than two solutions
k = 4
(a) (b) k = 3 (d) infinitely many solutions
k = 6
(c) (d) k = 2 9. ` 49 was divided among 150 children. Each girl got 50 paise
3.
Which of the following is true if following pair of linear and each boy got 25 paise. How many boys were there?
equations has unique solution? (a) 100 (b) 102
3x – 2y = – 8 (c) 104 (d) 105
(2m – 5)x + 7y – 6 = 0 10. If the sum of the ages (in years) of a father and his son is
11 11 65 and twice the difference of their ages (in years) is 50,
m=
(a) (b) m = – what is the age of the father?
4 4
(a) 45 years (b) 40 years
11 11
m≠–
(c) (d) m≠ (c) 50 years (d) 55 years
4 4
4. The perimeter of a rectangle is 40 cm. The ratio of its sides 11. The value of k for which the system of linear equations
is 2 : 3. Find its length and breadth. x + 2y = 3, 5x + ky + 7 = 0 is inconsistent is
(a) l = 10 cm, b = 8 cm (b) l = 12 cm, b = 8 cm 14 2
(a) − (b)
3 5
l = 12 m, b = 8 m
(c) (d) l = 40 m, b = 30 m
(c) 5 (d) 10
Pair of Linear Equations in Two Variables M-19
12. A can do a piece of work in 24 days. If B is 60% more (a) 4 : 1 (b) 1 : 4
efficient than A, then the number of days required by B to (c) 7 : 1 (d) 1 : 7
do the twice as large as the earlier work is
21. In a number of two digits, unit’s digit is twice the tens digit.
(a) 24 (b) 36
If 36 be added to the number, the digits are reversed. The
(c) 15 (d) 30 number is
13. X’s salary is half that of Y’s. If X got a 50% rise in his (a) 36 (b) 63
salary and Y got 25% rise in his salary, then the percentage (c) 48 (d) 84
increase in combined salaries of both is
1 22. A man can row a boat in still water at the rate of 6 km per
(a) 30 (b) 33 hour. If the stream flows at the rate of 2 km/hr, he takes
3
1 half the time going downstream than going upstream the
(c) 37 (d) 75 same distance. His average speed for upstream and down
2
stream trip is
14. The points (7, 2) and (–1, 0) lie on a line
(a) 6 km/hr
(a) 7y = 3x – 7 (b) 4y = x + 1
(b) 16/3 km/hr
y = 7x + 7
(c) (d) x = 4y + 1
(c) Insufficient data to arrive at the answer
15. At present ages of a father and his son are in the ratio 7 : 3,
and they will be in the ratio 2 : 1 after 10 years. Then the (d) none of the above
present age of father (in years) is 23. A boat travels with a speed of 15 km/hr in still water. In
(a) 42 (b) 56 a river flowing at 5 km/hr, the boat travels some distance
(c) 70 (d) 77 downstream and then returns. The ratio of average speed
to the speed in still water is
16. A fraction becomes 4 when 1 is added to both the numerator
(a) 8 : 3 (b) 3 : 8
and denominator and it becomes 7 when 1 is subtracted
from both the numerator and denominator. The numerator (c) 8 : 9 (d) 9 : 8
of the given fraction is 24. x and y are two non-negative numbers such that 2x + y = 10.
(a) 2 (b) 3 The sum of the maximum and minimum values of (x + y)
is
(c) 5 (d) 15
(a) 6 (b) 9
17. A motor boat takes 2 hours to travel a distance 9 km
down the current and it takes 6 hours to travel the same (c) 10 (d) 15
distance against the current. The speed of the boat in still 25. The average incomes of the people in two villages are P
water and that of the current (in km/hour) respectively and Q respectively. Assume that P ≠ Q. A person moves
are from the first village to the second village. The new average
(a) 3, 1.5 (b) 3, 2 incomes are P’ and Q’ respectively. Which of the following
is not possible?
(c) 3.5, 2.5 (d) 3, 1
P’ > P and Q’ > Q (b)
(a) P’ > P and Q’ < Q
18. The 2 digit number which becomes (5/6)th of itself when
its digits are reversed. The difference in the digits of the P’ = P and Q’ = Q (d)
(c) P’< P and Q’ < Q
number being 1, then the two digits number is 26. The graphs of the equations x – y = 2 and kx + y = 3,
(a) 45 (b) 54 where k is a constant, intersect at the point (x, y) in the
(c) 36 (d) None of these first quadrant, if and only if k is
(a) equal to –1 (b) greater than –1
19. x and y are 2 different digits. If the sum of the two digit
numbers formed by using both the digits is a perfect square, (c) less than 3/2 (d) lying between –1 and 3/2
then value of x + y is 27. For what value of p, the following pair of linear equations
(a) 10 (b) 11 in two variables will have infinitely many solutions ?
(c) 12 (d) 13 px + 3y – (p – 3) = 0, 12x + py – p = 0
20. If 3x + 4y : x + 2y = 9 : 4, then 3x + 5y : 3x – y is equal (a) 6 (b) – 6
to (c) 0 (d) 2
M-20 Mathematics
28. In a classroom, one-fifth of the boys leave the class and 36. If x = a, y = b is the solution of the equations x – y = 2 and
the ratio of the remaining boys to girls is 2 : 3. If further x + y = 4, then the values of a and b are, respectively.
44 girls leave the class, then the ratio of boys to girls is 5: (a) 3 and 5 (b) 5 and 3
2. How many more boys should leave the class so that the
(c) 3 and 1 (d) – 1 and – 3
number of boys equals that of girls?
5
(a) 16 (b) 24 37. I. If x – y = xy = 1 – x – y, then x + y is
3
(c) 30 (d) 36 II. The system of equations 3x + 2y = a and 5x + by
1 1 = 4 has infinitely many solutions for x and y, then
1 1
29. The equations + = 15 and − = 5 are such that a = 4, b = 3
x y x y
x y
ax = 1 and by = 1. The values of ‘a’ and ‘b’ respectively are III. If+ = 2 and ax – by = a2 – b2, then x = a, y = b
a b
(a) 10, 5 (b) 10, –5
Which is true?
(c) –5, 10 (d) 5, 10
(a) I only (b) II only
30. Consider the following two statements: (c) III only (d) None of these.
I. Any pair of consistent linear equations in two variables 38. I. If 3x – 5y = –1 and x – y = – 1, then x = –2, y = –1
must have a unique solution.
II. 2x + 3y = 9, 3x + 4y = 5 ⇒ x = –21, y = 17
II. There do not exist two consecutive integers, the sum
2x y x y
of whose squares is 365. III. + = 2, – = 4 ⇒ x = 2a, y = 2b
a b a b
Then,
Which is true?
(a) both I and II are true (b) both I and II are false
(a) I (b) II
(c) I is true and II is false (d) I is false and II is true
(c) III (d) None of these
31. In village Madhubani 8 women and 12 girls can paint a
39. If a pair of linear equations is inconsistent, then the lines
large mural in 10 hours. 6 women and 8 girls can paint it
will be
in 14 hours. The number of hours taken by 7 women and
14 girls to paint the mural is (a) parallel (b) always coincident
(a) 10 (b) 15 (c) intersecting (d) coincident
(c) 20 (d) 35 40. For what values of k, do the equations 3x – y + 8 = 0 and
32. A boat takes 3 hours to travel 30 km downstream and takes 6x – ky = –16 represent coincident lines?
5 hours to return to the same spot upstream. Find the speed (a) solution of 3k – 9 = 0 (b) solution of 2k – 8 = 0
of the boat in still water. (km/hr)
(c) 2 (d) 3
(a) 10 km/hr (b) 8 km/hr
(c) 6 km/hr (d) 5 km/hr
24
33. The pair of equations 5x – 15y = 8 and 3 x – 9 y = has
5 DIRECTIONS : Study the given Case/Passage and answer the
(a) one solutio
following questions.
(b) two solutions
(c) infinitely many solutions Case/Passage-I
A test consists of ‘True’ or ‘False’ questions. One mark is
(d) no solution
awarded for every correct answer while 1/4 mark is deducted
34. The sum of the digits of a two-digit number is 9. If 27 for every wrong answer. A student knew answers to some of the
is added to it, the digits of the number get reversed. The questions. Rest of the questions he attempted by guessing. He
number is answered 120 questions and got 90 marks.
(a) 25 (b) 72 Type of Question Marks given for Marks deducted for
(c) 63 (d) 36 correct answer wrong answer
True/False 1 0.25
35. The value of c for which the pair of equations cx – y = 2
[From CBSE Question Bank-2021]
and 6x + 2y = 3 will have infinitely many solutions is
41. If answer to all questions he attempted by guessing were
(a) 3 (b) – 3 wrong, then how many questions did he answer correctly?
(c) – 12 (d) no value 42. How many questions did he guess?
Pair of Linear Equations in Two Variables M-21
43. If answer to all questions he attempted by guessing were Refer situation 1
wrong and answered 80 correctly, then how many marks 50. If the fixed charges of auto rickshaw be ` x and the
he got? running charges be ` y km/hr, the pair of linear equations
44. If answer to all questions he attempted by guessing were representing the situation is
wrong, then how many questions answered correctly to (a) x + 10y = 110, x + 15y = 75
score 95 marks?
(b) x + 10y = 75, x + 15y = 110
Case/Passage-II
(c) 10x + y = 110, 15x + y = 75
Amit is planning to buy a house and the layout is given below.
The design and the measurement has been made such that areas (d) 10x + y = 75, 15x + y = 110
of two bedrooms and kitchen together is 95 sq.m. 51. A person travels a distance of 50km. The amount he has
x 2 y to pay is
(a) ` 155 (b) ` 255 (c) ` 355 (d) ` 455
5m Bath
Bedroom 1 room Kitchen Refer situation 2
52. What will a person have to pay for travelling a distance
2m of 30km?
Living Room (a) ` 185 (b) ` 289 (c) ` 275 (d) ` 305
5m Bedroom 2 53. The graph of lines representing the conditions are:
(situation 2)
Y
15 m 25 (20, 25)
[From CBSE Question Bank-2021] 20
Based on the above information, answer the following questions: 15
45. Form the pair of linear equations in two variables from (a) 10
this situation. 5 (0, 5) (30, 5)
46. Find the length of the outer boundary of the layout.
47. Find the area of each bedroom and kitchen in the layout. X¢ –5 0 5 10 15 20 25 30 35 X
–5
48. Find the area of living room in the layout.
–10
49. Find the cost of laying tiles in kitchen at the rate of ` 50 Y¢
per sq.m Y
25
Case/Passage-III
20
It is common that Governments revise travel fares from time to
15
time based on various factors such as inflation ( a general increase
in prices and fall in the purchasing value of money) on different (b) 10 (0, 10) (20, 10)
types of vehicles like auto, rickshaws, taxis, radio cab etc. The auto 5 (12.5, 0)
charges in a city comprise of a fixed charge together with the charge
X¢ –5 0 5 10 15 20 25 30 35 X
for the distance covered. Study the following situations. –5
(5, –10) (25, –10)
–10

50
45
40
35
Name of the city Distance travelled (km) Amount paid (`)
30
City A 10 75
25
15 110
20
City B 8 91 15 (11, 10) (19, 9)
14 145 (47, 7)
(c) 10 (5, 10)
Situation 1: In city A, for a journey of 10 km, the charge paid 5 (27, 8)
is ` 75 and for a journey of 15 km, the charge paid is ` 110. 0 5 10 15 20 25 30 35 40 45 50 55
Situation 2: In a city B, for a journey of 8 km, the charge paid
is ` 91 and for a journey of 14km, the charge paid is ` 145.
[From CBSE Question Bank-2021]
M-22 Mathematics
Y 58. Assertion : If kx – y – 2 = 0 and 6x – 2y – 3 = 0 are
25 inconsistent, then k = 3
20
Reason : a1x + b1y + c1 = 0 and a2x + b2y + c2 = 0 are
15 (15, 15)
(35, 10) a1 b1 c1
(d)
10 inconsistent if = ≠
5 (0, 10) a2 b2 c2
59. Assertion : 3x – 4y = 7 and 6x – 8y = k have infinite
X¢ –5 0 5 10 15 20 25 30 35 X number of solution if k = 14
–5 (15, –5)
–10
Y¢ Reason : a1x + b1y + c1 = 0 and a2x + b2y + c2 = 0 have
a1 b1
a unique solution if ≠
Assertion & Reason a2 b2
60. Assertion : The linear equations x – 2y – 3 = 0 and
DIRECTIONS : Each of these questions contains an Assertion 3x + 4y – 20 = 0 have exactly one solution
followed by Reason. Read them carefully and answer the Reason : The linear equations 2x + 3y – 9 = 0 and
question on the basis of following options. You have to select 4x + 6y – 18 = 0 have a unique solution
the one that best describes the two statements.
Match the Following
(a) If both Assertion and Reason are correct and Reason is
the correct explanation of Assertion.
DIRECTIONS : Each question contains statements given in
(b) If both Assertion and Reason are correct, but Reason is
two columns which have to be matched. Statements (A, B, C, D)
not the correct explanation of Assertion. in column-I have to be matched with statements (p, q, r, s) in
(c) If Assertion is correct but Reason is incorrect. column-II.
(d) If Assertion is incorrect but Reason is correct.
61. Column-II give value of x and y for pair of equation given
54. Assertion : 3x + 4y + 5 = 0 and 6x + ky + 9 = 0 in Column-I.
represent parallel lines if k = 8 Column-I Column-II
Reason : a1x + b1y + c1 = 0 and a2x + b2y + c2 = 0 (A) 2x + y = 8, x + 6y = 15 (p) (3, 4)
(B) 5x + 3y = 35, 2x + 4y = 28 (q) (1/14, 1/6)
a1 b1 c1
represent parallel lines if = ≠ 1 1 1 1
a2 b2 c2 (C) + = 3, − = 5 (r) (4, 5)
7x 6 y 2x 3y
55. Assertion : x + y – 4 = 0 and 2x + ky – 3 = 0 has no solution (D) 15x + 4y = 61 (s) (3, 2)
if k = 2
4x + 15y = 72
Reason : a1x + b1y + c1 = 0 and a2x + b2y + c2 = 0 are 62. Column-I Column-II
a1 b1 (A) 5y – 4x = 14, y – 2x = 1 (p) Infinite solutions
consistent if ≠
a2 b2 (B) 6x – 3y + 10 = 0, (q) Consistent
56. Assertion : If the system of equations 2x + 3y = 7 and 2x – y + 9 = 0
2ax + (a + b) y = 28 has infinitely many solutions, then (C) 3x – 2y = 4, 9x – 6y = 12 (r) No solution
2a – b = 0
(D) 2x – 3y = 8, 4x – 6y = 9 (s) Inconsistent
Reason : The system of equations 3x – 5y = 9 and
63. Column-I Column-II
6x – 10y = 8 has a unique solution.
24
(A) No solution (p) 5x – 15y = 8, 3x – 9y =
57. Assertion : If the pair of lines are coincident, then we 5
say that pair of lines is consistent and it has a unique
solution. (B) Infinitely many solutions (q) 2x + 4y = 10, 3x + 6y = 12

Reason : If the pair of lines are parallel, then the (C) Unique solution (r) x + y = 6, x – y = 2
pair has no solution and is called inconsistent pair of (D) Integral solution (s) 2x + y = 6, 4x – 2y – 2 = 0
equations.
Pair of Linear Equations in Two Variables M-23

Fill in the Blanks True / False

DIRECTIONS : Complete the following statements with an DIRECTIONS : Read the following statements and write your
appropriate word / term to be filled in the blank space(s). answer as true or false.
64. If the lines intersect at a point, then that point gives the 72. If a pair of linear equations is given by a1x + b1y + c1 = 0
unique solution of the two equations. In this case, the pair
a1 b1
of equations is ................. and a2x + b2y + c2 = 0 and ≠ . In this case, the pair
a2 b2
65. If the lines are parallel, then the pair of equations
has no solution. In this case, the pair of equations of linear equations is consistent.
is .............. 73. If a pair of linear equations is given by a1x + b1y + c1 = 0 and
66. Two distinct natural numbers are such that the sum of one a1 b1 c1
a2x + b2y + c2 = 0 and = ≠ . In this case, the pair
number and twice the other number is 6. The two numbers a2 b2 c2
are .............. of linear equations is consistent.
67. If p + q = k, p – q = n and k > n, then q is ................... 74. If a pair of linear equations is given by a1x + b1y + c1 = 0 and
(positive/negative).
a1 b1 c1
68. Sum of the ages of X and Y, 12 years, ago, was 48 years a2x + b2y + c2 = 0 and = = . In this case, the pair
a2 b2 c2
and sum of the ages of X and Y, 12 years hence will be
96 years. Present age of X is .............. of linear equations is consistent.

69. The number of common solutions for the system 75. 3x – y = 3, 9x – 3y = 9 has infinite solution.
of linear equations 5x + 4y + 6 = 0 and 10x + 8y = 12
76. 2x + 3y = 0 , 3 x − 8 y = 0 has no solution.
is .............
77. 3x + 2y = 5, 2x – 3y = 7 are consistent pair of equation.
70. If 2x + 3y = 5 and 3x + 2y = 10, then x – y = ............... .
1 1 1 1 78. In a ∆ ABC, ∠C = 3 ∠B = 2 (∠ A + ∠ B), then angles are
71. If + = k and – = k , then the value of y is ........... 20°, 40°, 100°.
x y x y
M-24 Mathematics

ANSWER KEY & SOLUTIONS


1. (a)
If the lines are parallel, then on solving, x = 12, y = 8
a1 b1 c1 ∴ length = 12 cm and breadth = 8cm.
a = b ≠ c 5. (b) The pair of linear equations in two variables is also
2 2 2
known as simultaneous equations.
Here, a1 = 3, b1 = – 1, c1 = –5,
6. (a) Let the two numbers be x and y (x > y). Then,
a2 = 6, b2 = – 2, c2 = – p x – y = 26 ... (i)
3 −1 −5 x = 3y ... (ii)
⇒ = ≠  ... (i)
6 −2 − p Substituting value of x from equation (ii) in (i)
Taking II and III part of equation (i), we get 3y – y = 26 ⇒ 2y = 26  ⇒  y = 13
1 −5 Substituting value of y in equation (ii), x = 3 × 13 = 39
⇒ ≠ ⇒ − p ≠ −10   ⇒  p ≠ 10
2 −p Thus, two numbers are 13 and 39.
So, option (a) is correct. 7. (b) Let the annual increment be ` y and initial salary be ` x
a1 k b 3 c k −3 ∴ x + 4y = 4500 ... (i)
2. (c)
Here, = , 1 = , 1 =
a2 12 b2 k c2 k x + 10y = 5400
and ... (ii)
For a pair of linear equations to have infinitely many Solving eqs. (i) and (ii), we get
solutions: x = 3900 and y = 150
a1 b1 c1 ∴ Initial salary = ` 3900
= =
a2 b2 c2 and increment = ` 150
k 3 k −3 k 3 8. (a) The pair of linear equations are
So, we have = = or = which gives
12 k k 12 k x + 2y – 5 = 0 and
k2 = 36 i.e., k = ± 6 3x + 12y – 10 = 0
3 k −3 Here, a1 = 1, b1 = 2, c1 = – 5
Also, = gives 3k = k2 – 3k, i.e., 6k = k2,
k k and a2 = 3, b2 = 12,
which means k = 0 or k = 6. a1 1 b1 2 1
c2 = – 10. Now, = , = = .
Therefore, the value of k that satisfies both the a2 3 b2 12 6
conditions, is k = 6. For this value, the pair of linear a1 b1
equations has infinitely many solutions. As ≠
a2 b2
3. (c) For a pair of linear equations having unique solution So, pair of equations has a unique solution.
a1 b1 3 −2
≠ ⇒ ≠ 9. (c) Let the no. of girls be ‘x’ and the no. of boys be ‘y’.
a2 b2 2m − 5 7
Given, 0.50x + 0.25y = 49
or – 4m + 10 ≠ 21
and x + y = 150 ...(i)
or – 4m ≠ 11
x x
11 ⇒ + = 49 ...(ii)
or m ≠ − 2 4
4
From (i) & (ii), x = 46, y = 104
4. (b) Let length and breadth be x cm and y cm respectively.
Hence, number of boys (y) = 104
According to problem,
2 (x + y) = 40 ... (i) 10. (a) Let the age of father be ‘x’ years and the age of son
be ‘y’ years
y 2
and = ... (ii)
x 3 According to question, x + y = 65 ...(i)
Pair of Linear Equations in Two Variables M-25
and 2(x – y) = 50 ⇒ x – y = 25 ...(ii) 9
and time (t2) = = 6 (for up = rate) ... (ii)
Adding eqs. (i) and (ii), we get, 2x = 90 ⇒ x = 45 x− y
Hence, the age of father = 45 years Solving equations (i) & (ii), we get
x = 3 km/hr and y = 1.5 km/hr
1 2 −3 Q For inconsistent 
11. (d) = ≠  
5 k 7  a1 b1 c1  Speed of the boat = 3 km/hr
= ≠
 a2 b2 c2  Speed of the current = 1.5 km/hr
⇒ k = 10
18. (b) If the two digits are x and y, then the number is 10x + y.
12. (d)
Work ratio of A : B = 100 : 160 or 5 : 8
5
∴ time ratio = 8 : 5 or 24 : 15 Given that, (10x + y) = 10y + x. Solving it,
6
x 5
If A takes 24 days, B takes 15 days. Hence, B takes we get 44x + 55y ⇒ = .
30 days to do double the work. y 4
A Also x – y = 1. Solving them, we get x = 5 and y = 4.
13. (b)
Let salary of Y be = A and of X is = Therefore, number is 54.
2
3A 19. (b)
The numbers that can be formed are xy and yx. Hence
∴ Total salary of X and Y = ... (i)
2 (10x + y) + (10y + x) = 11(x + y). If this is a perfect
Let X’ and Y ’ be the new salary after increment, then square then x + y = 11.
we get
3x + 4 y 9
3A 5A 20. (c) =
X ' = and Y ' = ⇒ X '+ Y ' = 2A ... (ii) x + 2y 4
4 4
 2 A − 3A ⇒ 4(3x + 4y) = 9(x + 2y)
  × 100
2  Hence, 12x + 16y = 9x + 18y or 3x = 2y
∴ Required percentage increase =
3A
2
2 \ x= y.
3
[from (i) & (ii) eqns.] 2
1 1 Substitute x = y in the required expression.
= × 100 ⇒ 33 % 3
3 3
i.e. 3 x + 5 y : 3 x − y
14. (b) The point satisfy the line 4y = x + 1 2  2 
= 3  y  + 5 y : 3  y  − y
15. (c) Let the ages of father and son be 7x, 3x 3  3 
After 10 years, = 2y + 5y : 2y – y
\ (7x + 10) : (3x + 10) = 2 : 1 or x = 10 = 7y : y = 7 : 1
\ Age of the father is 7x i.e. 70 years.
21. (c) Let unit’s digit : x, tens digit : y
x
16. (d)
Let the fraction be then x = 2y, number = 10y + x
y
According to given conditions, Also 10y + x + 36 = 10x + y

x +1 ∴ 9x – 9y = 36 or x – y = 4
= 4 ... (i)
y +1 Solve, x = 2y, x–y=4

x −1 Substitute x = 2y in x – y = 4
and = 7 ... (ii)
y −1 we get, 2y – y = 4 ⇒ y = 4
Solving (i) and (ii), we have x = 15, y = 3 and x = 8
i.e. numbers = 15 So, the number = 10y + x = 48
17. (a) Let speed of boat in still water = x km/hr 22. (b)
Upstream speed = 4 km/hr and time = x hrs.
and speed of stream = y km/hr Downstream speed = 8 km/hr and
According to question,
time taken = x/2 hrs.
9 4 x + 8 × x / 2 16
time (t1) = = 2 (for down = rate) ... (i) Hence average speed = = km/hr.
x+ y x+ x/2 3
M-26 Mathematics
23. (c) Let distance = d, Adding (i) and (ii), we have
d d 5
Time taken upstream = = kx + x = 5 ⇒ x (k + 1) = 5 ⇒ x =
15 − 5 10 k +1
d d Putting the value of x in equation (i), we have
Time taken downstream = = 5
15 + 5 20 −y=2
Hence, average speed k + 1

2d 2d × 20 40 5 5 − 2k − 2 3 − 2k
= = = km/hr ⇒ −2= y⇒ = y⇒ y=
d d 3d 3 k + 1 k +1 k +1
+
10 20 y should be positive as they intersect in 1st quadrant
40 Therefore, y > 0
Ratio = :15 = 40 : 45 = 8 : 9
3 3 − 2k 2k − 3
>0⇒ <0
24. (d) Given 2x + y = 10 k + 1 k +1
on adding y both sides, we get, 2x + y + y = 10 + y + – +
y –∞ –1 3 +∞
⇒ 2(x + y) = 10 + y ⇒ x + y = 5 + 2
2
Now, (x + y)max when y is maximum & maximum \ k should lie between – 1 and 3/2
value of y will be 10. (Q y = 10 – 2x) 27. (a) Condition for infinite many solutions.
So (x + y)max = 5 + 5 = 10 & (x + y)min when y = 0 p 3 p – 3  a1 b1 c1 
= =  = = 
\ minimum value of x + y = 5 12 p p  a2 b2 c2 

So, sum of (x + y)max & (x + y)min = 15 p2 = 36 ; p = {From I and II}
25. (c) Let the number of people in first and second village p2 – 3p = 3p {From II and III}
be x and y respectively. p=6
According to given condition, \ p = 6
average income of x people = P and 28. (b) Let the number of boys and girls in classroom is x
and y.
average income of y people = Q, where P ≠ Q
According to question
\ Total income of people in two villages are Px

x −x/5 2 4x 2 x 5
and Qy respectively. = ⇒ = ⇒ =  ...(i)
y 3 5y 3 y 6
When, one person moves from first village to second
village. x − x/5 5 4x 5
Also, = ⇒ =
Then, number of people in first village = x – 1 and in y − 44 2 5 (y − 44) 2
second village = y + 1. ⇒ 8x = 25y – 1100 ...(ii)
New average income = P′ and Q′ From Eqs. (i) and (ii), we get, x = 50, y = 60
(Total income = no. of persons × average income) Let n number of boy leaves the class so number of
boys and number of girls become equal.
\ Total income = P′(x – 1) and Q′(y + 1)
\ 50 – 10 – n = 60 – 44
Total income in both cases are same n = 40 – 16 = 24
\ Px + Qy = P′(x – 1) + Q′(y + 1) 1 1
29. (a) x = ⇒ a = 10 and y = ⇒ b = 5
⇒ Px – P′(x – 1) = Q′(y + 1) – Qy 10 5
⇒ x(P – P′) + P′ = y(Q′ – Q) + Q′ 30. (b) (I) Statement I is false. Consistent Linear equations
\ P′ ≠ P and Q′ ≠ Q may have unique or infinite solutions.
(II) Statement or is also false
Hence, option (c) is not possible.
\ Q 132 + 142 = 365
26. (d)
x – y = 2 .... (i)
31. (a) Let one woman can paint a large mural in W hours
kx + y = 3 .... (ii) and one girl can paint it in G hours
Pair of Linear Equations in Two Variables M-27
According to question, 3 –1 8
40. (c) For coincident lines, = =
8 12 1 2 3 1 6 – k 16
+ = fi + = ...(i)
W G 10 W G 40 1 1
= ⇒ k = 2
6 8 1 3 4 1 2 k
Also, + = fi + = ...(ii)
W G 14 W G 28 Sol. (41-44):
On solving equation (i) and (ii), we get Let x be number of known questions and y be number of
W = 140 and G = 280 questions cheating by the student.
7 14 1 1 Here, x + y = 120
Now, + = = (say)
140 280 Time taken t 1
x − y = 90
4
1 1 1
⇒ = + ⇒ t = 10 hours On solving these two equations
t 20 20
32. (b) Let speed of boat in still water be x km/hr We have, x = 96 and y = 24
and speed of stream be y km/hr 41. No. of correct questions are 96
30 42. He guessed 24 questions.
= 3  ⇒  x + y = 10 …(i)
x+y 1
43. Marks = 80 – of 40 = 70
4
30
= 5  ⇒  x – y = 6 …(ii) 44. Here, x + y = 120 ...(i)
x−y
1
From solving equations (i) and (ii) x − y = 95
4 ...(ii)
–x + y = 10 On solving (i) & (ii) x = 100
–x – y = 6
45. Given area of two bedrooms and a kitchen is 95 sq m.
  +  –
2 × Area of bedroom + Area of kitchen = 95
2y = 4   y = 2 km/hr. and
2 × 5 x + 5y = 95
x = 8 km/hr or 2x + y = 19 ...(i)
a1 b1 c1 5 and x + 2 + y = 15
33. (c) a = b = c = 3
2 2 2 or x + y = 13 ...(ii)
34. (d)
Let x & y be the unit and tenth digits respectively of 46. Length of outer boundary = 12 + 15 + 12 +15 = 54 m
a two digit number. Then,
47. On solving x + y = 13
x + y = 9 (∵ Given) ... (i)
2x + y = 19
and according to given condition,
10x + y = 10 y + x + 27 x = 6m, y = 7m
⇒ 9x – 9y = 27 Area of a bedroom = 5x = 5 × 6 = 30 sq m
⇒ x – y = 3 ... (ii) Area of kitchen = 5y = 5 × 7 = 35 sq m
On adding (i) & (ii) 48. Area of living room = 9 × 5 + 2 × 15 = 75 sq m
2x = 12 ⇒ x = 6 49. Total cost of laying tiles in the kitchen = ` 50 × 35 = ` 1750
Hence, from equation (i), 50. (b) Given, fixed charges of auto rickshaw be ` x and
6 + y = 9 ⇒ y = 3 running charges be ` y km/hr, so representing situation 1
So number will be 10 × 3 + 6 = 36 x + 10y = 75
−c −1 −2 x + 15y = 110
35. (d) For solution to be infinite, = = must satisfy. 51. (c) On solving x + 10y = 75
6 2 −3
−1 2  x + 15y = 110
but ≠ , so, infinite solution don’t exist, for given
2 3 we get x = 5 km,
equations.
 y = ` 7/km
36. (c) On adding both the equations, we get x = 3, y = 1 Charges to go 50 km.
37. (c) 38. (a) 39. (a) x + 50y = 5 + 50 × 7 = ` 355
M-28 Mathematics
52. (b) To cover 30 km distance, 57. (d) Assertion is clearly false.
x + 30y = 19 + 30 × 9 = 289 [Q If the lines are coincident, then it has infinite
53. (c) number of solutions]

54. (a) Reason is true. Reason is clearly true.


In assertion, given lines represent parallel lines if 58. (a) 59. (b) 60. (c)
3 4 5 6×4
= ≠ ⇒k= = 8 \ Reason is also true 61. (A) → (s); (B) → (r); (C) → (q); (D) → (p)
6 k 9 3
Also, reason is the correct explanation for assertion. 62. (A) → (q); (B) → (s); (C) → (p); (D) → (r)
55. (b) Reason is true. 63. (A) → (q); (B) → (p); (C) → (s); (D) → (r)
For assertion, given equation has no solution if 64. consistent 65. inconsistent.
1 1 –4 4 1 4
= ≠ i.e. ⇒ k = 2  ≠ holds  66. 4 and 1 67. positive
2 k –3 3 2 3 
Assertion is true. 68. Cannot be determined 69.
zero
Reason does not give result of assertion.
70. 5 71. Does not exist
56. (c) Assertion: given system of equations has infinitely
many solutions if 72. True 73. False 74. True
2 3 –7
= = 75. True 76. False 77. True
2a a + b –28
1 1 3 1
i.e. ⇒ = = ⇒ 3a = a + b ⇒ 2a – b = 0 78. False
4 a a+b 4
Also clearly a = 4, and a + b = 12 ⇒ b = 8
\ 2a – b = 8 – 8 = 0
\ Assertion is true
3 –5
But reason is false ∵ =
6 –10
[Q3(–10) = (–5)(6) = –30]
a1 b1
For unique solution if a1x + b2y + c2 = 0, then ≠
a2 b2
4 Acids, Bases and
Coordinate
Salts
Geometry
7. If P (x , y) is any point on the line joining the points A (a, 0)
Multiple Choice Questions (MCQs) and B (0, b), then
x y x y
DIRECTIONS : This section contains multiple choice (a) + = 1 (b) − = 1
questions. Each question has 4 choices (a), (b), (c) and (d) out b a a b
of which only one is correct. x y x y
(c) + = 1 (d) − = 1
a b b a
1. If P = (2, 5), Q = (x, –7) and PQ = 13, what is the value of
‘x’? 8. The perimeter of a triangle with vertices (0, 4),
(0, 0) and (3, 0)is
(a) 5 (b 3
(a) 5 (b) 12
(c) –3 (d) –5
(c) 11 (d) 7 + 5
2. The points (a, b), (a1, b1) and (a – a1, b – b1) are collinear
if 9. The point P on x-axis equidistant from the points A(–1, 0)
(a) ab = a1b1 (b) ab1 = a1b and B(5, 0) is
(c) a = b (d) a1 = b1 (a) (2, 0) (b) (0, 2)
(c) (3, 0) (d) (2, 2)
a 
3. If  , 4  is the midpoint of the line segment joining
3  10. The coordinates of the point which is reflection of point
A(–6, 5) and B(–2, 3), then what is the value of ‘a’? (–3, 5) in x-axis are
(a) –4 (b) –12 (a) (3, 5) (b) (3, –5)
(c) 12 (d) –6 (c) (–3, –5) (d) (–3, 5)
4. If the mid point of the line joining (3, 4) and (k, 7) is 11. If the point P(6, 2) divides the line segment joining
(x, y) and 2x + 2y + 1 = 0. Find the value of k. A(6, 5) and B(4, y) in the ratio 3 : 1, then the value
(a) 10 (b) –15 of y is
(c) 15 (d) –10 (a) 4 (b) 3 (c) 2 (d) 1
5. In what ratio is the line segment joining the points (3, 5) 12. P, Q, R are three collinear points. The coordinates of P
& (–4, 2) divided by y–axis? and R are (3, 4) and (11, 10) respectively and PQ is equal
(a) 3 : 2 (b) 3 : 4 to 2.5 units. Coordinates of Q are
(c) 2 : 3 (d) 4 : 3 (a) (5, 11/2) (b) (11, 5/2)
6. In what ratio does the point (–2, 3) divide the line-segment (c) (5, –11/2) (d) (–5, 11/2)
joining the points (–3, 5) and (4, –9) ? 13. C is the mid-point of PQ, if P is (4, x), C is (y, –1) and Q is
(a) 2 : 3 (b) 1 : 6 (–2, 4), then x and y respectively are
(c) 6 : 1 (d) 2 : 1 (a) – 6 and 1 (b) – 6 and 2
(c) 6 and – 1 (d) 6 and – 2
M-30 Mathematics
14. The ratio in which the point (2, y) divides the join of 23. P is a point on the graph of y = 5x + 3. The coordinates of
(– 4, 3) and (6, 3) and hence the value of y is a point Q are (3, –2). If M is the mid point of PQ, then M
must lie on the line represented by
(a) 2 : 3, y = 3 (b) 3 : 2, y = 4
y = 5x + 1
(a) (b) y = 5x – 7
(c) 3 : 2, y = 3 (d) 3 : 2, y = 2
5 7 5 1
15. Ratio in which the line 3x + 4y = 7 divides the line segment y = x –
(c) (d) y = x+
2 2 2 2
joining the points (1, 2) and (–2, 1) is
24. A line l passing through the origin makes an angle q
(a) 3 : 5 (b) 4 : 6 3
with positive direction of x-axis such that sin θ = . The
(c) 4 : 9 (d) None of these 5
coordinates of the point, which lies in the fourth quadrant
16. The point on the X-axis which is equidistant from the points at a unit distance from the origin and on perpendicular to
A(–2, 3) and B(5, 4) is l, are

(a) (0, 2) (b) (2, 0)  3 4  4 3


 ,− 
(a) (b)  ,− 
5 5 5 5
(c) (3, 0) (d) (–2, 0)
(c) (3, –4) (d) (4, –3)
17. The point which divides the line joining the points A (1, 2)
25. The centre of the circle passing through the ponts (6, – 6),
and B(–1, 1) internally in the ratio 1 : 2 is (3, – 7) and (3, 3) is
 –1 5  1 5 (a) (3, 2) (b) (–3, –2)
(a)
 ,  (b)
 , 
 3 3 3 3 (c) (3, – 2) (d) (–3, 2)
(c) (–1, 5) (d) (1, 5) 26. A circle passes through the vertices of a triangle ABC.
18. The centroid of the triangle whose vertices are (3, –7), If the vertices are A(–2, 5), B(–2, –3), C(2, –3), then the
centre of the circle is
(–8, 6) and (5, 10) is
(a) (0, 0) (b) (0, 1)
(a) (0, 9) (b) (0, 3)
(c) (–2, 1) (d) (0, –3)
(c) (1, 3) (d) (3, 5)
27. Which of the following points is 10 units from the origin?
19. The points A (– 4, – 1), B (–2, – 4), C (4, 0) and D (2, 3)
(a) (– 6, 8) (b) (– 4, 2)
are the vertices of a
(c) (– 6, 5) (d) (6, 4)
(a) Parallelogram (b) Rectangle
28. The distance between which of the following two points
(c) Rhombus (d) Square is 2 units?
20. If the point P (p, q) is equidistant from the points (a) (–2, –3) and (–2, –4) (b) (0, 4) and (0, 6)
A (a + b, b – a) and B (a – b, a + b), then (c) (7, 2) and (6, 2) (d) (4, –3) and (2, 3)
ap = by
(a) (b) bp = ay 29. Which of the following is / are not correct ?
ap + bq = 0
(c) (d) bp + aq = 0 Three points will form :
21. The distances of a point from the x-axis and the y-axis (a) an equilateral triangle, if all the three sides are equal.
are 5 and 4 respectively. The coordinates of the point (b) an isosceles triangle, if any two sides are equal.
can be (c) a collinear or a line, if sum of two sides is equal to
third side.
(a) (5, 4) (b) (5, 0)
(d) a rhombus, if all the four sides are equal.
(c) (0, 4) (d) (4, 5)
30. Which of the following is / are correct?
22. If the points (a, 0), (0, b) and (1, 1) are collinear then which
of the following is true : Four points will form :

1 1 1 1 (a) a rectangle, if opposite sides and diagonals are not equal.


(a) + = 2 (b) − = 1
a b a b (b) a parallelogram, if opposite sides are not equal.
1 1 1 1 (c) a square, if all the four sides and diagonals are equal.
(c) − = 2 (d) + = 1
a b a b (d) a right angle triangle, if sum of squares of any two
sides is equal to square of third largest side.
Coordinate Geometry M-31

Assertion & Reason

DIRECTIONS : Study the given Case/Passage and answer the DIRECTIONS : Each of these questions contains an Assertion
following questions. followed by Reason. Read them carefully and answer the
question on the basis of following options. You have to select
Case/Passage-I the one that best describes the two statements.
In order to conduct Sports Day activities in your School, (a) If both Assertion and Reason are correct and Reason is
lines have been drawn with chalk powder at a distance the correct explanation of Assertion.
of 1 m each, in a rectangular shaped ground ABCD, 100
(b) If both Assertion and Reason are correct, but Reason is
flowerpots have been placed at a distance of 1 m from
not the correct explanation of Assertion.
each other along AD, as shown in given figure below.
Niharika runs 1/4 th the distance AD on the 2nd line and (c) If Assertion is correct but Reason is incorrect.
posts a green flag. Preet runs 1/5 th distance AD on the (d) If Assertion is incorrect but Reason is correct.
eighth line and posts a red flag.
[From CBSE Question Bank 2021] 36. Assertion : If A(2a, 4a) and B(2a, 6a) are two vertices of
D C an equilateral triangle ABC then, the vertex C is given by
(2a + a 3,5a) .

Reason : In equilateral triangle, all the coordinates of three


G
R vertices can be rational.

37. Assertion : The points (k, 2 – 2k), (– k+ 1, 2k) and

1
2
(– 4 – k, 6 – 2k) are collinear if k = .
1 2
A
1 2 3 4 5 6 7 8 9 10
Reason : Three points A, B and C are collinear in same
straight line, if AB + BC = AC.
31. Find the position of green flag
38. Assertion : Mid-point of a line segment divides line in the
(a) (2, 25) (b) (2, 0.25)
ratio 1 : 1.
(c) (25, 2) (d) (0, –25)
Reason : If area of triangle is zero that means points are
32. Find the position of red flag
collinear.
(a) (8, 0) (b) (20, 8)
(c) (8, 20) (d) (8, 0.2) Match the Following
33. What is the distance between both the flags?
(a) √41 (b) √11 DIRECTIONS : Each question contains statements given in
(c) √61 (d) √51 two columns which have to be matched. Statements (A, B, C, D)
in column-I have to be matched with statements (p, q, r, s) in column-II.
34. If Rashmi has to post a blue flag exactly halfway between
the line segment joining the two flags, where should she
39. Column-II gives distance between pair of points given in
post her flag?
Column-I
(a) (5, 22.5) (b) (10, 22)
Column-I Column-II
(c) (2, 8.5) (d) (2.5, 20)
(A) (–5, 7), (–1, 3) (p) 17
35. If Joy has to post a flag at one-fourth distance from green
flag ,in the line segment joining the green and red flags, (B) (5, 6), (1, 3) (q) 8
then where should he post his flag?
( 3 + 1,1), (0, 3) (r) 6
(C)
(a) (3.5, 24) (b) (0.5, 12.5)
(c) (2.25, 8.5) (d) (25, 20) (0,0) (− 3, 3) (s)
(D) 4 2
M-32 Mathematics
40. Column-II gives the coordinates of the point P that divides 48. (1, 2), (4, y), (x, 6) and (3, 5) are the vertices of a
the line segment joining the points given in Column-I. parallelogram taken in order, then the value of x and y are
Column-I Column-II ...........

A (–1, 3) and
(A) (p) (7, 3) 49. Relation between x and y if the points (x, y), (1, 2) and
(7, 0) are collinear is ...........
B (5, –6) internally
in the ratio 1 : 2 50. The distance of the point (x1, y1) from the origin is ............

A (–2, 1) and
(B) (q) (0, 3)
True / False
B (1, 4) internally
in the ratio 2 : 1 DIRECTIONS : Read the following statements and write your
A (–1, 7) and
(C) (r) (1, 3) answer as true or false.
B (4, –3) internally
51. The distance between P (x1, y1) and Q (x2, y2) is
in the ratio 2 : 3
A (4, –3) and
(D) (s) (1, 0) ( x2 + x1 )2 + ( y2 + y1 )2

B (8, 5) internally 52. The coordinates of the point P(x, y) which divides the
in the ratio 3 : 1 line segment joining the points A(x1, y1) and B(x2, y2)
internally in the ratio m1 : m2 are
Fill in the Blanks  m1 x2 − m2 x1 m1 y2 − m2 y1 
 m + m , m + m 
1 2 1 2
DIRECTIONS : Complete the following statements with an
appropriate word / term to be filled in the blank space(s). 53. The mid-point of the line segment joining the points

41. Points (3, 2), (–2, –3) and (2, 3) form a ............. triangle. x +x y + y 
P (x1, y1) and Q (x2, y2) is  1 2 , 1 2  .
 2 2 
42. If x – y = 2, then point (x, y) is equidistant from (7, 1) and 54. Points (1, 7), (4, 2), (–1, –1) and (– 4, 4) are the vertices
(.........) of a square.
43. Distance between (2, 3) and (4, 1) is .............. 55. Coordinates of the point which divides the join of (–1, 7)
and (4, –3) in the ratio 2 : 3 is (1, 3).
44. Points (1, 5), (2, 3) and (– 2, – 11) are ...........
56. Ratio in which the line segment joining the points (– 3, 10)
45. (5, – 2) (6, 4) and (7, – 2) are the vertices of an ..............
and (6, – 8) is divided by (– 1, 6) is 3 : 7.
triangle.
57. The ratio in which the point (3, 5) divides the join of
46. Point on the X-axis which is equidistant from (2, –5) and (1, 3) and (4, 6) is 2 : 1.
(–2, 9) is .............
58. The distance of the point (5, 3) from the X-axis is 5 units
47. Point (– 4, 6) divide the line segment joining the points
A(– 6, 10) and B(3, – 8) in the ratio .............. 59. The distance of a point (2, 3) from Y-axis is y-units.
Coordinate Geometry M-33

ANSWER KEY & SOLUTIONS


1. (c) PQ = 13 ⇒ PQ2 = 169 ∴ Its x-cordinates = 0

⇒ (x – 2)2 + (–7 – 5)2 = 169 −4 K + 3


∴ = 0 ⇒ −4 K + 3 = 0
K +1
⇒ x2 – 4x + 4 + 144 = 169
3
⇒ x2 – 4x – 21 = 0 ⇒K =
4
⇒ x2 – 7x + 3x – 21 = 0 3
⇒ Required ratio = :1
⇒ (x – 7) (x + 3) = 0 4

⇒ x = 7, –3 ∴ ratio = 3 : 4

2. (b) We have, 6. (b) Suppose the required ratio is m1 : m2


a(b1 – b + b1) + a1(b – b1 – b) + (a – a1) (b – a1) (b – b1) = 0 Then, using the section formula, we get
⇒ 2ab1 – ab – a1b1 + ab – ab1 – a1b + a1b1 = 0 m1 (4) + m 2 ( –3)
–2 =
⇒ ab1 – a1b = 0 m1 + m 2

⇒ ab1 = a1b. ⇒ – 2m1 – 2m 2 = 4m1 – 3m 2

3. (b) Coordinates of mid-point are given by ⇒ m2 = 6 m1 ⇒ m1 : m2 =1:6


 x1 + x 2 y1 + y 2 
 ,  7. (c) As the point P (x, y) lies on the line joining the points
 2 2 
A (a, 0) and B (0, b), the points A, B and P are collinear
a  ⇒ a (b – y) + 0 (y – 0) + x (0 – b) = 0
Here, coordinates of mid-point are  , 4 
3 
a −6 − 2 ⇒ ab – ay – bx = 0 ⇒ bx + ay = ab
So, =
3 2 x y
⇒ + =1
∴ a = – 12 a b
4. (b) Since (x, y) is midpoint of (3, 4) and (k, 7) 8. (b) A(0, 4), B(0, 0), C(3, 0)

3+ k 4+7
∴ x= and y = AB = (0 − 0)2 + (0 − 4)2 = 4
2 2
Also 2x + 2y + 1 = 0 putting values we get BC = (3 − 0)2 + (0 − 0)2 = 3
3+k+4+7+1=0
CA = (0 − 3)2 + (4 − 0)2 = 5
⇒ k + 15 = 0 ⇒ k = – 15
AB + BC + CA = 12
5. (b) Let the required ratio be K : 1
 5 −1 
∴ The coordinates of the required point on the y-axis is 9. (a) P(x, 0) =  , 0  = (2, 0)
 2 
K(−4) + 3(1) K(2) + 5(1) [Q A and B both lies on x-axis]
x= ; y=
K +1 K +1
Three or more points lies in same line are called collinear.
Since, it lies on y-axis
M-34 Mathematics
10. (c) For reflection of a point with respect to x-axis change 19. (b)
sign of y-coordinate and with respect to y-axis change sign
20. (b)
of x-coordinate.
21. (d) 4
P (4, 5)
 4 × 3 + 1× 6 3 × y + 1× 5 
11. (d) P(6, 2) =  , 
 3 +1 3 +1  5
18
Q 6 ≠  (Question is wrong)
4
22. (d) As (a, 0), (0, b) and (1, 1) are collinear
3y + 5
2=   ⇒ 3y + 5 = 8 \ a(b – 1) + 0(1 – 0) + 1(0 – b) = 0
4
3y = 3  ⇒  y = 1 ab – a – b = 0

12. (a) ab = a + b

13. (a) Since, C (y, – 1) is the mid-point of P (4, x) and Q (–2, 4). 1 1
1= +
a b
23. (b) Let coordinate of point p be (h, 5h + 3)
P(h, 5h + 3)
y = 5x + 3
4−2 4+ x
We have, = y and = −1
2 2
∴ y = 1 and x = – 6 M

14. (c) Let the required ratio be k : 1


Q (3, –2)
6k − 4(1) 3
Then, 2 = or k =
k +1 2 Since, M is the mid-point of PQ, therefore by mid-point
3  h + 3 5h + 3 − 2 
∴ The required ratio is :1 or 3 : 2 formula, we have M =  ,
2  2  .
2
3(3) + 2(3)
Also, y = =3
3+ 2 Clearly by observing the options, we can say that M must
3(1) + 4(2) − 7 4 4 lie on the line
15. (c) − = − =
3(−2) + 4(1) − 7 −9 9 y = 5x – 7
16. (b) Let P (x, 0) be a point on X-axis such that AP = BP
A
⇒ AP2 = BP2
⇒ (x + 2)2 + (0 – 3)2 = (x – 5)2 + (0 + 4)2
24. (a)
⇒ x2 + 4x + 4 + 9 = x2 – 10x + 25 + 16 90 – θ θ
B
⇒ 14x = 28 ⇒ x = 2 O 90 – θ
y
Hence, required point is (2, 0).

17. (b) x
C
x + x + x y + y + y3 
18. (b) Centroid is  1 2 3 , 1 2 
 3 3  ∠AOB = q
3 + (–8) + 5 –7 + 6 + 10   0 9 
i.e.  ,  =  3 , 3  = (0, 3) Q CO ^ OA
 3 3   
Coordinate Geometry M-35
\ ∠BOC = (90° – q)
⇒ ( x + 2) 2 + ( y + 3) 2 = ( x - 2) 2 + ( y + 3) 2
3 4 
sin θ = cos θ = Q cos θ = 1 −sin 2 θ  ⇒ x2 + 4 + 4x + (y + 3)2 = x2 + 4 – 4x + (y + 3)2
5; 5 
⇒ 8x = 0 ⇒ x = 0
−y
Now, sin(90° − θ) =
1 \ centre of the circle is (0, 1).

−4 27. (a)
⇒ y = – cos q  ⇒  y =
5
28. (b)
x 3
cos(90° – q) =   ⇒  x = sin q, x = 29. (d) 
All the statements given in option ‘a’, ‘b’ and ‘c’ are
1 5
correct.
25. (c) (x – 6)2 + (y + 6)2 = (x – 3)2 + (y + 7)2...(i) 30. (c)

Also, (x – 3)2 + (y – 3)2 = (x – 3)2 + (y + 7)2  1 


31. (a) (2, 25) ∵ x = 2, y = 4 × 100 = 25
 
y2 – 6y + 9 = y2 + 14y + 49
 1 
– 20y = 40 ⇒ y = – 2 32. (c) (8, 20) ∵ x = 8, y = 5 × 100 = 20 
 

Putting y = – 2 in equation (i), we have 33. (c) (8 − 2)2 + (25 − 20)2 = 36 + 25 = 61


(x – 6)2 + (4)2 = (x – 3)2 + (5)2
 8 + 2 25 + 20 
34. (a)  ,  = (5, 22.5)
x2 – 12x + 36 + 16 = x2 – 6x + 9 + 25 2 2 

 2 + 5 25 + 22.5 
–6x = – 18 ⇒ x = 3 35. (a)  ,  = (3.5, 24)
 2 2 
26. (b) A (–2, 5)
36. (c) Let A(x1, y1), B(x2, y2) & C(x3, y3) are all rational
coordinates of a triangle ABC.

x1 y1 1
O 1
(x, y) ar (∆ ABC) = x2 y2 1
2
x3 y3 1
B C 3
(–2, 3) (2, –3) = [(x1 – x2)2 + (y1 – y2)2 ]
4
LHS = rational, RHS = irrational
Let O(x, y) is the centre of the given circle.
Hence, (x1, y1) (x2, y2) & (x3, y3) cannot be all rational.
Join OA, OB & OC.
37. (a) Both assertion and reason are correct. Reason is correct
Q OA = OB = OC
explanation of assertion.
\ OA2 = OB2
38. (b) Both statements are individually correct.

⇒ ( x + 2) 2 + ( y - 5) 2 = ( x + 2) 2 + ( y + 3) 2 39. (A) → (s); (B) → (p); (C) → (q); (D) → (r)

⇒ x2 + 4 + 4x + y2 + 25 – 10y = x2 + 4 + 4x + y2 + 9 + 6x 40. (A) → (s); (B) → (q); (C) → (r); (D) → (p)
41. right angle
⇒ 16y = 16 ⇒ y = 1
42. (3, 5)
Again: OB2 = OC2
M-36 Mathematics
51. False
43. 2 2
52. False
44. Non-collinear
53. True
45. isosceles
54. True
46. (–7, 0)
55. True
47. 2 : 7
56. False
48. (6, 3)
57. True
49. x + 3y = 7
58. False
50. x12 + y12
59. False
5 Acids, Bases and
Triangles
Salts
A
Multiple Choice Questions (MCQs)

DIRECTIONS : This section contains multiple choice


questions. Each question has 4 choices (a), (b), (c) and (d) out
of which only one is correct

1. In the given figure, AD is the bisector of ∠A. If BD = 4 cm, 70° 50°


DC = 3 cm and AB = 6 cm, determine AC B D C
A (a) 30° (b) 40°
(c) 50° (d) 45°

6 cm 5. If ∆ABC ~ ∆DEF such that BC = 2.1cm and EF = 2.8 cm. If


the area of triangle DEF is 16 cm2, then the area of triangle
ABC (in sq. cm) is
(a) 9 (b) 12
B C (c) 8 (d) 13
4 cm D 3 cm
6. From the given figure, then length of the sides AB and BD.
(a) 4.5 cm (b) 3.5 cm A
(c) 4.8 cm (d) 3.2 cm
24 cm
2. ∆ABC is an isosceles triangle right angled at B. 20 cm
Similar triangles ACD and ABE are constructed on D
sides AC and AB. Ratio between the areas of ∆ABE
B
and ∆ACD is C 15 cm

(a) 1 : 4 (b) 2 : 1 (a) 25 cm and 7 cm (b) 25 cm and 17 cm


(c) 1 : 2 (d) 4 : 3 (c) 7 cm and 15 cm (d) 18 cm and 7 cm
3. Two isosceles triangles have their corresponding angles 7. The diagonal BD of a parallelogram ABCD intersects the
equal and their areas are in the ratio 25 : 36. The ratio of segment AE at the point F, where E is any point on the side
their corresponding height is BC. Then
D C
(a) 25 : 35 (b) 36 : 25
4
(c) 5 : 6 (d) 6 : 5
AB BD E
4. In ∆ ABC, = , ∠B = 70° and ∠C = 50°. Then, 1 F 2
AC DC
3
∠BAD = _______.
A B
M-38 Mathematics
EF FB 15. The perimeters of two similar triangles ABC and PQR are
(a)  (b) DF × EF = FB × FA respectively 36 cm and 24 cm. If PQ = 10 cm, then AB =
FA AB
(a) 10 cm (b) 20 cm
(c) DF × EF = (FB)2 (d) None of these
(c) 25 cm (d) 15 cm
8. ABC is an isosceles triangle in which AB = AC = 10 cm,
16. In the given figure, DE || BC. The value of EC is
BC = 12 cm. PQRS is a rectangle inside the isosceles
A
triangle. Given PQ = SR = y cm and PS = QR = 2x cm, cm 1 cm
.5
then x = D1 E
3y cm
(a) 6 − (b) 6 + 6y 3
4
4y 7x + 8y B C
(c) 6 + (d)
3 4 (a) 1.5 cm (b) 3 cm
9. If ABC and EBC are two equilateral triangles such that D (c) 2 cm (d) 1 cm
is mid-point of BC, then the ratio of the areas of triangles 17. In the given figure, express x in terms of a, b and c.
ABC and BDE is L
(a) 2 : 1 (b) 1 : 2 (c) 1 : 4 (d) 4 : 1
10. Given ∆ABC ~ ∆DEF, if AB = 2DE and area of ∆ABC is a P
56 cm2, find the area of ∆DEF. x
(a) 14 sq.cm (b) 5 sq.cm 46° 46°
M N K
(c) 18 sq.cm (d) 56 sq.cm b c
11. The area of a right angled triangle is 40 sq. cm. and its ab ac
(a) x = (b) x =
perimeter is 40 cm. The length of its hypotenuse is a+b b+c
(a) 16 cm (b) 18 cm bc ac
(c) x = (d) x =
(c) 17 cm (d) Data insufficient b+c a+c
12. In the given figure, P and Q are points on the sides AB and 18. Which of the following statement is false?
AC respectively of a triangle ABC. PQ is parallel to BC (a) All isosceles triangles are similar.
and divides the triangle ABC into 2 parts, equal in area. (b) All quadrilateral triangles are similar.
The ratio of PA : AB = (c) All circles are similar.
A (d) None of the above
19. If ∆ABC ~ ∆APQ and ar (∆APQ) = 4 ar (∆ABC), then the
P Q ratio of BC to PQ is
(a) 2 : 1 (b) 1 : 2
B C
(c) 1 : 4 (d) 4 : 1
(a) 1 : 1 (b) ( 2 − 1) : 2
20. The areas of two similar triangles ABC and PQR are in the
(c) 1: 2 (d) ( 2 − 1) :1 ratio 9 : 16. If BC = 4.5 cm, then the length of QR is
(a) 4 cm (b) 4.5 cm
BC 1
13. It is given that ∆ABC ~ ∆PQR with = . Then (c) 3 cm (d) 6 cm
QR 3
ar(∆PQR) 21. The length of the side of a square whose diagonal is 16 cm, is
is equal to
ar(∆ABC )
(a) 8 2 cm (b) 2 8 cm
1 1
(a) 9 (b) 3 (c) (d) (c) 4 2 cm (d) 2 2 cm
3 9
22. ∆ABC is an equilateral triangle with each side of length
14. The area of a right angled isosceles triangle whose
2p. If AD ⊥ BC, then the value of AD is
hypotenuse is equal to 270 m is-
(a) 3 (b) 3p
(a) 19000 m2 (b) 18225 m2
(c) 2p (d) 4p
(c) 17256 m2 (d) 18325 m2
Triangles M-39
23. The areas of two similar triangles are 81 cm2 and 49 cm2 29. Let D be a point on the side BC of a triangle ABC such
respectively, then the ratio of their corresponding medians that ∠ADC = ∠BAC. If AC = 21 cm, then the side of an
is equilateral triangle whose area is equal to the area of the
(a) 7 : 9 (b) 9 : 81 rectangle with sides BC and DC is
1/2 –1/2
(a) 14 × 3 (b) 42 × 3
(c) 9 : 7 (d) 81 : 7 3/4 1/2
(c) 14 × 3 (d) 42 × 3
24. In the figure, ABC is a triangle in which AD bisects
30. In a triangle ABC, ∠BAC = 90°; AD is the altitude from
∠A, AC = BC, ∠B = 72° and CD = 1cm. Length of BD
A on to BC. Draw DE perpendicular to AC and DF
(in cm) is
perpendicular to AB. Suppose AB = 15 and BC = 25.
C
Then the length of EF is
(a) 12 (b) 10
(c) 5 3 (d) 5 5
D 31. If ∆ABC is an equilateral triangle such that AD ­⊥ BC, then
AD2 =
3a 2 3a 2
A. B.
A B 4 2
1 3 3
(a) 1 (b) C. BC2 D. a
2 4 2
(a) A and C (b) A
5 –1 3 +1
(c) (d)
2 2 (c) D (d) B and C
25. Let P be an interior point of a DABC. Let Q and R be the 32. Which among the following is/are correct?
reflections of P in AB and AC, respectively. If Q, A, R are (a) The ratios of the areas of two similar triangles is equal
collinear, then ∠A equals to the ratio of their corresponding sides.
(a) 30° (b) 60° (b) The areas of two similar triangles are in the ratio of
(c) 90° (d) 120° the corresponding altitudes.
26. Let ABC be a triangle and M be a point on side AC closer (c) The ratio of area of two similar triangles are in the
to vertex C than A. Let N be a point on side AB such that ratio of the corresponding medians.
MN is parallel to BC and let P be a point on side BC such (d) If the areas of two similar triangles are equal, then the
that MP is parallel to AB. If the area of the quadrilateral triangles are congruent.
5
BNMP is equal to of the area of DABC, then the ratio 33. Which among the following is/are correct?
AM/MC equals 18 (I) If the altitudes of two similar triangles are in the ratio
(a) 5 (b) 6 2 : 1, then the ratio of their areas is 4 : 1.
18 15 (II) PQ || BC and AP : PB = 1 : 2.
(c) (d)
5 2 area ( ∆APQ )1
Then, =
27. In DABC, AB = AC, P and Q are points on AC and area ( ∆ABC )4
AB respectively such that BC = BP = PQ = AQ. Then, (III) The areas of two similar triangles are respectively
∠AQP is equal to (use p =180º) 9cm2 and 16cm2. The ratio of their corresponding
2π 3π sides is 3 : 16.
(a) (b)
7 7 (a) I (b) II
4π 5π (c) III (d) None of these
(c) (d)
7 7 34. In a right angled triangle ∆ABC, length of two sides are
28. Consider a DPQR in which the relation QR2 + PR2 = 5 8cm and 6cm, then which among the given statements is/
PQ2 holds. Let G be the points of intersection of medians are correct?
PM and QN. Then ∠QGM is always A
(a) less than 45°
(b) obtuse
(c) a right angle
B C
(d) acute and larger than 45°
M-40 Mathematics
(a) Length of greatest side is 10cm 39. What is the height of Ajay’s house?
(b) ∠ACB = 45° (a) 30m (b) 40m
(c) ∠BAC = 45° (c) 50m (d) 20m
(d) Pythagoras theorem is not applicable here. 40. When the tower casts a shadow of 40m, same time what
35. Two triangles are similar if will be the length of the shadow of Ajay’s house?
(a) their corresponding angles are equal. (a) 16m (b) 32m
(b) their corresponding sides are equal. (c) 20m (d) 8m
(c) both are right triangle. 41. When the tower casts a shadow of 40m, same time what
(d) None of the above will be the length of the shadow of Vijay’s house?
36. Which of the following is/are not correct? (a) 15m (b) 32m
(a) If the diagonals of a quadrilateral divide each other (c) 16m (d) 8m
proportionally, then it is a trapezium.
(b) The line segments joining the mid-points of the Case/Passage-II
adjacent sides of a quadrilateral form a parallelogram.
Rohan wants to measure the distance of a pond during the visit
(c) If corresponding sides of two similar triangles are to his native. He marks points A and B on the opposite edges of a
in the ratio 4 : 5, then corresponding medians of the
pond as shown in the figure below. To find the distance between the
triangles must be in the ratio 4 : 5.
points, he makes a right-angled triangle using rope connecting B
(d) None of the above with another point C are a distance of 12m, connecting C to point D
at a distance of 40m from point C and the connecting D to the point
A which is are a distance of 30m from D such the ∠ADC=90° .
[From CBSE Question Bank-2021]
DIRECTIONS : Study the given Case/Passage and answer the B 12 m
following questions. A C

Case/Passage-I
Vijay is trying to find the average height of a tower near his
30

m
m

40
house. He is using the properties of similar triangles.The height
of Vijay’s house if 20m when Vijay’s house casts a shadow 10m
long on the ground. At the same time, the tower casts a shadow D
50m long on the ground and the house of Ajay casts 20m shadow
on the ground. [From CBSE Question Bank-2021] 42. Which property of geometry will be used to find the
distance AC?
(a) Similarity of triangles (b) Thales Theorem
(c) Pythagoras Theorem (d) Area of similar triangles
43. What is the distance AC?
(a) 50m (b) 12m
(c) 100m (d) 70m
Vijay's House Tower 44. Which is the following does not form a Pythagoras triplet?
Ajay's House
(a) (7, 24, 25) (b) (15, 8, 17)
37. What is the height of the tower? (c) (5, 12, 13) (d) (21, 20, 28)
(a) 20m (b) 50m 45. Find the length AB?
(c) 100m (d) 200m (a) 12m (b) 38m
38. What will be the length of the shadow of the tower when (c) 50m (d) 100m
Vijay’s house casts a shadow of 12m? 46. Find the length of the rope used.
(a) 75m (b) 50m (a) 120m (b) 70m
(c) 45m (d) 60m (c) 82m (d) 22m
Triangles M-41
51. In figure, the line segment XY is parallel to the side AC of
Assertion & Reason ∆ABC and it divides the triangle into two parts of equal
areas, then,
DIRECTIONS : Each of these questions contains an Assertion A
followed by reason. Read them carefully and answer the question
on the basis of following options. You have to select the one that
X
best describes the two statements.
(a) If both Assertion and Reason are correct and Reason is
the correct explanation of Assertion.
(b) If both Assertion and Reason are correct, but Reason is
B Y C
not the correct explanation of Assertion.
(c) If Assertion is correct but Reason is incorrect. Column-I   Column-II
(d) If Assertion is incorrect but Reason is correct. (A) AB : XB (p)  2 :1
47. Assertion : If in a ∆ABC, a line DE || BC, intersects AB (B) ar (∆ ABC) : ar (∆ XBY) (q)  2 : 1
AB AC (C) AX : AB (r)  ( 2 − 1) 2 : 2
in D and AC in E, then = .
AD AE (D) ∠ X : ∠ A (s)  1 : 1
Reason : If a line is drawn parallel to one side of a triangle 52. Column-I Column-II
intersecting the other two sides, then the other two sides A
are divided in the same ratio.
(A) (p) 36 : 49
48. Assertion : ABC is an isosceles, right triangle, right angled
at C. Then AB2 = 3AC2.
C B
Reason : In an isosceles triangle ABC if AC = BC and ABC is an isosceles
AB2 = 2AC2, then ∠C = 90°. right angled triangle.
49. Assertion : ABC and DEF are two similar triangles such AB2 =?
that BC = 4 cm, EF = 5 cm and area of ∆ABC = 64 cm2, (B) ∆ABC ~ ∆DEF, (q) AB2 = 2AC2
then area of ∆DEF = 100 cm2.
such that
Reason : The areas of two similar triangles are in the ratio
AB = 1.2 cm and
of the squares of the corresponding altitudes.
DE = 1.4 cm
Match the Following area ( ∆ABC )
= ?
area ( ∆DEF )
DIRECTIONS : Each question contains statements given in (C) ∆ABC ~ ∆APQ and (r) 36 : 49
two columns which have to be matched. Statements (A, B, C, D) area ( ∆APQ ) 36
=
in column-I have to be matched with statements (p, q, r, s) in area ( ∆ABC ) 49
column-II.
BC
= ?
50. If in a ∆ ABC, DE || BC and intersects AB in D and AC in PQ
E, then. A
Column-I Column-II
AD AC (D) D E (s) 6 : 7
(A) (p)
DB AE A
AB
(B) (q) AE B C
AD EC If DE || BC and
AE AD 6
(C) DB
D E
(r) =
AB AC DB 7

AD EC B C AE 2
(D) (s) then, =?
AB AC EC 2
M-42 Mathematics

Fill in the Blanks True / False

DIRECTIONS : Complete the following statements with an DIRECTIONS : Read the following statements and write your
appropriate word / term to be filled in the blank space(s). answer as true or false.
53. All circles are .................. 66. Two figures having the same shape but not necessarily the
54. All squares are .................. same size are called similar figures.
55. All ........... triangles are similar. 67. All the congruent figures are similar but the converse is
56. Two polygons of the same number of sides are similar, not true.
if their corresponding angles are ......... and their 68. If in two triangles, corresponding angles are equal, then
corresponding sides are in the same ........... their corresponding sides are in the same ratio and hence
57. If a line is drawn parallel to one side of a triangle to intersect the two triangles are similar.
the other two sides in distinct points, then the other two
69. If in two triangles, two angles of one triangle are
sides are divided in the ............. ratio.
respectively equal to the two angles of the other triangle,
58. If a line divides any two sides of a triangle in the same then the two triangles are similar.
ratio, then the line is parallel to the .............. side.
70. If in two triangles, corresponding sides are in the same
59. All congruent figures are similar but the similar figures
ratio, then their corresponding angles are equal and hence
need ............. be congruent.
the triangles are similar.
60. Two polygons of the same number of sides are similar, if
71. If one angle of a triangle is equal to one angle of another
all the corresponding angles are .............
triangle and the sides including these angles are in the
61. The diagonals of a quadrilateral ABCD intersect each other same ratio (proportional), then the triangles are similar.
AO CO 72. The ratio of the areas of two similar triangles is equal to
at the point O such that = . ABCD is a .............
BO DO the square of the ratio of their corresponding sides.
62. A line drawn through the mid-point of one side of a triangle
73. In a right triangle, the square of the hypotenuse is equal to
parallel to another side bisects the ............. side.
the sum of the squares of the other two sides.
63. Line joining the mid-points of any two sides of a triangle
74. If, in a triangle, square of one side is equal to the sum of
is ............. to the third side.
the squares of the other two sides, then the angle opposite
64. In fig., MN || BC and AM : MB = 1 : 2, then A the first side is a right angle.
ar(∆AMN) 75. Diagonals AC and BD of a trapezium ABCD with
= ..................
ar(∆ABC) M N OA OB
AB || DC intersect each other at the point O, = .
OC OD
B C
65. In DABC, AB = 6 3 m, AC = 12 cm and BC = 6 cm,
then ∠B = .................. .
OR
Two triangles are similar if their corresponding sides are
.................. .
Triangles M-43

ANSWER KEY & SOLUTIONS


1. (a)
It is given that AD is the bisector of ∠A.
ar (∆ABC) BC2
5. (a) =
AB BD 6×3 ar (∆DEF) EF2
= ⇒ AC = = 4.5 cm
AC DC 4
2
2.1 
D ⇒ ar (∆ABC) =   × ar (∆DEF) = 9cm
2
 2.8 

6. (a)
From the right angled ∆ACB,
2. (c)
E A
AB2 = AC2 + CB2
= (20)2 + (15)2 = 400 + 225 = 625
x
∴ AB = 625  25cm
2x
Again, from right angled ∆ABD
x
AB2 = AD2 + BD2
B C

Let AB = BC = x. ⇒ 625 = (24)2 + (BD)2
Since, ∆ABC is right-angled with ∠B = 90°
⇒ (BD)2 = 625 – 576 = 49
∴ AC2 = AB2 + BC2 = x2 + x2 = 2x2
⇒ BD = 7 cm
⇒ AC = 2x
7. (a) In DAFD & DFEB,
Since, ∆ABE ~ ∆ACD ∠1 = ∠2 (V.O.A)
Area ( ∆ABE ) AB x 1 2 2
∠3 = ∠4 (Alternate angle)
∴ = = 2 = .
Area ( ∆ACD ) AC 2
2x 2
\ ∆ FBE ~ ∆ FDA
Area (∆ABE) 1 EF FB
Thus, = So, 
Area (∆ACD) 2 FA DF

Thus, required ratio is 1 : 2. 8. (a) In DABC, AB = AC


3. (c) Here, the two triangles are similar. Draw AL ⊥ BC,
Ratio of areas of two similar triangles is equal to the then L is the mid-point of BC
ratio of squares of their corresponding altitudes. Using Pythagoras theorem in ∆ABL, we get
h12 25 AL = 8cm
So, =
h22 36 Also, ∆BPS ≅ ∆CQR,
h1 5 \ BS = RC
∴ =
h2 6 SL = LR = x cm
\ BS = CR = 6 – x
4. (a) In DABC, ∠A = 180° – (70° + 50°) = 60°.
In DABL, PS || AL
BD AB
= . It means AD is the bisector of ∠A.
DC AC PS BS y 6− x
\ = ⇒ =
AL BL 8 6
1
∴ ∠BAD = × 60° = 30° 3
2 or x = 6 − y
4
M-44 Mathematics
A Perimeter of ∆ABC AB BC AC
15. (d) ∵ = = =
Perimeter of ∆PQR PQ QR PR

10
m
P

cm
16. (c) Since, DE || BC ∴ ∆ADE ~ ∆ABC
10
P 2x Q
AD AE 1.5 1
y ∴ = ⇒ = ⇒ EC = 2 cm
y
2x DB EC 3 EC
B S L R C
17. (b) In ∆KPN and ∆KLM, we have
9. (d) All equilateral triangles are similar ∠KNP = ∠KML = 46°
∴ ∆ ABC~ ∆EBD ∠K = ∠K (Common)
2 A ∴ ∆KNP ~ ∆KML (By A A criterion of similarity)
Area of ABC BC
⇒ 
Area of BDE BD 2 E ⇒ KN = NP ⇒ c = x
D is mid-point of BC KM ML b+c a
B C
2 D 18. (a) Statement given in option (a) is false.
( 2BD) 4
\ BC = 2BD = = 19. (b) Since, ∆ABC ~ ∆APQ
BD 2 1
⇒ Area (∆ABC) : Area (∆BDE) = 4 : 1 ar( ∆ABC ) BC 2
∴ =
ar( ∆APQ) PQ 2
10. (a) Given, AB = 2DE and ∆ABC ~ ∆DEF
2
ar( ∆ABC ) BC 2  BC  1
area(∆ABC ) AB 2 ⇒ = ⇒  =
Hence, = 4 ⋅ ar( ∆ABC ) PQ 2  PQ  4
area(∆DEF ) DE 2
BC 1
56 4 DE 2 ⇒ =
or = = 4 [∵ AB = 2DE] PQ 2
area(∆DEF ) DE 2
20. (d) Since, ∆ABC ~ ∆PQR
56
area (∆DEF) = = 14sq.cm. ar(∆ABC ) BC 2 9 (4.5) 2
4 ∴ = ⇒ =
ar(∆PQR) QR 2 16 QR 2
11. (b)
12. (c) As PQ is parallel to BC ⇒ ∆ABC ~ ∆APQ 16 × (4.5) 2
⇒ QR 2 = ⇒ QR = 6 cm
9
Area of ∆ABC 2
⇒ = 21. (a) Let side of a square = x cm
Area of ∆APQ 1
∴ By Pythagoras theorem, x2 + x2 = (16)2 = 256
Ratio of sides = AB = 2 \ AP : AB = 1 : 2 ⇒ 2x2 = 256 ⇒ x2 = 128 ⇒ x = 8 2 cm.
AP 1
22. (b) Given an equilateral triangle ABC in which
13. (a) Since, ∆ABC ~ ∆PQR
AB = BC = CA = 2p A
ar(∆PQR) PR 2 QR 2 9  QR 3 
\ = = = ∵ = =9 and AD ⊥ BC.
ar(∆ABC ) AC 2 BC 2 1  BC 1  2p 2p
∴ In ∆ADB,
14. (b) Hypotenuse = 270m AB2 = AD2 + BD2
⇒ Hypotenuse2 = Side2 + Side2 =2 Side2 B D C
(By Pythagoras theorem)
⇒ Side2 = (270)2/2 = 72900/2 = 36450 ⇒ (2p)2 = AD2 + p2 ⇒ AD2 =
3 p.
or Side = 190.91m
23. (c) Given, area of two similar triangles,
⇒ Required area = 1/2 × 190.91 × 190.91
A1 = 81cm2 , A2 = 49 cm2
= 36446.6/2 = 18225 m2 (approx). A1 81 9
Ratio of corresponding medians = = =
A2 49 7
Triangles M-45
24. (c) Let BD = x cm \ ∠QAB = ∠BAP
R is reflection of P on AC
Since AC = BC, therefore DABC is an isoscele triangle.
\ ∠RAC = ∠CAP
⇒ ∠B = ∠CAB = 72°
∠QAR = 180°
Since AD bisects ∠A
\ 2∠BAP + 2∠CAP = 180°
\ ∠DAB = 36° so, In DADB, ∠ADB = 72°
∠BAP + ∠CAP = 90°  ⇒  ∠BAC = 90°
⇒ DADB is an isoscele triangle
26. (a) DABC ~ DANM
\ AB = AD = 1cm
⇒ AB = 1 cm Area of ∆ABC AC 2
\ =  ...(i)
Similarly, DADC is also an isoscele triangle. Area of ∆ANM AM 2

\ AD = CD ⇒ AD = 1 cm DABC ~ DMPC
2
\ Area of ∆ABC = AC 
C
...(ii)
Area of ∆MPC MC 2
36° 1
From Eqs. (i) and (ii,) we get

Area of ∆ANM AM 2
D =
x
1+

Area of ∆MPC MC 2
72°
36° x Area of ∆ANM + Area of ∆MPC AM 2 + MC 2
=
36° Area of ∆MPC MC 2
72°
A B Now, Area of DANM + Area of DMPC
AC CD = Area of DABC – Area of BNMP
Now   =
AB BD 5
Using Area of BNMP = of area of DABC
1+ x 1 18
⇒ =   ⇒  x + x2 – 1 = 0
1 x 2 2
2 \ 13 (Area of ∆ABC ) = AM + MC  ...(iii)
– 1 ± (1) – 4(1)(–1) –1 ± 5 18 (Area of ∆MPC ) MC 2
⇒ x = =
2 2
13  AC 2  AM 2 + MC 2
From Eq. (iii), =
BD =
5 –1 18  MC 2  MC 2
2
⇒ 13 (AM + MC)2 = 18 (AM2 + MC2)
25. (c) Here, ABC is a triangle & P be interior point of a
AM 1
DABC, Q and R be the reflections of P in AB and AC, ⇒ = 5, . Hence, option (a) is correct.
MC 5
respectively.
27. (d) A
R

A
b
Q θ

φ b
Q P
P 2b a
2b
a
a C
B C B
In DABC
As QAR are collinear
AB = AC
\ ∠QAR = 180°
⇒  ∠C = ∠B ⇒  ∠B = ∠C = a
Q is reflection of P on AB
M-46 Mathematics
By angle sum properly in DABC,
1  2QR 2 + 7QR 2  1 2 2
b + a + a = 180 =
9   = QR = QM
 4  4
⇒  b + 2a = 180°  ...(i) QG 2 + GM 2 = QM 2 ∴ ∠QGM = 90
In  DQPB
29. (c) ∵ ∠BAC = ∠ADC (given)
⇒  ∠QPB = 180 – 4b
∠C = ∠C  (common)
Since ‘APC’ is a straight line A
⇒  180 – 4b + a + b = 180
⇒  a = 3b  ...(ii)
From equations (i) & (ii)
180 B D C
b + 2(3b) = 180 ⇒ b =
7 \ DABC ~ DDAC  (by AA similarity criterion)
 180  5
∠AQP = 180° – 2  = p BC AC
 7  7
2
⇒ = ⇒ BC × DC = AC
AC DC
28. (c) Let DPQR 2
⇒ BC × DC = (21) = area of rectangle with sides BC
Given, QR2 + PR2 = 5PQ2 & DC
Median PM and QN intersect at G.
Now, Area of equilateral triangle = area of rectangle
1
⇒ PN = NR = PR &
⇒ 3 (side) = (21) ⇒ Side = 14 × 3
2 2 3/4
2
1 4
QM = MR = QR
2 30. (a) Here, BAC is a right angle triangle
P B
AB = 15 & BC = 25
N D
∴ AC = BC 2 − AB 2 = 20 F
G
1 A
Area of ∆ABC =
BC. AD
Q R 2 E C
M
1
= AB. AC
2 1 2
QG = QN , GM = PM
3 3 ⇒ BC. AD = AB. AC
2 2
2 2 2  1  ⇒ 25(AD) = 15(20) ⇒  AD = 12
⇒ QG + GM =  QN  +  PM 
3 3
∵ AEDF is rectangle then, AD = EF = 12
4 1
= QN 2 + PM 2
9 9 31. (a) We know that height of an
4  2 PQ 2 + 2QR 2 − PR 2  3 A
= a,
equilateral triangle
9  4 
2
1  2 PQ 2 + 2 PR 2 − QR 2  where a is the side of 30° 30°
+  
9 4 equilateral triangle
 8 PQ + 8QR − 4 PR 
2 2 2
3 3
∴ AD 2 = a 2 = BC 2
60° 60°
  B C
1  +2 PQ 2 + 2 PR 2 − QR 2  4 4  D
=
9 4 
32. (d) 33. (a) 34. (a)
 

  35. (a) (By definition of similar triangles).
1 10 PQ + 7QR − 2 PR 
2 2 2
36. (d) All the statements given in option (a, b, c) are correct.
= 9  4

 
Triangles M-47
Sol. (37-41) DB EC
P
⇒ 1 + =1+ A
AD AE
X
AD + DB AE + EC
A
⇒ =
AD AE D E

20 m AB AC
⇒ =
B C Q R Y Z AD AE B C
10 m 50 m 20 m
[Vijay’s house] [Tower] [Ajay’s house] ∴ Assertion is true.
37. (c) Q DABC ~ DPQR Since reason gives Assertion.
AB BC 20 10 48. (d) In right angled ∆ABC,
∴ = ⇒ =
PQ QR PQ 50
AB2 = AC2 + BC2 (By Pythagorus Theorem)
⇒ PQ = 100 = AC2 + AC2 [∵ BC = AC]
\ Height of the tower = 100 m = 2AC2 A
38. (d) Let BC = 12 m and PQ = 100 m \ AB2 = 2AC2
AB BC 20 12 ∴ Assertion is false.
= ⇒ =
PQ QR 100 QR
Again since C B
⇒ QR = 60 AB2 = 2AC2 = AC2 + AC2
39. (b) Q DABC ~ DXYZ = AC2 + BC2 (∵ AC = BC given)
AB BC 20 10 \ ∠C = 90° (By converse of Pythagoras Theorem)
∴ = ⇒ =
XY YZ XY 20 \ Reason is true.
⇒ XY = 40 49. (b) Reason is true [standard result]
40. (a) Let QR = 40 m, PQ = 100 m and XY = 40 m For Assertion, since ∆ABC ~ ∆DEF
PQ QR 100 40 area ( ∆ABC )
∴ = ⇒ = BC 2 (4)2 16
XY YZ 40 YZ \ = = =
area ( ∆DEF ) EF 2 (5)2 25
⇒ YZ = 16 m.
(∵ ratio of areas of two similar ∆s is equal to the ratio
41. (d) Let QR = 40m, PQ =100m and AB = 20 m of the squares of corresponding sides)
AB BC 20 BC 64
∵ = ⇒ = 16 64 × 25
PQ QR 100 40 \ = ⇒ area (∆DEF) =
area (∆DEF ) 25 16
⇒ BC = 8 m. = 4 × 25 = 100 cm2
42. (c) Pythagoras theorem \ Assertion is true. But reason is not the correct
43. (a) AC2 = 302 + 402 = 2500 ⇒ AC = 50m explanation for assertion.
44. (d) (21, 20, 28) Q 282 ≠ (21)2 + (20)2 50. (A) → q; (B) → p; (C) → s; (D) → r
45. (b) AB = 50 – 12 = 38m 51. (A) → p; (B) → q; (C) → r; (D) → s
46. (c) 82m
52. (A) → q; (B) → p; (C) → s; (D) → r
47. (a) Reason is true. [This is Thale’s Theorem]
(A) AB2 = AC2 + BC2
For Assertion
Since, ∆ABC is an isosceles right angled triangle.
Since DE || BC ∴ by Thale’s Theorem
\ AC = BC
AD AE DB EC
= ⇒ = Now, AB2 = AC2 + AC2 = 2AC2
DB EC AD AE
M-48 Mathematics
2 2 1
area ( ∆ABC ) ( AB ) (1.2) 1.44 64.  
(B) = = = 9
area ( ∆DEF ) ( DE ) 2
(1.4) 2 1.96
AM AM 1 1
36 (36 × 2) 72 = = =
= = = AB AM + BM 1 + 2 3
49 (49 × 2) 98 2 2
ar ( ∆AMN)  AM   1 1
2 \ =  =   =
area ( ∆APQ ) ( BC ) 36 BC 6 ar ( ∆ABC)  AB 3 9
(C) = = = =
area ( ∆ABC ) ( PQ ) 2 49 PQ 7 65. [90°]
AD AE 6 Q AB2 + BC2 = 108 + 36 = 144 = AC2
∵ DE || BC \
(D) = =
DB EC 7 So, AC is hypotenuse and ∠B = 90°.

53. similar 54. similar 55. equilateral OR


[Proportional]
56. equal , ratio 57. same 58. third 59. not
66. True 67. True 68. True 69. True
60. equal 61. trapezium 62. third
70. True 71. True 72. True 73. True
63. parallel
74. True 75. True
6 Acids, Bases and
Introduction
Salts
Trigonometry
to

7. If x = a cos2θ + b sin2θ, then (x – a) (b – x) is equal to


Multiple Choice Questions (MCQs) (a) (a – b) sinθ cosθ (b) (a – b)2 sin2θ cos2θ
(c) (a – b)2 sinθ cosθ (d) (a – b) sin2θ cos2θ
DIRECTIONS : This section contains multiple choice
questions. Each question has 4 choices (a), (b), (c) and (d) out 3
8. , find the value of 9 cot2A – 1.
If cos A =
of which ONLY ONE is correct. 5
16
(a) 1 (b)
65
1. If (sec2θ) (1 + sinθ) (1 – sinθ) = k, then find the value of k.
65
(a) sinθ (b) secθ (c) (d) 0
(c) 1 (d) cotθ 16
9. cos 1° . cos 2°. cos 3° ......... cos 179° is equal to
 15  (2 + 2sin θ) (1 − sin θ) (a) –1 (b) 0
2. If cot θ =   , then evaluate (c) 1 (d) 1/ 2
8 (1 + cos θ) (2 − 2 cos θ)
225 10. sin2q + cosec2q is always
(a) 1 (b) (a) greater than 1
64
(b) less than 1
156
(c) (d) –1 (c) greater than or equal to 2
7
(d) equal to 2
b(1 − cos θ)
3. If x = a (cosec θ + cot θ) and y = , then xy = 11. If x = p sec q and y = q tan θ , then
2 2 sin θ
a +b (a) x2 – y2 = p2q2
(a) a2 – b2
(b)
2 2
a −b (b) x2q2 – y2p2 = pq
a
ab
(c) (d) 1
b (c) x2q2 – y2p2 = 2 2
4. If p sin θ + q cos θ = a and p cos θ – q sin θ = b, then p q
p+a q−b (d) x2q2 – y2p2 = p2q2
+ =
q+b p−a
a sin θ − b cos θ
(a) 1 (b) a2 + b2 12. If b tan θ = a, the value of is
a sin θ + b cos θ
(c) 0 (d) 2
a−b a+b
5. If x = r sinA cos C, y = r sin A sin C, z = r cos A, then (a) (b) 2
a 2 + b2 a + b2
(a) r2 = x2 + y2 + z2 (b) r2 = 2xy

a 2 + b2 a 2 − b2
(c) r2 = x + y + z (d) r2 = y2 + z2 + 2xy (c) (d)
a 2 − b2 a 2 + b2
6. If tan2θ = 1– a2, then the value of 13. (cos4A – sin4A) is equal to
sec θ + tan3θ cosec θ is
(a) 1 – 2 cos2A (b) 2 sin2 A – 1
(a) (2 – a2) (b) (2 – a2)1/2
(c) sin2A – cos2A (d) 2 cos2A – 1
(c) (2 – a2)2/3 (d) (2 – a2)3/2
M-50 Mathematics
a sin φ b sin θ a cos θ cos θ
14. If tan θ = and tan φ = , then = 22. If + = 4, then
1 − a cos φ 1 − b cos θ b 1 − sin θ 1 + sin θ

sin θ sin θ 3 1
(a) (b) (a) cos θ = (b) sin θ =
1 − cos θ 1 − cos φ 2 2
sin θ 1
sin φ
(c) (d) (c)
θ = 60° (d) tan θ =
sin φ 3
sin θ
1 tan θ − cot θ
23. is equal to
15. If cosec x – cot x = , where x ≠ 0, then the value of sin θ cos θ
cos2x – sin2x is 3
(a) sec2 θ + cosec2 θ (b) cot2 θ – tan2 θ
16 9 (c) cos2 θ – sin2 θ (d) tan2θ – cot2θ
(a) (b)
25 25
2 tan 30°
8 7 24. =
(c) (d) 1 + tan 2 30°
25 25 (a) sin 60° (b) cos 60°
16. If cosec x + sin x = a and sec x + cos x = b, then
(c) tan 60° (d) sin 30°
2 2
2 3 2 3
(a) ( a b) + (ab ) =1 1 − tan 2 45°
25. =
2 2 1 + tan 2 45°
(ab 2 ) 3
(b) + ( a 2b 2 ) 3 =1 (a) tan 90° (b) 1
(c) a2 + b2 = 1 (c) sin 45° (d) 0
(d) b2 – a2 = 1 26. sin 2A = 2 sin A is true when A =
17. If tan2q = 1 – e2, then the value of (a) 0° (b) 30°
secq + tan3q cosec q is equal to (c) 45° (d) 60°
(a) (1 – e2)1/2 (b) (2 – e2)1/2 2 tan 30°
27. =
(c) (2 – e2)3/2 (d) (1 – e2)3/2 1 − tan 2 30°
18. If sinq + sin3q = cos2q, then the value of (a) cos 60° (b) sin 60°
cos6q – 4cos4q + 8cos2q is (c) tan 60° (d) sin 30°
(a) 1 (b) 4
28. 9 sec2 A – 9 tan2 A =
(c) 2 (d) 0 (a) 1 (b) 9
11 (c) 8 (d) 0
19. If cosec A + cot A = , then tan A
2 29. (1 + tan θ + sec θ) (1 + cot θ– cosec θ) =
21 15 (a) 0 (b) 1
(a) (b)
22 16 (c) 2 (d) –1
44 11 30. (sec A + tan A) (1 – sin A) =
(c) (d)
117 117 (a) sec A (b) sin A
(c) cosec A (d) cos A
2 tan 30°
20. is equal to
1 + tan 2 30° 1 + tan 2 A
31. =L
(a) sin 30° (b) cos 60° 1 + cot 2 A
1 3 (a) sec2 A (b) –1
(c) (d)
2 2 (c) cot2 A (d) tan2 A
32. The value of (sin 30° + cos 30°) – (sin 60° + cos 60°) is
sin θ − 2sin 3 θ
21. is equal to (a) –1 (b) 0
2 cos3 θ − cos θ (c) 1 (d) 2
(a) sec θ (b) tan θ
(c) sec θ − 1 (d) cot θ
Introduction to Trigonometry M-51
tan 30° 39. The value of sec C is
33. The value of is 4 5
cot 60° (a) (b)
1 1 3 3
(a) (b) 1
2 3 (c) (d) None of these
3
(c) 3 (d) 1
40. sin2C + cos2C =
34. The value of (sin 45° + cos 45°) is (a) 0 (b) 1
1 (c) –1 (d) None of these
(a) (b) 2
2
3 Assertion & Reason
(c) (d) 1
2
a DIRECTIONS : Each of these questions contains an Assertion
35. Given that sin θ = , then cos θ is equal to
b followed by reason. Read them carefully and answer the
b question on the basis of following options. You have to select
b
(a) 2 2
(b) the one that best describes the two statements.
b −a a
(a) If both Assertion and Reason are correct and Reason is
b −a 2 2 a the correct explanation of Assertion.
(c) (d)
b b2 − a 2 (b) If both Assertion and Reason are correct, but Reason is
not the correct explanation of Assertion.
36. If sin A + sin2A = 1, then the value of the expression (c) If Assertion is correct but Reason is incorrect.
(cos2A + cos4A) is (d) If Assertion is incorrect but Reason is correct.
1 3
(a) 1 (b) 41. Assertion: In a right angled triangle, if tan θ = , the
2 4
greatest side of the triangle is 5 units.
(c) 2 (d) 3
Reason: (greatest side)2 = (hypotenuse)2
3 1 = (perpendicular)2 + (base)2.
37. If sin (A + B) =
and sin 2B = , then
2 2 1
(a) tan B = 1 (b) B = 30° 42. Assertion : In a right angled triangle, if cos θ = and
2
1 3
(c) B = 45° (d) cos A = sin θ = , then tan θ = 3
2 2
sin θ
Reason: tan θ = cos θ

DIRECTIONS : Study the given Case/Passage and answer the Match the Following
following questions.
Case/Passage DIRECTIONS : Each question contains statements given in
In ∆ABC, right angled at B two columns which have to be matched. Statements (A, B, C, D)
C in column-I have to be matched with statements (p, q, r, s) in
column-II.

3cm 43. In ∆ ABC, ∠ B = 90°, AB = 3 cm and BC = 4 cm, then


match the column.
A B Column-I Column-II
AB + AC = 9 cm and BC = 3cm. (A) sin C (p) 3/5
38. The value of cot C is (B) cos C (q) 4/5
[From CBSE Question Bank-2021] (C) tan A (r) 5/3
3 1 (D) sec A (s) 4/3
(a) (b)
4 4
5
(c) (d) None of these
4
M-52 Mathematics
44. Column-I Column-II 50. In ∆ ABC, right-angled at B, AB = 24 cm, BC = 7 cm.
cos A 1 + sin A sin A = ...........
(A) + (p) cosec A + cot A
1 + sin A cos A 51. If 15 cot A = 8, sec A = ..............
cos A − sin A + 1 52. In ∆ PQR, right-angled at Q, PR + QR = 25 cm and
(B) (q) 2 sec A
cos A + sin A − 1 PQ = 5 cm. The value of tan P is ...........
1 + sin A 53. sin 60° cos 30° + sin 30° cos60° = ................
(C) (r) sec A + tan A
1 − sin A 54. 2 tan2 45° + 3 cos2 30° – sin2 60° = .............
sin 2 A 1 + sec A cos 45°
(D) (s) 55. = ..............
1 − cos A sec A sec30° + cosec 30°
7
45. If sin A = , then
25 True / False
Column-I Column-II
(A) cos A (p) 24/25 DIRECTIONS : Read the following statements and write your
(B) tan A (q) 7/24 answer as true or false.
(C) cosec A (r) 25/7 56. The value of tan A is always less than 1.
(D) sec A (s) 25/24 57. sec A = 12/5, for some value of angle A.
58. cos A is the abbreviation used for the cosecant of angle A.
Fill in the Blanks 59. cot A is the product of cot and A.
4
DIRECTIONS : Complete the following statements with an 60. sin θ = , for some angle θ.
3
appropriate word / term to be filled in the blank space(s).
61. sin (A + B) = sin A + sin B.
46. The value of sin A or cos A never exceeds ...... 62. cot A is not defined for A = 0°.
47. sin2 A+ cos2 A = .............. 63. If ∠B and ∠Q are acute angles such that sin B = sin Q,
48. If tan A = 4/3, then sin A ............... then ∠B ≠ ∠Q.
49. In a right trianlge ABC, right angled at B, if tan A = 1,
sin A cos A = ..........
Introduction to Trigonometry M-53

ANSWER KEY & SOLUTIONS


sec2θ (1 + sinθ) (1– sinθ) = k
1. (c) 7. (b) x – a = b sin2θ – a sin2θ = (b – a) sin2θ

 1  b – x = b cos2θ – a cos2θ = (b – a) cos2θ


 (1 – sin2θ) = k
 cos 2 θ  ∴ (x – a) (b – x) = (b – a)2 sin2θ cos2θ
= (a – b)2 sin2θ cos2θ
 1 
⇒  (cos2θ) = k ⇒ 1 = k.
 cos 2 θ  8. (c) cos A =
3
⇒ sin A = 1 −
9
=
4
(2 + 2sin θ) (1 − sin θ) 2(1 + sin θ) (1 − sin θ) 5 25 5
2. (b) = Consider,
(1 + cos θ) (2 − 2 cos θ) (1 + cos θ) (2) (1 − cos θ)

2(1 − sin 2 θ) 2cos 2 θ


2 9 cos 2 A 9 cos 2 A − sin 2 A
 15  225 2 9 cot 2 A − 1 = − 1 =
= 2
= 2
= cot θ =   =
8 64 sin 2 A sin 2 A
2(1 − cos θ) 2sin θ
3. (c)
We have, x = a (cosec θ + cot θ)  9   16 
9  −  
 25   25  (81 − 16) 25 65
x = = × =
⇒ = (cosec θ + cot θ) ...(1) 16 25 16 16
a
25
 1 − cos θ  y 1 cos θ
and y = b  ⇒ = −
 sin θ  b sin θ sin θ 9. (b) 10.  (c)
y x
⇒ = cosec θ – cot θ ...(2) 11. (d) We know that sec2θ – tan2θ = 1 and sec θ = ,
b p
x y y
⇒ × = (cosec θ + cot θ) (cosec θ – cot θ) tan θ =
a b q
xy ∴ x2q2 – p2y2 = p2q2
⇒ = (cosec2θ – cot2 θ) ∴ xy = ab
ab
4. (c) By squaring and adding both the given equations, we a
12. (d) Given, tan θ =
get b
p2 (sin2θ + cos2θ) + q2 (cos2θ + sin2θ) a sin θ − b cos θ a tan θ − b a 2 − b 2
∴ = =
= a2 + b2 a sin θ + b cos θ a tanθ + b a 2 + b 2

⇒ p2 + q2 – a2 – b2 = 0 13. (d) (cos 4 A − sin 4 A) = (cos 2 A) 2 − (sin 2 A) 2


⇒ (p – a) (p + a) + (q – b) (q + b) = 0
= (cos 2 A − sin 2 A)(cos 2 A + sin 2 A)
p+a q−b
⇒ + =0
q+b p−a = (cos 2 A − sin 2 A)(1) = cos 2 A − (1 − cos 2 A)
5. (a) x = r sin A cos C, y = r sin A sin C, z = r cos A = 2 cos2A – 1

x2+ y2+ z2 = r2sin2A cos2C + r2 sin2A sin2C + r2 cos2A a sin φ


14. (d) We have, tan θ =
1 − a cos φ
( )( )
= r 2 sin 2 A cos 2 C + sin 2 C + r 2 cos 2 A
1 1
⇒ cot θ = − cot φ ⇒ cot θ + cot φ = ...(i)
2 2 2 2 2
= r sin A(1) + r cos A = r (sin A + cos A) =
2 2
r2 a sin φ a sin φ

6. (d) sec θ + tan3θ cosec θ b sin θ


and tan φ =
1 − b cos θ
sin θ
sec θ +
= tan2 θ cosec θ = sec θ (1 + tan2θ) 1
cos θ ⇒ cot φ = − cot θ
b sin θ
(1 + tan2θ)3/2 = [1+ (1 – a2)]3/2
=
M-54 Mathematics
1 1 sin q 1
⇒ cot φ + cot θ = ...(ii) \ secq + tan3q cosecq = + tan 2 q. .  
b sin θ cos q cos q sin q
From (i) and (ii), we have 1 sec 2 q
= (1 + tan 2 q) = = sec3 q = (2 – e2)3/2  [from (i)]
1 1 a sin θ cos q cos q
⇒ =
a sin φ b sin θ b sin φ 18. (b) sinq + sin3q = cos2q
1 sinq(1 + 1 – cos2q) = cos2q
15. (d) Let cosec x – cot x =
3 ⇒ sin2q(2 – cos2q)2 = cos4q
1 cos x 1 ⇒ (1 – cos2q)(4 + cos4q – 4cos2q) = cos4q
⇒ – =
sin x sin x 3 ⇒ 4 + cos4q – 4cos2q – 4cos2q – cos6q + 4 cos4q = cos4q
x
2sin 2 ⇒ cos6q – 4 cos4q + 8 cos2q = 4
1 – cos x 1 1
2
⇒ = ⇒ =
sin x 3 x x 3 19. (c)
2 sin cos
2 2
2 tan 30°
x 1 20. (d)
We have,
⇒ tan = 1 + tan 2 30°
2 3
Consider 2 1

x 2 3 2×3
3 3
2 tan = = = =
2 = 3 =3  1 
2
1+
1 3×4 2
tan x = 1+  
x 1 4 3
1 – tan 2 1–  3
2 9
Alternate method:
3 4
Thus sin x = , cos x =
5 5  2 tan A 
 Using identity,sin 2 A = 
∴ cos2 x – sin2x =
16 9
– =
7  1 + tan 2 A 
25 25 25
2 tan 30° 3
16. (a) cosec x – sin x = a & sec x – cos x = b sin 60° = 2
=
1 + tan 30° 2
1 1
cosec x − = a & sec x − =b 21. (b) We have,
cosec x sec x
2 2 sin θ − 2sin 3 θ sin θ(1 − 2sin 2 θ)
cosec x − 1 sec x − 1 =
⇒ =a& =b 2 cos3 θ − cos θ cos θ(2 cos 2 θ − 1)
cosec x sec x
1 − 2(1 − cos 2 θ)   (2 cos 2 θ − 1) 
cot 2 x tan 2 x = tan θ   = tan θ  
⇒ = a& =b 2 2
cosec x sec x  2 cos θ − 1   2 cos θ − 1 
2 2 = tan q
cos x = a & sin x = b
sin x cos x cos θ cos θ
22. (c) We have, + =4
1 − sin θ 1 + sin θ
cos 4 x sin 2 x
Now, a 2b = 2
. = cos3 x  1 + sin θ + 1 − sin θ 
sin x cos x ⇒ cos θ   = 4
 1 − sin 2 θ
⇒ cos x = (a2b)1/3 ⇒ cos2 x = (a2b)2/3
2 cos θ 1
Similarly, sin2 x = (ab2)2/3 ⇒ 2
= 4 ⇒ cos θ = ⇒ θ = 60°
cos θ 2
We know that, sin2x + cos2x = 1 tan θ − cot θ
23. (d) We have,
sin θ cos θ
⇒ (ab2)2/3 + (a2b)2/3 =1
tan θ cot θ
= −
17. (c) ∵ tan2 q = 1 – e2 sin θ cos θ sin θ cos θ

⇒ secq = 1 + tan 2 q = 1 + 1 - e2 sin θ cos θ


= −
cos θ sin θ cos θ sin θ cos θ cos θ
⇒ secq = 2 - e 2 ...(i)
Introduction to Trigonometry M-55

=
1

1
= sec2 θ − cosec2θ = (1 + sin A)(1 – sin A)
2 2 cos A
cos θ sin θ
= 1 + tan θ − 1 − cot 2 θ = tan 2 θ − cot 2 θ
2 1 – sin 2 A cos 2 A
= =  (∵ cos 2 A = 1 – sin 2 A)
cos A cos A
 1  = cosA.
2 
2 tan 30°  3
24. (a) = 1 + tan 2 A (sec2 A – tan 2 A) + tan 2 A
1 + tan 2 30°  1 
2 31. (d) =
1+   1 + cot 2 A (cosec2 A – cot 2 A) + cot 2 A
 3
2
sec2 A sin 2 A sin A  2
2 = = =
  = tan A.
cosec2 A 2
cos A cos A
2 3 3
= 3 = × = = sin 60°. 32. (b) (sin 30° + cos 30°) – (sin 60° + cos 60°)
1 3 4 2
1+
3 1 3  1 3
2 2 =  2 + 2  −  2 + 2  = 0
1 – tan 45° 1 – (1)    
25. (d) = = 0.
1 + tan 45°2
1 + (1)2 1
tan 30° 3
26. (a) Here, when A = 0° 33. (d) = =1
cot 60° 1
LHS = sin 2 A = sin 0° = 0 3
and RHS = 2 sin A= 2 sin 0° = 2 × 0 = 0 1 1 2
34. (b) sin 45° + cos 45° = + = = 2
In the other options, we will find that 2 2 2
(on rationalizing)
LHS ≠ RHS
a2 b2 − a 2
 1  35. (c) cos θ = 1 − sin 2 θ = 1 − =
2  b2 b
2 tan 30°  3
27. (c) = 36. (a) Given, sin A + sin2A =1
1 – tan 2 30°  1 
2
1–   ⇒ sin A = 1 – sin2A = cos2 A
 3
Consider, cos2A + cos4A = sinA + (sin A)2 = 1
2
3
= 3 = 2 × 3 = 3 = tan 60°. 37. (c) We have, sin ( A + B) =
1 3 2 2
1– ⇒ A + B = 60° ...(i)
3
and 2B = 30° ∴ B = 15°
28. (b) 9sec2 A – 9 tan 2 A = 9(sec2 A – tan 2 A)
= 9 × 1 = 9. Putting B in (i), we get

29. (c) (1 + tan θ + sec θ)(1 + cot θ – cosec θ) A + 15° = 60° ⇒ A = 45°

 sin θ 1   cos θ 1  Sol. (38-40):


= 1 + +  × 1 + – 
 cos θ cos θ   sin θ sin θ  In ∆ABC, by Pythagoras theorem,
{(cos θ + sin θ) + 1} × {(cos θ + sin θ) –1} AC2 = AB2 + BC2 ⇒ AB = 4 cm.
=
cos θ × sin θ AC = 5 cm.
2 2 2 2 BC 3
= (cos θ + sin θ) – (1) {∵ (a + b)(a – b) = a – b } 38. (a) cot C = =
cos θ × sin θ AB 4
1 + 2 cos θ sin θ –1 AC 5
= = 2. 39. (b) sec C = =
cos θ × sin θ BC 3
30. (d) (sec A + tan A) (1 – sin A) 4 3
40. (b) sin C = , cos C =
 1 sin A  5 5
=  + × (1 – sin A)
 cos A cos A 
M-56 Mathematics
2 2 44. (A) → q; (B) → p; (C) → r; (D) → s
2 2  4 3
L.H.S = sin C + cos C =   +   45. (A) → p; (B) → q; (C) → r; (D) → s
5 5
16 + 9 46. 1 47. 1 48. 4/5
= = 1 = R.H.S
25 1
41. (a) Both Assertion and Reason are correct and Reason is 49. 50. 7/25 51. 17/8
2
the correct explanation of the assertion.
52. 12/5 53. 1
greatest side = (3)2 + (4)2 = 5 units.
7 3( 3 − 1)
54. 55.
42. (a) Both assertion and reason are correct and reason is 2 4
the correct explanation of the assertion.
56. False 57. True 58. False
3
tan θ = ×2= 3 . 59. False 60. False 61. False
2
43. (A) → p; (B) → q; (C) → s; (D) → r 62. True 63. False
7 Acids,
Areas Bases
Salts
Circles
and
Related to

5. Mrs. Vidya bought a piece of cloth as shown in the figure.


Multiple Choice Questions (MCQs) The portion of the cloth that is not coloured consists of 6
identical semi-circles.
DIRECTIONS : This section contains multiple choice 42 cm
questions. Each question has 4 choices (a), (b), (c) and (d) out
of which only one is correct.

1. Find area of minor segment made by a chord which


subtends right-angle at the centre of a circle of radius
10 cm.
(a) 24.5 cm2 (b) 25.5 cm2
Find the area of the coloured portion.
(c) 24.5 cm2 (d) 28.5 cm2
(a) 144 cm2 (b) 126 cm2
2. The figure shows two concentric circles B
(c) 195 cm2 (d) 243 cm2
with centre O and radii 3.5 m and 7 D A
6. The figure given shows a rectangle with a semi-circle and
m. If ∠BOA = 40°, find the area of the C
2 identical quadrants inside it.
shaded region. O
28 cm
77 2 76
(a) cm (b)
6 5
73
(c) (d) None of these 16 cm
6 23 cm
3. A drain cover is made from a square metal plate of side
40 cm having 441 holes of diameter 1 cm each drilled in
it. Find the area of the remaining square plate.
22
(a) 1250.5 cm2 (b) 1253.5 cm2 What is the shaded area of the figure? (Use π = )
7
(c) 1240.2 cm2 (d) 1260.2 cm2 (a) 363 cm2 (b) 259 cm2
4. The figure given shows two 15 cm 4 cm (c) 305 cm2 (d) 216 cm2
identical semi-circles cut out 7. The perimeter of a sector of a circle with central angle 90° is
from a piece of coloured paper. 25 cm. Then the area of the minor segment of the circle is.
Find the area of the remaining (a) 14 cm2 (b) 16 cm2
22 (c) 18 cm2 (d) 24 cm2
piece of paper (Use π = ) 20 cm
7
(a) 296.1 cm 2 8. The sum of the areas of two circles, which touch each other
externally, is 153 π. If the sum of their radii is 15, then the
(b) 265.4 cm2
ratio of the larger to the smaller radius is
(c) 221.5 cm2
(a) 4 : 1 (b) 2 : 1
(d) 201.7 cm2 7 cm
(c) 3 : 1 (d) None of these
M-58 Mathematics
9. A race track is in the form of a ring whose inner and outer
circumference are 437m and 503m respectively. The area
of the track is
(a) 66 sq. cm (b) 4935 sq. cm
(c) 9870 sq. cm (d) None of these
10. If the sum of the circumferences of two circles with
diameters d1 and d2 is equal to the circumference of a
circle of diameter d, then
2 2 2 π 3π
(a) d1 + d 2 = d (b) d1+ d2 = d (a) (b)
4 2
(c) d1+ d2 > d (d) d1+ d2 < d
π
11. In the adjoining figure, OABC is a O (c) (d) p
C 2
square of side 7 cm. OAC is a
quadrant of a circle with O as centre. 17. If a circular grass lawn of 35m in radius has a path 7m
The area of the shaded region is wide running around it on the outside, then the area of the
(a) 10.5 cm2 path is
(b) 38.5 cm2 (a) 1450 m2 (b) 1576 m2
(c) 49 cm2 (c) 1694 m2 (d) 3368 m2
A B
(d) 11.5 cm2 18. In the adjoining figure, OACB is a quadrant of a circle of
12. The area of a circular ring formed by two concentric circles radius 7 cm. The perimeter of the quadrant is
whose radii are 5.7 cm and 4.3 cm respectively is B
(Take π = 3.1416) C
(a) 43.98 sq.cm (b) 53.67 sq. cm
(c) 47.24 sq.cm (d) 38.54 sq.cm
13. A sector is cut from a circular sheet of radius 100 cm, the
angle of the sector being 240º. If another circle of the area O A
same as the sector is formed, then radius of the new circle (a) 11 cm (b) 18 cm
is (c) 25 cm (d) 36 cm
(a) 79.5 cm (b) 81.6 cm
19. If the circumference of a circle increases from 4π to 8π,
(c) 83.4 cm (d) 88.5 cm then its area is
14. The area of a sector of angle p (in degrees) of a circle with (a) halved (b) doubled
radius R is (c) tripled (d) quadrupled
p p
(a) × 2πR (b) × πR 2 20. If the radius of a circle is diminished by 10%, then its area
360° 180° is diminished by
p p (a) 10% (b) 19%
(c) × 2πR (d) × 2πR 2
720° 720° (c) 36% (d) 20%
15. If the sector of a circle of diameter 10 cm subtends an 21. If the perimeter of a semi-circular protractor is 36 cm, then
angle of 144° at the centre, then the length of the arc of its diameter is
the sector is
(a) 10 cm (b) 14 cm
(a) 2π cm (b) 4π cm (c) 12 cm (d) 16 cm
(c) 5π cm (d) 6π cm
22. The area of a circular path of uniform width ‘d’ surrounding
16. The figure below shows two concentric circles with centre a circular region of radius ‘r’ is
O. PQRS is a square inscribed in the outer circle. It also
(a) πd(2r + d) (b) π(2r + d) r
circumscribes the inner circle, touching it at point B, C,
(c) π(d + r)d (d) π(d + r)r
D and A. The ratio of the perimeter of the outer circle to
that of polygon ABCD is
Areas Related to Circles M-59
23. If Anish is moving along the boundary of a triangular field (c) The ratio between the circumference and area of a
of sides 35 m, 53 m and 66 m and you are moving along circle of radius 5 cm is 2 : 5.
the boundary of a circular field whose area is double the (d) Area of a circle whose radius is 6 cm, when the length
area of the triangular field, then the radius of the circular of the arc is 22 cm, is 66 cm2.
22
field is (Take π = ) 32. Which of the following statement is/are not correct?
7
(a) A chord divides the interior of a circle into two parts.
(a)
14 3 m (b) 3 14 m (c) 28 3 m (d) 7 3 m (b) An arc of a circle whose length is less than that of a
24. If the area of a square inscribed in a semicircle is 2cm2, semicircle of the same circle is a called a minor arc.
then the area of the square inscribed in a full circle of the (c) Circles having the same centre but different radii are
same radius is ______ called concentric circles.
(a) 5 cm2 (b) 10 cm2 (c) 5 2 cm2 (d) 25 cm2 (d) A line segment joining any two points of a circle is
called an arc.
25. Suppose we have two circles of radius 2 each in the plane
such that the distance between their centers is 2 3 . The 33. Tick the correct answer in the following and justify
area of the region common to both circles lies between your choice. If the perimeter and the area of a circle are
numerically equal, then the radius of the circle is
(a) 0.5 and 0.6 (b) 0.65 and 0.7
(a) 2 units (b) π units
(c) 0.7 and 0.75 (d) 0.8 and 0.9
26. A circle is inscribed in a right angled triangle of perimeter (c) 4 units (d) 7 units
7p . Then the ratio of numerical values of circumference 34. If θ is the angle (in degree) of a sector of a circle of
of the circle to the area of the right angled triangle is radius r, then, area of the sector is
(a) 4 : 7 (b) 3 : 7 (c) 2 : 7 (d) 1 : 7
πr 2θ πr 2θ
(a) (b)
27. How much time the minute hand of a clock will take to 360° 180°

describe an angle of radians? 2πrθ 2πrθ
3 (c) (d)
360° 180°
(a) 15 minutes (b) 20 minutes
35. If the sum of the areas of two circles with radii R1 and R2
(c) 10 minutes (d) 25 minutes
is equal to the area of a circle of radius R, then
7 2 2 2
28. If the radius of a circle is cm, then the area of the circle (a) R1 + R2 = R (b) R1 + R 2 = R
π
is equal to
(c) R1 + R2 < R (d) R12 + R 22 < R 2
49
(a) cm2 p cm2
(b) 36. It is proposed to build a single circular park equal in area
π
to the sum of areas of two circular parks of diameters
(c) 154 cm2 (d) 49 cm2
16 m and 12 m in a locality. The radius of the new park
29. If the ratio of the areas of the two circles is 25 : 16, then would be
the ratio of their circumferences is (a) 10 m (b) 15 m (c) 20 m (d) 24 m
25 4 5 500
(a) (b) (c) (d)
16 5 4 625

30. If the sector of a circle of diameter 14cm subtends an angle


of 30° at the centre, then its area is DIRECTIONS : Study the given Case/Passage and answer the
following questions.
(a) 49π (b) 49π (c) 242
(d)
121
12 3π π Case/Passage-I
Pookalam is the flower bed or flower pattern designed during
31. Which of the following is/are not correct?
Onam in Kerala. It is similar as Rangoli in North India and
(a) Area of a circle with radius 6 cm, if angle of sector is Kolam in Tamil Nadu.
132 2
60°, is cm . During the festival of Onam, your school is planning to conduct
14 a Pookalam competition. Your friend who is a partner in
(b) If a chord of circle of radius 14 cm makes an angle competition , suggests two designs given below.
of 60° at the centre of the circle, then area of major Observe these carefully. [From CBSE Question Bank-2021]
sector is 512.87 cm2.
M-60 Mathematics
A Refer to Design A
42. The total length of silver wire required is
A B
(a) 180 mm (b) 200 mm
(c) 250 mm (d) 280 mm
43. The area of each sector of the brooch is
(a) 44 mm2 (b) 52 mm2
B
C (c) 77 mm2 (d) 68 mm2
D C
Refer to Design B
I II 44. The circumference of outer part (golden) is
Design I: This design is made with a circle of radius 32cm (a) 48.49 mm (b) 82.2 mm
leaving equilateral triangle ABC in the middle as shown in the
(c) 72.50 mm (d) 62.86 mm
given figure.
45. The difference of areas of golden and silver parts is
Design II: This Pookalam is made with 9 circular design each
(a) 18 p (b) 44 p
of radius 7cm.
Refer Design I: (c) 51 p (d) 64 p
46. A boy is playing with brooch B. He makes revolution
37. The side of equilateral triangle is with it along its edge. How many complete revolutions
(a) 12√3 cm (b) 32√3 cm must it take to cover 80 p mm ?
(c) 48 cm (d) 64 cm (a) 2 (b) 3
38. The altitude of the equilateral triangle is (c) 4 (d) 5
(a) 8 cm (b) 12 cm
(c) 48 cm (d) 52 cm Assertion & Reason
Refer Design II:
39. The area of square is DIRECTIONS : Each of these questions contains an Assertion
(a) 1264 cm2 (b) 1764 cm2 followed by Reason. Read them carefully and answer the
(c) 1830 cm2 (d) 1944 cm2 question on the basis of following options. You have to select
40. Area of each circular design is the one that best describes the two statements.
(a) 124 cm2 (b) 132 cm2
2
(a) If both Assertion and Reason are correct and Reason is
(c) 144 cm (d) 154 cm2
the correct explanation of Assertion.
41. Area of the remaining portion of the square ABCD is
(b) If both Assertion and Reason are correct, but Reason is
(a) 378 cm2 (b) 260 cm2
not the correct explanation of Assertion.
(c) 340 cm2 (d) 278 cm2
(c) If Assertion is correct but Reason is incorrect.
Case/Passage-II (d) If Assertion is incorrect but Reason is correct.
A brooch is a small piece of jewellery which has a pin at the
47. Assertion : If the circumference of a circle is 176 cm, then
back so it can be fastened on a dress, blouse or coat.
its radius is 28 cm.
Designs of some brooch are shown below. Observe them
carefully. [From CBSE Question Bank-2021] Reason : Circumference = 2π × radius.
48. Assertion : If the outer and inner diameter of a circular
path is 10m and 6m, then area of the path is 16π m2.
Reason : If R and r be the radius of outer and inner circular
path respectively then area of path = π (R2 – r2).
A B 49. Assertion : If a wire of length 22 cm is bent in the
Design A: Brooch A is made with silver wire in the form of shape of a circle, then area of the circle so formed is
a circle with diameter 28mm. The wire used for making 4 40 cm 2 .
diameters which divide the circle into 8 equal parts.
Reason : Circumference of the circle = length of the wire.
Design B: Brooch b is made two colours-Gold and silver. Outer
part is made with Gold. The circumference of silver part is 44mm
and the gold part is 3mm wide everywhere.
Areas Related to Circles M-61
C
Match the Following

DIRECTIONS : Each question contains statements given in (B)


A B (q) 115 m2
two columns which have to be matched. Statements (A, B, C, D) D
in column-I have to be matched with statements (p, q, r, s) in
column-II.
AD = 6.5 cm, CA = 5 cm.
50. For circle shown, match the column. Y
Area of ∆ABC = ?
Column-I Column-II A M B

441 120°
21
cm
D
(A) Area of (p) 3 C
segment AYB 4 O O
(C) (r) 114.7 cm2
21
(B) Area of (q) (88 − 21 3) A B
sector OAYB 4

BC = 6 cm, AB = 8 cm.
(C) Area of ∆ (r) 462
Area of the shaded region = ?
OAB
(D) OM (s) 21/2
51. Two circular flower beds have been shown on two sides O
(D) (s) 30.57 cm2
of a square lawn ABCD of side 56m. If the centre of each
circular flowered bed is the point of intersection O of the 60°
diagonals of the square lawn, then match the column. A B
Column-I Column-II OB = 14.8 m.
Area of the shaded region = ?
(A) Area of ∆OAB (p) 4032
A B
(B) Area of flower bed (q) 784 O
56m Fill in the Blanks
(C) Area of sector OAB (r) 448 D C
(D) Total area (s) 1232 DIRECTIONS : Complete the following statements with an
appropriate word / term to be filled in the blank space(s).
52. Column - I Column - II
θ 54. A sector of a circle is called a ................. sector if the minor
(A) Circumference (p) 2r + × 2πr
360° arc of the circle is a part of its boundary.
θ 55. The boundary of a sector consists of an arc of the circle
(B) Area of a quadrant (q) × πr 2
360° and the two .................. .

πr 2 56. The region enclosed by an arc and a chord is called the


(C) Length of the arc of the (r) ................. of the circle.
4
sector
57. Circumference of a circle is .................. .
θ
(D) Perimeter of the sector (s) × 2πr 58. Area of a circle is .................. .
360°
(E) Area of the sector (t) 2πr 59. Length of an arc of a sector of a circle with radius r and
53. Column-I Column-II angle with degree measure θ is ................. .
60. The area of a circle is the measurement of the region
enclosed by its ................. .
(A) (p) 30 cm2
61. If the area of a circle is 154 cm2, then its circumference is
................. .
62. Area of a sector of a circle with radius 6 cm, if angle of
the sector is 60°, is ................. .
OA = 26m, OC = 23m.
Area of shaded region = ?
M-62 Mathematics
67. Distance moved by a rotating wheel in one revolution is
True / False equal to the circumference of the wheel.
68. In a circle of radius 21 cm, an arc subtends an angle of 60°
DIRECTIONS : Read the following statements and write your
at the centre, then the length of the arc is 22 cm.
answer as true or false.
69. If the circumference of a circle is 88 cm, then its radius is
63. A segment corresponding a major arc of a circle is known 14 cm.
as the major segment.
70. The length of an arc of a sector of a circle of radius r units
64. If the boundary of a segment is a minor arc of a circle, θ
and of centre angle θ is × πr 2 .
then the corresponding segment is called a minor 360°
segment. 71. The length of a rope by which a cow must be tethered in
order that it may be able to graze of an area of 616cm2 is
65. A minor sector has an angle ‘θ’ subtended at the centre
18m.
of the circle, whereas a major sector has no angle.
66. The perimeter of a circle is generally known as its
circumference.
Areas Related to Circles M-63

ANSWER KEY & SOLUTIONS


1. (d) Let AB be the chord of circle such that ∠AOB = 90° πr 2
Let OA = 10 cm Area of 6 semi-circle = 6 × = 3pr2
2
22
∴ AB = 10 2 cm = 3× × 7× 7 = 462 cm2
7
Area of minor segment A X B
Area of cloth piece = 42 × 14 = 588 cm2
= Area of the sector AOB – Area of ∆AOB
Area of the coloured portion = 588 – 462 = 126 cm2
90° 1
= × π(10)2 − × 10 × 10 6. (b) Area of rectangle = 28 × 23 = 644 cm2
360° 2
Radius of semi-circle = 28 ÷ 2 = 14 cm
Radius of quadrant = 23 – 16 = 7 cm
O
Area of unshaded region
m
c

 1 22   1 22 
10

=  × × 14 × 14  + 2
 2 × 4 × 7 × 7 × 7  = 385 cm
A B 2 7   
X
\ Shaded area = (644 – 385) = 259 cm2
= 25 π – 50 = 25 × 3.14 – 50 = 78.5 – 50 = 28.5 cm2.
7. (a) Perimeter of sector = 25 cm
2. (a) Area of the shaded region
θ
40° 22 40° 22 ⇒ 2r + × 2pr = 25
= × × (7)2 − × × (3.5)2 360°
360° 7 360° 7
90° 22
1 22 1 22 49
= × × (7 2 − 3.52 ) = × ×  49 −  ⇒ 2r + 360° × 2 ×
7
× r = 25
9 7 9 7  4 
1 22 49 77 2 11 25
= × × × 3 = cm ⇒ 2r + r = 25 ⇒ r = 25 ⇒ r = 7
9 7 4 6 7 7
3. (b) We have,  πθ sinθ  2
Area of minor segment =  − r
Area of square metal plate = 40 × 40 = 1600 cm2  360° 2 
2
22  1  11  22 90° sin 90°  2
Area of each hole = πr 2 = × = cm 2 =  × −  (7)
7  2  14  7 360° 2 

11  11 1  4
∴ Area of 441 holes = 441× = 346.5cm 2 =  −  × 49 = × 49 = 14 cm2.
14  14 2  14
Hence, area of the remaining square plate 8. (a) Let the radii of the two circles be r1 and r2, then
= (1600 – 346.5) = 1253.5 cm2 r1 + r2 = 15       (given) ..... (i)
4. (c) Area of the remaining piece of paper πr12 + πr22 = 153π   
and (given)
= Area of trapezium – Area of 2 semi-circles
1 1 ⇒ r12 + r22 = 153  ..... (ii)
= (7 + 19)× 20 – 2 × pr2
2 2 On solving, we get
1 22 7 7 r1 = 12, r2 = 3
= × 26 × 20 – × × Required ratio = 12 : 3 = 4 : 1
2 7 2 2
= 260 – 38.5 = 221.5 cm2 9. (b)
2 πr1 = 503 and 2 πr2 = 437
42 503 437
5. (b) Diameter of each semi-circle = = 14 cm ∴ r1 = and r2 =
3 2π 2π
Radius of each semi-circle = 7 cm Area of ring = π (r1 + r2 ) (r1 − r2 )
M-64 Mathematics
 503 + 437   503 − 437  2
= π   r  2  81 
    then, area = π  r −  = πr  
2π 2π  10   100 
940  66  66  81 
=   = 235 × × 7 = 235 × 21 = 4935 sq. cm. Thus, area is diminished by 1 −  % = 19%
2  2π  22  100 
10. (b) πd1 + πd2 = πd ⇒ d1 + d2 = d 2πr
21. (b) Perimeter = + 2r
2
 2 1 22 2  = πr + 2r
11. (a) Required area =  7 – × × 7  cm2
 4 7 
⇒ (π + 2) r = 36
= (49 – 38.5) cm2 = 10.5 cm2
 36  r r
12. (a) Let the radii of the outer and inner circles be r1 and ⇒   – r = 36
r2 respectively; we have  7 
Area = pr12 – pr22 = p(r12 – r22) ⇒ r = 7 cm
= p(r1 – r2) (r1 + r2) Hence, diameter = 7 × 2 = 14 cm.
= p(5.7 – 4.3) (5.7 + 4.3) = p × 1.4 × 10 sq. cm
22. (a) Required area = π (r + d )2 − r 2 
= 3.1416 × 14sq. cm. = 43.98 sq. cms.  
13. (b) Area of sector = 240°/360° × p(100)2 = 20933 cm2. d
Let r be the radius of the new circle, then
20933
20933 = pr2 ⇒ r = = 81.6 cm. r
π
14. (d) 15. (b)
16. (c) Joining B to O and C to O
Let the radius of the outer cirlce be r = π[r2 + d2 + 2rd – r2]
∴ perimeter = 2πr = π[d2 + 2rd] = πd[d + 2r]
But OQ = BC = r [diagonals of the square BQCO] 23. (a) A
∴ Perimeter of ABCD = 4r. 53
m
m

2πr π
Hence, ratio = =
35

4r 2
17. (c) Radius of outer concentric circle = (35 + 7) m = 42 m.
B 66 m C
22
Area of path = π (422 – 352) m2 = (422 – 352) m2 Here, a = 66 m, b = 53 m & c = 35 m
7
a + b + c 66 + 53 + 35
1 s= = = 77m
18. (c) Perimeter = × 2πr + 2r 2 2
4
Area of ∆ = s(s − a)(s − b)(s − c)
 1 22 
=  × × 7 + 2 × 7  cm = 25 cm
2 7  So, area of ∆ = 77(11)(24)(42) = 924
19. (d) 2πr = 4π ⇒ r = 2
πr2 = 2(924)
Area = π(2)2 = 4π
When, 2πr = 8π 2 × 924 + 7 r
⇒ r 2 = ⇒ r2 = 588
⇒r=4 22

Area = 16π ⇒ r = 14 3 m

20. (b) Let r be the radius of circle, then area = πr2 24. (a) Area of square = 2 cm2
Side of square = 2 cm
When r is diminished by 10%
Areas Related to Circles M-65
We know,
2
OP = cm, OQ = x cm Area of common region
2
Q = 2 (Area of sector – Area of ∆APQ)

 60° 1 
x = 2  × π(2) 2 − × (2) 2 × sin 60°
2
360° 2 
P O  4π 4 3 
x
= 2  − 
2  6 4 
 2
( 2)
2
⇒ x2 = +   2 
 2  = 2  (3.14) − (1.73)
3
2
⇒ x2 = 2 + = 2 (2.09 – 1.73) = 2 (0.36) = 0.72.
4
∴ Area of region lie between 0.7 and 0.75.
5 2 5 26. (a) A
⇒ x = ⇒x= cm.
2 2

5 r
AC = 2 cm (AC = Diameter) r
2 O
r
D a C
B C

1  circumference of circle = 2pr ...(i)


  × d1 × d 2 
a a 2  Area of DABC = [ar(DAOB) + ar(DBOC) + ar(DAOC)]
O
1 1
= AB × r + × BC × r + AC × r
2 2
A a B
1 1
= r [AB + BC + AC] = r × 7p...(ii)
1 2 2
Area of square = × AC × BD
2 From (i) and (ii),
1 Circmference of circle 2pr 4
Area of square = × d1 × d 2 = =
Area of triangle 1
2 r ¥ 7p 7
2
1 5 5
= ×2 ×2 = 5 cm2 180°
2 2 2 27. (b) As 1 radian = 1 degree ×
π
25. (c) Given,
2π  2π 180° 
Two circle each of radius is 2 and difference between their \ radian =  × 
3  3 π 
centre is 2 3
120
1 \ Time = = 20 min.
AB = 2 3 ⇒ AC = AB 6
2 2
 7  π ( 49)
AC = 3 = CB 28. (d) Area of the circle = π   = = 49 cm2.
 π π
P
154 154 × 7
Now, consider = = 49 cm2
2 2
π 22
 C
A
3 3
B
πr12 25
29. (c) =
πr22 16
Q
r1 5
⇒ =
r2 4
AC 3
In ∆APC, cos θ = = (∠C = 90°) 2πr1 5 5 × 125
AP 2 625
⇒ = = =
⇒ θ = 30° 2πr2 4 4 × 125 500
M-66 Mathematics
θ 30° 2 49π 39. (b)
Side of square = 6 × 7 = 42 cm.
30. (b) Area = × πr 2 = × π ( 7) =
360° 360° 12 Area of square = 42 × 42 = 1764 cm2
θ 60° 22 2 132
40. (d)
Area of each circular
31. (a) (a) Area= × πr 2 = × × (6 ) = cm2
360° 360° 7 7 22
= p(7)2 = × 49
θ 7
(b) Area of minor sector = × πr 2
360° = 154 cm2
60° 22 41. (a) Area of remaining
= × × 14 × 14 = 102.57 cm2
360° 7 portion = 1764 – 9 × 154 = 378 cm2
Area of major sector
42. (b)
Here r = 14 mm
= Area of circle – Area of minor sector
Length of silverwire
22
= (14) 2 –102.57 = 2pr + 8r
7
22
= 615.44 – 102.57 = 512.87 cm2 =2× × 14 + 8 × 14
7
C 2π ( 5) 2 = 200 mm
(c) = 2
=
A π ( 5) 5 43. (c) Area of each sector
1 22
 θ  = × × 14 × 14 = 77 mm2
(d) Given,   2πr = 22 8 7
 360° 
44. (d)
Circumference of inner part = 44 mm
 θ  2  θ  πr
∴ Area of sector =   πr =   ( 2r ) 22
 360°   360°  2 ⇒2× × r = 44
7
 θ   r  22 × 6
=   2πr   = = 66 cm2 ⇒ r = 7 mm
 360°  2 2
outer radius = 7 + 3 = 10 mm
32. (d)
outer circumference
33. (a) Justification : Let the radius of the circle be r units.
22
When 2 π r = π r2, (numerically) =2× × 10 = 62.86 mm
7
We have, 2 r = r2, i.e. r = 2 45. (c) Difference of areas
34. (a) 35. (b) 36. (a) 22
= ( 102 – 72)
Sol. (37-41) A 7
= 51 p mm2
46. (c) Number of revolution
Distance
=
O Outer circumference

cm 80π
32 = = 4.
20π
30° 47. (a) Both assertion and reason are correct. Also, reason is
B D C
the correct explanation of the assertion.
BD 22
cos 30° =
37. (b) C = 2 × × r = 176
32 7
BD = 16 3 cm. 176 × 7
⇒ r = = 28 cm.
side BC = 32 3 cm 2 × 22
48. (a) Both assertion and reason are correct. Also, reason is
38. (c) AD = AB2 − BD 2
the correct explanation of the assertion.
= (32 3)2 − (16 3)2  10 2  6 2 

π
Area of the path =   –   
= 48 cm 2 2
    
= π(25 – 9) = 16π
Areas Related to Circles M-67
49. (d) Assertion is not correct, but reason is true. 1 1
Area (∆ABC) = × AC × BC = (5) (12) = 30 cm2
2πr = 22 2 2
⇒ r = 3.5 cm = 0.003 m2
22 (C) (AC)2 = (AB)2 + (BC)2 = 64 + 36 = 100
∴ Area of the circle = × 3.5 × 3.5 = 38.5 cm2.
7 AC = 10 cm
50. (A) → (q); (B) → (r); (C) → (p); (D) → (s) Area of the shaded region
51. (A) → (q); (B) → (r); (C) → (s); (D) → (p) = (area of the circle) – (area of the rectangle ABCD)
52. (A) → (t); (B) → (r); (C) → (s); (D) → (p); (E) → (q)  22  10 2 
53. (A) → (q); (B) → (p); (C) → (s); (D) → (r) =  ×    – (8 × 6)
 7  2  
(A) Area of the shaded region
= (78.57 – 48) = 30.57cm2
= 90° π ( 26)2 – ( 23)2  60° 22 2
360°   (D) Area of the shaded region = × × (14.8)
360° 7
1 22 
= × ( 26) 2 – ( 23) 2  = 115.5 ≈ 115 m2 = 114.7 m2
4 7 
54. minor 55. radii 56. segment
AB = 2 × AD = 2 × 6.5 = 13 cm
(B)
θ
2 2
57. 2πr. 58. πr2 59. × 2πr
BC = ( AB ) – ( AC ) 360°
60. boundary 61. 44 cm 62. 132/7 cm2
2 2
= (13) – ( 5) = 12 cm 63. True 64. True 65. True
66. True 67. True 68. True
69. True 70. False 71. False
8 Acids, Bases and
Probability
Salts
3 5
Multiple Choice Questions (MCQs) (a) (b)
5 12
7 4
DIRECTIONS : This section contains multiple choice (c) (d)
12 5
questions. Each question has 4 choices (a), (b), (c) and (d) out
of which only one is correct. 6.
Two fair dice are thrown. Find the probability that both
dice show different numbers.
1. Two dice are thrown at a time, then find the probability that 1 5
the difference of the numbers shown on the dice is 1. (a) (b)
6 6
3 5 32 29
(a) (b) (c) (d)
16 18 36 36
7 7
(c) (d) 7. A bag contains 40 coins, consisting of `2, `5 and
36 18
`10 denominations. If a coin is drawn at random, the
2. A book containing 100 pages is opened at random. Find 5
probability of drawing a `2 coin is . If x number of `2
the probability that a doublet page is found. 8
8 9 coins are removed from the bag and then a coin is drawn
(a) (b)
25 100 1
at random, the probability of drawing a `2 coin is . Find
7 11 the value of x. 2
(c) (d)
100 100 (a) 5 (b) 2
(c) 10 (d) 8
3.
When two dice are thrown, find the probability of getting
a number always greater than 4 on the second dice. 8.
A factory has 120 workers in January, 90 of them are
2 1 female workers. In February, another 15 male workers
(a) (b)
3 3 were employed. A worker is then picked at random.
3 2 Calculate the probability of picking a female worker.
(c) (d) 4
5 5 3
(a) (b)
4 9
4.
Two numbers are chosen from 1 to 5. Find the probability
2 1
for the two numbers to be consecutive. (c) (d)
3 2
3 2
(a) (b)
5 5 9.
A box contains a number of marbles with serial number 18
4 2 to 38. A marble is picked at a random. Find the probability
(c) (d) that it is a multiple of 3.
5 3
3 7
5. An unbiased die is rolled twice. Find the probability of (a) (b)
5 20
getting the sum of two numbers as a prime
3 1
(c) (d)
4 3
Probability M-69
10. One card is drawn from a well shuffled deck of 52 cards. 18. A single letter is selected at random from the word
I. The probability that the card will be diamond, is 1/2. “PROBABILITY”. The probability that the selected letter
II. The probability of an ace of heart is 1/52. is a vowel is
III. The probability of not a heart is 3/4. 2 3
(a) (b)
IV. The probability of king or queen is 1/26. 11 11
Which of the statement(s) is/are true? 4
(c) (d) 0
(a) I and II (b) II and III 11
(c) III and IV (d) None of these
19. A three digit number is to be formed using the digits 3, 4,
11. A fair die is thrown once. The probability of getting a
7, 8 and 2 without repetition. The probability that it is an
composite number less than 5 is
odd number is
1 1
(a) (b) 2 1
3 6 (a) (b)
5 5
2 4 3
(c) (d) 0 (c) (d)
3 5 5
12. If a letter is chosen at random from the letter of English
20. An urn contains 6 blue and ‘a’ green balls. If the probability
alphabet, then the probability that it is a letter of the word
of drawing a green ball is double that of drawing a blue
‘DELHI’ is
ball, then ‘a’ is equal to
1 1
(a) (b) (a) 6 (b) 18
5 26 (c) 24 (d) 12
5 21
(c) (d) 21. Two coins are tossed simultaneously. The probability of
26 26
getting at most one head is
13. The probability of raining on day 1 is 0.2 and on day 2 is 1 1
(a) (b)
0.3. The probability of raining on both the days is 4 2
(a) 0.2 (b) 0.1 3
(c) (d) 1
(c) 0.06 (d) 0.25 4
14. Which of the following cannot be the probability of an 22. If in a lottery, there are 5 prizes and 20 blanks, then the
event? probability of getting a prize is
(a) 2/3 (b) – 1/5 2 4
(c) 15% (d) 0.7 (a) (b)
5 5
15. The probability that a two digit number selected at random 1
(c) (d) 1
will be a multiple of ‘3’ and not a multiple of ‘5’ is 5
2 4
(a) (b) 23. Which of the following relationship is the correct ?
15 15
P (E) + P ( E ) = 1
(a)
1 4
(c) (d)
15 90 (b) P ( E ) – P(E) = 1

16. The probability of getting a number greater than 2 in P(E) = 1 + P ( E )


(c)
throwing a die is (d) None of these
(a) 2/3 (b) 1/3
(c) 4/3 (d) 1/4 24. For an event E, P (E) + P ( E ) = q, then
17. Out of one digit prime numbers, one number is selected at 0 ≤ q <1
(a) (b) 0 < q ≤ 1
random. The probability of selecting an even number is
(c) 0 < q < 1 (d) None of these
1 1
(a) (b)
2 4 25. A girl calculates that the probability of her winning the
first prize in a lottery is 0.08. If 6000 tickets are sold, how
4 2
(c) (d) many tickets has she bought?
9 5
(a) 40 (b) 240
(c) 480 (d) 750
M-70 Mathematics
26. Three - digit numbers formed by using digits 0, 1, 2 and 33. Two dice are rolled simultaneously. Find the probability
5 (without repetition) are written on different slips with that they show different faces.
distinct number on each slip, and put in a bowl. One slip 3 1
(a) (b)
is drawn at random from the bowl. The probability that 4 6
the slip bears a number divisible by 5 is 1 5
5 4 (c) (d)
(a) (b) 3 6
9 9
34. A die is thrown once then,
2 1
(c) (d) 2
3 3 (a) the probability of getting an odd number is
3
27. A box contains four cards numbered as 1, 2, 3 and 4 and (b) the probability of getting multiple of 3 is 1/3
another box contains four cards numbered as 1, 4, 9 and (c) the probability of getting a prime number is 2/3
16. One card is drawn at random from each box. What is (d) the probability of getting number greater than 5 is 1/3
the probability of getting the product of the two numbers
so obtained , more than 16? 35. The probability of an event can not be
5 1 (a) positive (b) negative
(a) (b) (c) zero (d) one
8 2
3 1 36. If the probability of an event is 0, then it is called a
(c) (d)
8 4 (a) sure event (b) certain event
28. From the data (1, 4, 7, 16, 27, 29) if 29 is removed, the (c) impossible event (d) none of these
probability of getting a prime number is 37. Which of the following statement(s) is/are not correct ?
1 1 (a) An event which can never happen is called impossible
(a) (b)
2 5 event.
2 1 (b) A n event which has more than one (favourable)
(c) (d) outcomes is called a compound event.
5 3
(c) T he collection of all possible outcomes of an
29. A bag contains card numbers 3, 4, 5, 6, 7....27. One card
experiment is called sample space.
is drawn, then probability of prime number card is
(d) None of these
9 8
(a) (b)
25 27
8 1
(c) (d)
25 5
DIRECTIONS : Study the given Case/Passage and answer the
30. Two dice are rolled, then probability of getting a total of following questions.
9 is
1 1 Case/Passage-I
(a) (b)
3 9 On a weekend Rani was playing cards with her family. The
9 8 deck has 52 cards. If her brother drew one card .
(c) (d)
10 9
31. From a bag containing 100 tickets numbered 1, 2, 3, .........,
100 one ticket is drawn. If the number on this ticket is x,
1
then the probability that x + > 2 is ......
x
(a) 0 (b) 0.99
(c) 1 (d) None of these
32. A coin is tossed. Then the probability of getting either head
or tail is
1 [From CBSE Question Bank-2021]
(a) 1 (b)
3 38. Find the probability of getting a king of red colour.
1 1 1 1 1
(c) (d) 1
2 4 (a) (b) (c) (d)
26 13 52 4
Probability M-71
39. Find the probability of getting a face card.
1 1 2 3
Assertion & Reason
(a) (b) (c) (d)
26 13 13 13
40. Find the probability of getting a jack of hearts. DIRECTIONS : Each of these questions contains an Assertion
followed by Reason. Read them carefully and answer the
1 1 3 3
(a) (b) (c) (d) question on the basis of following options. You have to select
26 52 52 26
the one that best describes the two statements.
41. Find the probability of getting a red face card.
(a) If both Assertion and Reason are correct and Reason is
3 1 1 1
(a) (b) (c) (d) the correct explanation of Assertion.
26 13 52 4
(b) If both Assertion and Reason are correct, but Reason is
42. Find the probability of getting a spade.
not the correct explanation of Assertion.
1 1 1 1 (c) If Assertion is correct but Reason is incorrect.
(a) (b) (c) (d)
26 13 26 4 (d) If Assertion is incorrect but Reason is correct.
Case/Passage-II 48. Assertion : If a box contains 5 white, 2 red and 4 black
Rahul and Ravi planned to play Business (board game) in marbles, then the probability of not drawing a white marble
which they were supposed to use two dice. 5
from the box is .
11

Reason : P ( E ) = 1 – P( E ), where E is any event.

49. Assertion : In rolling a dice, the probability of getting


number 8 is zero.
Reason : Its an impossible event.
[From CBSE Question Bank-2021]
50. Assertion : An event is very unlikely to happen. Its
43. Ravi got first chance to roll the dice. What is the
probability is 0.0001
probability that he got the sum of the two numbers
appearing on the top face of the dice is 8? Reason : If P(A) denote the probability of an event
1 5 1 A, then 0 < P(A) < 1.
(a) (b) (c) (d) 0
26 36 18 51. Assertion : If the probability of an event is P then
44. Rahul got next chance. What is the probability that he got probability of its complementary event will be 1 – P.
the sum of the two numbers appearing on the top face of Reason : When E and E are complementary events, then
the dice is 13? P(E) + P ( E ) = 1
5 1
(a) 1 (b) (c) (d) 0 52. Assertion : If a die is thrown, the probability of getting a
36 18 number less than 3 and greater than 2 is zero.
45. Now it was Ravi’s turn. He rolled the dice. What is Reason : Probability of an impossible event is zero.
the probability that he got the sum of the two numbers
appearing on the top face of the dice is less than or equal
to 12 ? Match the Following
5 1
(a) 1 (b) (c) (d) 0 DIRECTIONS : Each question contains statements given in
36 18
46. Rahul got next chance. What is the probability that he got two columns which have to be matched. Statements (A, B, C,
the sum of the two numbers appearing on the top face of D,............) in Column-I have to be matched with statements
the dice is equal to 7 ? (p, q, r, s,...............) in Column-II.
5 5 1
(a) (b) (c) (d) 0 53. Match the proposed probability under Column-I with the
9 36 6
appropriate written description under column-II :
47. Now it was Ravi’s turn. He rolled the dice. What is Column -I Column-II
the probability that he got the sum of the two numbers
appearing on the top face of the dice is greater than 8 ? (Probability) (Written Description)
5 1 5 (A) 0.95 (p) An incorrect assignment
(a) 1 (b) (c) (d)
36 18 18 (B) 0.02 (q) No chance of happening
M-72 Mathematics
(C) –0.3 (r) A
 s much chance of happening 62. If A is an event of a random experiment, then AC or A or
as not A′ is called the ..................of the event.
(D) 0.5 (s) Very likely to happen
63. A set of events which have no pair in common are called
(E) 0 (t) Very little chance of happening .............
54. Two unbiased coins are tossed simultaneously. Match 64. An outcome of a random experiment is called an .............
Column-I with the probabilities given in Column-II. event.
Column-I Column-II
3
(A) The probability (p) True / False
4
of getting one head is
1 DIRECTIONS : Read the following statements and write your
(B) The probability of (q)
4 answer as true or false.
getting at least one
head is
65. The sum of the probabilities of all the elementary events
1
(C) The probability of (r) of an experiment is 1.
2
getting two heads is
66. For any event E, P (E) + P ( E ) = 1, where E stands for
‘not E’. E and E are called complementary events.
Fill in the Blanks
67. The probability of an event can be greater than 1.

DIRECTIONS : Complete the following statements with an 68. If the probability of an event is 1, then it is an impossible
appropriate word / term to be filled in the blank space(s). event.
69. If A is any event in a sample space, then P( A) = 1 + P ( A)
55. Probability of an event E + Probability of the event ‘not
E’ = .................... 70. The sum of probabilities of two students getting distinction
in their final examinations is 1.2.
56. The probability of an event that cannot happen is
................. Such an event is called .................. 71. A bag contains 5 red balls and some blue balls. If the
probability of drawing a blue ball is double that of a red
57. The probability of an event that is certain to happen is
ball, the number of blue balls in the bag is 10.
............... Such an event is called ................
72. A box contains 90 discs which are numbered from 1 to 90.
58. The sum of the probabilities of all the elementary events
If one disc is drawn at random from the box, the probability
of an experiment is ................
that it bears a two-digit number is 0.9
59. The probability of an event is greater than or equal to .........
73. An event A associated to a random experiment is said to
and less than or equal to ..............
occur if any one of the elementary events associated to the
60. If P(E) = 0.05, the probability of ‘not E’ is ........... event A is an outcomes.
61. A die is thrown once, the probability of getting a prime 74. An event associated to a random experiment is a
number is .............. compound event if it is obtained by combining two or more
elementary events associated to the random experiment.
Probability M-73

ANSWER KEY & SOLUTIONS


1. (b) n(S) = 6 × 6 = 36 6. (b) S = {(1, 1), ..., (1, 6), (2, 1), ..., (2, 6), (3, 1), ..., (3, 6),
(4, 1), ..., (4, 6), (5, 1), ..., (5, 6), (6, 1), ...., (6, 6)}
E= {(1, 2), (2, 1), (2, 3), (3, 2), (3, 4), (4, 3), (4, 5),
(5, 4), (5, 6), (6, 5)} n(S) = 36
n(E) = 10 Let E be the event that both dice show different
n(E) 10 5 numbers.
\ P(E) = = =
n(S) 36 18 E {(1, 2), (1, 3),...., (1, 6), (2, 1), (2, 3), (2, 4),...., (2, 6),
2. (b) S = {1, 2, 3, ......, 100} (3, 1), (3, 2), (3, 4), (3, 5), (3, 6), (4, 1), (4, 2), (4, 3),
(4, 5), (4, 6), (5, 1), (5, 2), (5, 3), (5, 4), (5, 6), (6, 1),
n(S) = 100 (6, 2), (6, 3), (6, 4), (4, 5)}
E = {11, 22, 33, 44, 55, 66, 77, 88, 99} n(E) = 30
n(E) = 9
n ( E) 30 5
9 \ P ( E) = = =
\ P(E) = n (S) 36 6
100
7. (c) n(S) = 40, let n(C) = C
3. (b) n(S) = 6× 6 = 36, E = {(1, 5), (2, 5), (3, 5), (4, 5),
(5, 5), (6, 5), (1, 6), (2, 6), (3, 6), (4, 6), (5, 6), (6, 6)} 5 C 5
P(C) = ⇒ = or C = 25
8 40 8
n(E) = 12
n(E) 12 1 25 − x 1
\ P(E) = = = Now, = ⇒ x = 10
n(S) 36 3 40 − x 2

4. (b) E = {(1, 2), (2, 3), (3, 4), (4, 5)} 8. (c) Initial number of workers = 120

n(E) = 4 When 15 male workers are added, then the total


number of workers = 120 + 15 = 135
n(E) 4 2
n(S) = 5 × 2 = 10 \ P(E) = = =
n(S) 10 5 Number of female workers = 90

5. (b) The sum of the two numbers lies between 2 and 12. 90 2
\ Probability of female workers = =
So, the primes are 2, 3, 5, 7, 11. 135 3
9. (d) Total number of marbles = 38 – 18 + 1 = 21
No. of ways for getting 2 = (1, 1) = 1
The multiples of 3 from 18 to 38 are 18, 21, 24, 27,
No. of ways of getting 3 = (1, 2), (2, 1) = 2
30, 33, 36.
No. of ways of getting 5
These are 7 in numbers
= (1, 4), (4, 1), (2, 3), (3, 2) = 4 7 1
∴ Required probability = =
No. of ways of getting 7 21 3
10. (b) Total number of cards = 52
= (1, 6), (6, 1), (2, 5), (5, 2), (3, 4), (4, 3) = 6
Total number of diamond cards = 13
No. of ways of getting 11 = (5, 6), (6, 5) = 2
I. P(diamond cards) = 13/52 = 1/4
No. of favourable ways = 1 + 2 + 4 + 6 + 2 = 15 II. P(an ace of heart) = 1/52
No. of exhaustive ways = 6 × 6 = 36 1 3
III. P(not a heart) = 1 − =
\ Probability of getting the sum as a prime 4 4
15 5 4 4 8 2
= = IV. P(king or queen) = + = =
36 12 52 52 52 13
M-74 Mathematics
11. (b) [Hint. The outcomes are 1, 2, 3, 4, 5, 6. Out of these, So, required probability of getting the product of the
4 is the only composite number which is less than 5]. 6 3
two numbers so obtained is =
12. (c) [Hint. The English alphabet has 26 letters in all. 16 8
The word ‘DELHI’ has 5 letter, so the number of No. of favourable outcomes 1
28. (b) Probability = =
favourable outcomes = 5.] Total number of outcomes 5
13. (c) P(raining on both day) = 0.2 × 0.3 = 0.06 29. (c) Total number of cards = 25
(Because both independent event) Prime number are 3, 5, 7, 11, 13, 17, 19, 23,
14. (b) 8
∴ Probability of prime number card =
15. (b) 24 out of the 90 two digit numbers are divisible by 25
‘3’ and not by ‘5’. 30. (b) No. of sample space = 6 × 6 = 36
24 4 Sum total of 9 = (3, 6), (4, 5), (5, 4), (6, 3)
The required probability is therefore, = .
90 15
4 1
∴ P = =
4 2 36 9
16. (a) Required probability = = .
6 3
31. (b) n(S) = [1, 2, 3, ..., 100] = 100
17. (b) [Hint. One digit prime numbers are 2, 3, 5, 7. Out of 1
these numbers, only the number 2 is even.] Q x + >2
x
1+ 2 +1 4
18. (c) Required probability = = . \ x2 + 1 > 2x
11 11
⇒ x2 – 2x + 1 > 0
19. (a) There are 2 favourable choice (3, 7) for unit place.
⇒ (x – 1)2 > 0
2 2
P = 1× 1× = x = [2, 3, ... ,100]
5 5
n(E) = [2, 3, 4, ..., 100] = 99
20. (d)
99
21. (c) Total outcomes = HH, HT, TH, TT P(E) = = 0.99
100
Favourable outcomes = HT, TH, TT 32. (a) 33. (d)
3 34. (b) A die is thrown once therefore, total number of
P(at most one head) = .
4 outcomes are {1, 2, 3, 4, 5, 6}
P(odd number) = 3/6 = 1/2
(a)
5 1
22. (c) Required probability = = .
25 5 (b) P(multiple of 3) = 2/6 = 1/3
P(prime number) = 3/6 = 1/2
(c)
23. (a) P (E) + P ( E ) = 1
P(greater than 5) = 1/6
(d)
24. (d) 25. (c)
35. (b) The probability of an event can never be negative.
26. (a) Total three digit number are : 3 × 3 × 2 = 18
36. (c)
Now, numbers divisible by 5 are :
37. (d) All the three statements are correct.
2 × 3 × 1 + 2 × 2 × 1 = 10
2 1
So, probability that the slip bears a number divisible 38. (a) P(king of red colour) = =
52 26
10 5 12 3
by 5 = = 39. (d) P(getting a face card) = =
18 9 52 13
27. (c) Possible products are 1, 4, 9, 16, 2, 8, 18, 32, 3, 12, 1
40. (b) P(getting a jack of hearts) =
27, 48, 4, 16, 36, 64 52
Probability M-75
3 49. (a) Assertion and Reason both are correct. Also Reason
41. (a) P(getting a red face card) = is the correct explanation of the Assertion.
26
13 1 50. (b) Assertion and Reason is correct but Reason is not
42. (d) P(getting a spade) = =
correct explanation for Assertion.
52 4
43. (b) Sum of the two numbers appearing on the top face 51. (a) Both statements are correct and Reason is the correct
of dice is 8. for Assertion.
52. (a) Both statements are correct. Event given in Assertion
(2, 6), (3, 5), (4, 4) (5, 3), (6, 2)
is an impossible event.
5 53. (A) → (s); (B) → (t); (C) → (p); (D) → (r); (E) → (q)
\ Required probability =
36 54. (A) → (r) ; (B) → (p) ; (C) → (q)
44. (d) Since, the sum of two numbers appearing on the top
Sample space = {HH, HT, TH, TT}
face of dice cannot be 13.
2 1
So, required probability = 0. A : P (one head) = =
4 2
45. (a) Since, the pair of number whose sum is less than 0
(∵ favourable event = HT, TH)
or equal to 12 in a pair of dice is 36.
3
36 B : P (at least one head) =
\ Required probability = =1 4
36
46. (c) Since, the pair of numbers on the top of dice whose (∵ favourable event = HH, HT, TH)
sum is 7 are (1, 6), (2, 5), (3, 4), (4, 3), (5, 2) , (6, 1)
6 1 1
\ Required probability = = C : P (two heads) = (∵ favourable event = HH)
36 6 4
5 55. 1 56. 0, impossible event
47. (d)
18 57. 1, sure or certain event 58. 1
59. 0, 1 60. .95
48. (d) Assertion is not correct, but reason is correct.
61. 1/2 62. complement
5 5
P(white marble) = = . 63. mutually exclusive 64. elementary
5 + 2 + 4 11
65. True
66. True 67. False 68. False
5 11 – 5 6 69. False 70. True 71. True 72. True
P(not white marble) = 1 – = = .
11 11 11
73. True 74. True
9 Acids, Bases and
Quadratic
Salts
Equations
7. Find the product of the roots of x2 + 8x – 16 = 0
Multiple Choice Questions (MCQs) (a) 8 (b) –8
(c) 16 (d) –16
DIRECTIONS : This section contains multiple choice
questions. Each question has 4 choices (a), (b), (c) and (d) out 8. If the roots of the equation ax2 + bx + c = 0 are α and β,
of which ONLY ONE is correct. then the quadratic equation whose roots are –α and –β
is _____ .
1. If the product of roots of the equation x3 – 3x + k = 10 is (a) ax2 – bx – c = 0 (b) ax2 – bx + c = 0
–2, then the value of k is
ax2 + bx – c = 0
(c) (d) ax2 – bx + 2c = 0
(a) –2 (b) –8
(c) 8 (d) 12 9. If the equation
(1 + m2) x2 + (2mc) x + (c2 – a2) = 0 has equal roots,
2. If one root of 5x2 + 13x + k = 0 be the reciprocal of the
then
other root, then the value of k is
(a) c2 – a2 = 1 + m2 (b) c2 = a2 (1 + m2)
(a) 0 (b) 1
c2a2 = (1 + m2)
(c) c2 + a2 = 1 + m2
(d)
(c) 2 (d) 5
10. Which of the following satisfy the equation
3. If the sum of the roots of a quadratic equation is 6 and their
product is 6, the equation is a2b2x2 + b2x – a2x – 1 = 0
(a) x2 – 6x + 6 = 0 (b) x2 + 6x – 6 = 0 1 1
(a) 2 (b)
x2 – 6x
(c) –6=0 (d) x2 + 6x + 6 = 0 a b2
4. If the equation x2 + 2(k + 2)x + 9k = 0 has equal roots, then −1
(c) (d) None of these
k=? b2
(a) 1 or 4 (b) –1 or 4
11. The roots of the quadratic equation x2 – 0.04 = 0 are
(c) 1 or – 4 (d) –1 or – 4
(a) ± 0.2 (b) ± 0.02
5. If the roots of 5x2 – kx + 1 = 0 are real and distinct, then
(c) 0.4 (d) 2
−2 5 < k < 2 5
(a)
12. One of the two students, while solving a quadratic equation
(b)
k > 2 5 only in x, copied the constant term incorrectly and got the roots
(c)
k < −2 5 only 3 and 2. The other copied the constant term and coefficient
of x2 correctly as –6 and 1 respectively. The correct roots
(d) either k > 2 5 or k < −2 5 are
6. If a – b, b – c are the roots of ax2 + bx + c = 0, then find (a) 3, –2 (b) –3, 2
(a − b)(b − c) (c) –6, –1 (d) 6, –1
the value of
c−a 13. The condition for one root of the quadratic equation
b c ax2 + bx + c = 0 to be twice the other, is
(a) (b)
c b b2 = 4ac
(a) (b) 2b2 = 9ac
ab bc c2 = 4a + b2
(c) (d) c2 = 9a – b2
(c) (d)
c a
Quadratic Equations M-77
2 2 24. If the equation (m2 + n2) x2 –2 (mp + nq) x + p2 + q2 = 0
1 3
14. If  x −  −  x −  = x + 2 , then x = has equal roots, then
 2  2
mp = nq
(a) mq = np
(b)
(a) 3 (b) 2
(c) 4 (d) None of these mn = pq
(c) mq =
(d) np

15. If x2 + y2 = 25, xy = 12, then x = 25. Each root of x2 – bx + c = 0 is decreased by 2. The resulting
(a) {3, 4} (b) {3, –3} equation is x2 – 2x + 1 = 0, then
(c) {3, 4, –3, –4} (d) {–3, –3} b = 6, c = 9
(a) (b) b = 3, c = 5

1 b = 2, c = –1
(c) (d) b = – 4, c = 3
16. If x = 7 + 4 3 , then x + = 26. Two distinct polynomials f (x) and g(x) are defined as
x
(a) 4 (b) 6 follows:
f (x) = x2 + ax + 2; g (x) = x2 + 2x + a.
(c) 3 (d) 2
If the equations f (x) = 0 and g(x) = 0 have a common
17. If the roots of the equation px 2 + 2qx + r = 0 and root, then the sum of the roots of the equation
qx 2 − 2 pr x + q = 0 be real, then f (x) + g(x) = 0 is

(a) p = q (b) q2 = pr 1
-
(a) (b) 0
2 2
(c) p = qr (d) r2 = pq
18. The equation 2x2 + 2(p + 1) x + p = 0, where p is real, 1
(c) (d) l
always has roots that are 2
(a) Equal 27. If a and b are the roots of the quadratic equation
x2 – 6x – 2 = 0 and if an = an – bn, then the value of
(b) Equal in magnitude but opposite in sign a10 − 2a 8
(c) Irrational is
2a 9
(d) Real
(a) 6.0 (b) 5.2
19. If the ratio of the roots of the equation x2 + bx + c = 0 is
the same as that of x2 + qx + r = 0, then (c) 5.0 (d) 3.0
(a) r2b = qc2 (b) r2c = qb2 28. Consider the quadratic equation nx 2 + 7 nx + n = 0,
(c) c2r = q2b (d) b2r = q2c where n is a positive integer. Which of the following
statements are necessarily correct?
20. The real roots of the equation x 2/3 + x1/3 − 2 = 0 are
I. For any n, the roots are distinct.
(a) 1, 8 (b) –1, –8
II. There are infinitely many values of n for which both
(c) –1, 8 (d) 1, –8
roots are real.
21. Which of the following is not a quadratic equation?
III. The product of the roots is necessarily an integer.
(a) x2 – 2x + 2 (3 – x) = 0
(a) III only (b) I and III
x (x + 1) + 1 = (x – 2) (x – 5)
(b)
(c) (2x – 1) (x – 3) = (x + 5) (x – 1) (c) II and III (d) I, II and III

(d) x3 – 4x2 – x + 1 = (x –2)3 29. Two quadratic equations x2 – bx + 6 = 0 and x2 – 6x + c – 0


have a common root. If the remaining roots of the first and
22. If one root of the quadratic equation ax2 + bx + c = 0 is
the reciprocal of the other, then second equations are positive integers and are in the ration
3 : 4 respectively, then the common root is
(a) b = c (b) a=b
(c) ac = 1 (d) a = c (a) 1 (b) 2

1 1 (c) 3 (d) 4
23. The roots of the equation x + = 3 , x ≠ 0, are 30. The values of k, so that the equations 2x2 + kx – 5 = 0 and
x 3
x2 – 3x – 4 = 0 have one root in common, are
1
(a) 3, 1 (b) 3, 27 27
3 (a) 3, (b) 9,
2 4
1 1 − 27 4
(c) 3, − (d) – 3, − (c) – 3, (d) 3,
3 3 4 27
M-78 Mathematics
38. Which of the following equations has 2 as a root?
3+ 5
31. If x = and y = x3, then y satisfies the quadratic (a) x2 – 4x + 5 = 0
2
equation x2 + 3x – 12 = 0
(b)
(a) y2 – 18y + 1 = 0 (b) y2 + 18y + 1 = 0 (c) 2x2 – 7x + 6 = 0
(c) y2 – 18y – 1 = 0 (d) y2 + 18y – 1 = 0 (d) 3x2 – 6x – 2 = 0
32. Let b be a non-zero real number. Suppose the quadratic 39. Values of k for which the quadratic equation 2x2– kx + k = 0
1 has equal roots is
equation 2x2 + bx + = 0 has two distinct real roots.
Then b (a) 0 only (b) 4 only

1 5 1 5 (c) 8 only (d) 0, 8


b + >
(a) (b) b+ < 40. If α, β are roots of the equation x2 – 5x + 6 = 0, then the
b 2 b 2
equation whose roots are α + 3 and β + 3 is
1
b2 – 3b > –2
(c) (d) b2 + <4 (a) 2x2 – 11x + 30 = 0 (b) –x2 + 11x = 0
b2
x2 – 11x + 30 = 0
(c) (d) 2x2 – 22x + 40 = 0
33. If the quadratic equations 2x2 + 4x + (a + 5) = 0 have equal
roots and (a + 4)x2 + ax – 3b = 0 have distinct real roots 41. If equation x2 – (2 + m) x + 1 (m2 – 4m + 4) = 0 has equal
then which of the following is true: roots, then:
3 3 (a) m = 0 (b) m = 6
(a) a = –3, b < (b) a = 3, b >
4 4 m = 2
(c) (d) m = 3
3 3 42. Which of the following equations have no real roots?
a = –3, b > – (d)
(c) a = 3, b <
4 4
x 2 − 2 3 x + 5 = 0 (b)
(a) 2 x2 + 6 2 + 8 = 0
34. The value of l such that sum of the squares of the roots of
(c) x 2 − 2 3 x − 5 = 0 (d) 2 x2 − 6 2 x − 9 = 0
the quadratic equation, x2 + (3 – l)x + 2 = l has the least
value is: 43. Two numbers whose sum is 8 and the absolute value of
whose difference is 10 are roots of the equation
15
(a) (b) 1 x2 – 8x + 9 = 0
(a) (b) x2 – 8x – 9 = 0
8
x2 + 8x – 9 = 0
(c) (d) –x2 + 8x + 9 = 0
(c) (d) 2 44. If α, β are roots of x2 + 5x + a = 0 and 2α + 5β = –1,
then
35. If α and β be two roots of the equation x2 – 64x + 256 = 0.
1 1 (a)
α = 8 (b) β = –3
 α 3  8  β3  8 (c) α = 9 (d) a = – 24
Then the value of  5  +  5  is:
β  α  45. The value of p for which the difference between the roots
of the equation x2 + px + 8 = 0 is 2, are
(a) 2 (b) 3
(a) 4 (b) 8
(c) 1 (d) 4
(c) 6 (d) – 4
36. Which one of the following is not a quadratic equation? 2
46. If the roots of x + px + 12 = 0 are in the ratio 1 : 3, then
(a) (x + 2)2 = 2(x + 3) value(s) of p are
x2 + 3x = (–1) (1 – 3x)2
(b) (a) 3 (b) 8
(c) (x + 2) (x – 1) = x2 – 2x – 3 (c) 6 (d) – 3
x3 – x2 + 2x + 1 = (x + 1)3
(d) 47. Roots of quadratic equation x2 – 3x + 2 = 0 are
(a) 3 (b) –1
37. Which constant should be added and subtracted to solve
(c) 2 (d) 4
the quadratic equation 4 x 2 – 3 x – 5 = 0 by the method
48. If x = 2 and x = 3 are roots of the equation
of completing the square?
3x2 – 2px + 2q = 0, then
9 3
(a) (b) 2
16 64 p=
(a) (b) p = 15
15
3 3
(c) (d) q = 9
(c) (d) 6p + 2q = 27
4 4
Quadratic Equations M-79
55. Which is the correct quadratic equation for the speed of
the current ?
(a) x2+ 30x − 200 = 0 (b) x2+ 20x − 400 = 0
DIRECTIONS : Study the given Case/Passage and answer the (c) x 2+ 30x − 400 = 0 (d) x2− 20x − 400 = 0
following questions. 56. What is the speed of current ?
Case/Passage-I (a) 20 km/hour (b) 10 km/hour
Raj and Ajay are very close friends. Both the families decide to
(c) 15 km/hour (d) 25 km/hour
go to Ranikhet by their own cars. Raj’s car travels at a speed of
x km/h while Ajay’s car travels 5 km/h faster than Raj’s car. Raj 57. How much time boat took in downstream?
took 4 hours more than Ajay to complete the journey of 400 km. (a) 90 minute (b) 15 minute
[From CBSE Question Bank-2021] (c)
30 minute (d)
45 minute

Assertion & Reason

DIRECTIONS : Each of these questions contains an Assertion


followed by Reason. Read them carefully and answer the
question on the basis of following options. You have to select
the one that best describes the two statements.
(a) If both Assertion and Reason are correct and Reason is
49. What will be the distance covered by Ajay’s car in two the correct explanation of Assertion.
hours? (b) If both Assertion and Reason are correct, but Reason is
(a) 2(x +5)km (b) (x – 5)km not the correct explanation of Assertion.
(c) 2( x + 10)km (d) (2x + 5)km (c) If Assertion is correct but Reason is incorrect.
50. Which of the following quadratic equation describe the (d) If Assertion is incorrect but Reason is correct.
speed of Raj’s car? 58. Assertion : If roots of the equation x2 – bx + c = 0 are two
(a) x2 – 5 x – 500 = 0 (b) x2 + 4x - 400 = 0 consecutive integers, then b2 – 4c = 1.
(c) x2 + 5x – 500 = 0 (d) x2 – 4x + 400 = 0 Reason : If a, b, c are odd integer then the roots of the
51. What is the speed of Raj’s car? equation 4abc x2 + (b2 – 4ac) x – b = 0 are real and distinct.
(a) 20 km/hour (b) 15 km/hour 59. Assertion : (2x – 1)2 – 4x2 + 5 = 0 is not a quadratic
(c) 25 km/hour (d) 10 km/hour equation.
52. How much time took Ajay to travel 400 km? Reason : x = 0, 3 are the roots of the equation 2x2 – 6x = 0.
(a) 20 hour (b) 40 hour 60. Assertion : The equation 9x2 + 3kx + 4 = 0 has equal roots
for k = ± 4.
(c) 25 hour (d) 16 hour
Reason : If discriminant ‘D’ of a quadratic equation is
Case/Passage-II equal to zero then the roots of equation are real and equal.
The speed of a motor boat is 20 km/hr. For covering the 61. Assertion : 4x2 – 12x + 9 = 0 has repeated roots.
distance of 15 km the boattook 1 hour more for upstream than
Reason : The quadratic equation ax2 + bx + c = 0 have
downstream. [From CBSE Question Bank-2021]
DOWNSTREAM(a) UPSTREAM(b)
repeated roots if discriminant D > 0.
62. Assertion : A quadratic equation ax2 + bx + c = 0, has two
distinct real roots, if b2 – 4ac > 0.
Reason : A quadratic equation can never be solved by
using method of completing the squares.
63. Assertion : Sum and product of roots of 2x2 – 3x + 5 = 0
53. Let speed of the stream be x km/hr. then speed of the 3 5
motorboat in upstream will be are and respectively.
2 2
(a) 20 km/hr (b) (20 + x) km/hr
Reason : If α and β are the roots of ax2 + bx + c = 0, a ≠ 0,
(c) (20 - x) km/hr (d) 2 km/hr
54. What is the relation between speed ,distance and time? b
then sum of roots = α + β = – and product of roots
(a) speed = (distance )/time (b) distance = (speed )/time a
c
(c) time = speed x distance (d) speed = distance x time = αβ = .
a
M-80 Mathematics
70. The equation ax2 + bx + c = 0, a ≠ 0 has no real roots, if
Match the Following .........
71. The values of k for which the equation 2x2 + kx + x + 8 = 0
DIRECTIONS : Each question contains statements given in
will have real and equal roots are .............
two columns which have to be matched. Statements (A, B, C, D)
in column-I have to be matched with statements (p, q, r, s) in 72. If α, β are roots of the equation ax2 + bx + c = 0, then the
column-II. quadratic equation whose roots are aα + b and aβ + b is
.............
64. Column-II give roots of quadratic equations given in
Column-I. 73. If r, s are roots of ax2 + bx + c = 0, then is ..........
Column-I Column-II
74. The quadratic equation whose roots are the sum and
(A) 6x2 + x – 12 = 0 (p) (– 6, 4) difference of the squares of roots of the equation
(B) 8x2 + 16x + 10 = 202 (q) (9, 36) x2 – 3x + 2 = 0 is....
x2 – 45x + 324 = 0
(C) (r) (3, –1/2) 75. If a, b are the roots of x2 + x + 1 = 0, then a2 + b2 = ...........
(D) 2x2 – 5x – 3 = 0 (s) (–3/2, 4/3)
76. If α, β are the roots of x2 + bx + c = 0 and α + h, β + h are
65. Column-I Column-II the roots of x2 + qx + r = 0, then h = ..........
(A) (x – 3) (x + 4) + 1 = 0 (p) Forth degree polynomial
77. A quadratic equation cannot have more than ........ roots.
(B) (x + 2)3 = 2x (x2 – 1) (q) Quadratic equation
78. Let ax2 + bx + c = 0, where a, b, c are real numbers,
(C) (2x – 2)2 = 4x2 (r) Non-quadratic equation
a ≠ 0, be a quadratic equation, then this equation has no
(D) (2x2 – 2)2 = 3 (s) Linear equation real roots if and only if ........... .
66. Column-I Column-II
(A) If α, β are roots of (p) a < 0, b > 0 True / False
ax2 + bx + c = 0,
then one of the roots DIRECTIONS : Read the following statements and write your
of the equation answer as true or false.
ax2 – bx (x –1)
+ c (x –1)2 = 0 79. A quadratic equation cannot be solved by the method of
completing the square.
(B) If the roots of (q) real and equal
ax2 + b = 0 are real, 80. If we can factorise ax2 + bx + c, a ≠ 0, into a product of
two linear factors, then the roots of the quadratic equation
then β ax2 + bx + c = 0 can be found by equating each factor to
(C) Roots of 4x2 – 4x + 1 = 0 (r)
1+ β zero.
(D) Roots of (s) Real 81. (x – 2) (x + 1) = (x – 1) (x + 3) is a quadratic equation.
(x – a) (x – b) + (x – b) 82 (x2 + 3x + 1) = (x – 2)2 is not a quadratic equation.
(x – c) + (x – c) (x – a) = 0
83. x2 + x – 306 = 0 represent quadratic equation where product
are always
of two consecutive positive integer is 306.

Fill in the Blanks 84. The roots of the equation (x – 3)2 = 3 are 3 ± 3
85. If sum of the roots is 2 and product is 5, then the quadratic
DIRECTIONS : Complete the following statements with an equation is x2 – 2x + 5 = 0
appropriate word/ term to be filled in the blank space(s).
86. Sum of the reciprocals of the roots of the equation
67. A quadratic equation in the variable x is of the form x2 + px + q = 0 is 1/p.
ax2 + bx + c = 0, where a, b, c are real numbers and a ......
87. The nature of roots of equation x 2 + 2 x 3 + 3 = 0 are real
68. A quadratic equation ax2 + bx + c = 0 has two distinct real
and equal.
roots, if b2 – 4ac ..............
69. The altitude of a right triangle is 7 cm less than its 88. For the expression ax2 + 7x + 2 to be quadratic, the possible
base. If the hypotenuse is 13 cm, the other two sides values of a are non-zero real numbers.
are ...............
Quadratic Equations M-81

ANSWER KEY & SOLUTIONS


11. (a) x2 – 0.04 = 0
1. (c) Given equation is x 2 − 3 x + (k − 10) = 0 .
⇒ x2 = 0.04
\ Product of roots = (k – 10).
⇒ x = ± 0.2
So, k – 10 = –2 ⇒ k = 8.
1 12. (d) Let a, b be the roots of the equation. Then a + b = 5
2. (d) Let the roots be α and . Then, and ab = –6. So, the equation is x2 – 5x – 6 = 0.
α
 1 The roots of the equation are 6 and –1.
product of roots =  α ×  = 1. b c
 α b
k 13. (b) α + 2α = − and α × 2α = ⇒ 3α = −
So, = 1 ⇒ k = 5. a a a
5 b 2 c  −b 
2
c
⇒ α = − and 2α = ⇒ 2   =
Required equation is x2 – 6x + 6 = 0.
3. (a) 3a a  3a  a
4. (a)
Since the roots are equal, we have D = 0. 2b 2 c
⇒ = ⇒ 2ab2 – 9a2c = 0 ⇒ a(2b2 – 9ac) = 0
2 2 a
∴ 4(k + 2) − 36k = 0 ⇒ (k +2)2 – 9k = 0 9a
Since a ≠ 0, ∴ 2b2 = 9ac
∴ k2 – 5k + 4 = 0 ⇒ k2 – 4k – k + 4 = 0
Hence, the required condition is 2b2 = 9ac
⇒ k(k – 4) – (k – 4) = 0
⇒ (k – 4)(k – 1) = 0 ⇒ k = 4 or k = 1. 14. (c) Use options or apply the formula

5. (d) The roots of 5x2 – kx + 1= 0 are real and distinct. a2 – b2 = (a – b) (a + b), we get x = 4

∴ (k2 – 4 × 5 × 1) > 0 ⇒ k2 > 20 15. (c) x2 + y2 = 25, xy = 12


2
⇒ k > 20 or k < – 20 ⇒ k > 2 5 or k < −2 5 .  12 
⇒ x 2 +   = 25 ⇒ x 4 + 144 − 25 x 2 = 0
c  x
6. (b)
(i) (a – b) (b – c) = product of the roots =
.
a ⇒ (x2 – 16) (x2 – 9) ⇒ x2 = 16 and x2 = 9
(ii) c – a = –(a – b + b – c) = – (sum of the roots) ⇒ x = ± 4 and x = ± 3
= b . 16. (a) We have, x = 7 + 4 3
a
x2 + 8x – 16 = 0
7. (d) 1 1 7−4 3
\ = = = 7−4 3
c x 7+4 3 7 + 4 3. 7 − 4 3
The product of the roots = = – 16.
a
1
8. (b) Use x2 – (α + β)x + αβ = 0 \ x + = 7+4 3 + 7−4 3
x
9. (b) Since the equation has two equal roots, D = 0 = ( 3 + 2) + (2 − 3) = 4
⇒ (2mc)2 – 4(1 + m2) (c2 – a2) = 0
17. (b) Equation px2 + 2qx + r = 0 and qx 2 − 2 pr x + q = 0
⇒ 4m2c2 – 4c2 + 4a2 – 4m2c2 + 4m2a2 = 0
have real roots then from first
⇒ –4c2 + 4a2 + 4m2a2 = 0 ⇒ 4c2 = 4a2 + 4m2a2
4q 2 − 4 pr ≥ 0 ⇒ q 2 ≥ pr ... (i)
⇒ 4c2 = 4a2 (1 + m2) ⇒ c2 = a2 (1 + m2)
10. (b) a2b2x2 + b2x – a2x – 1 = 0 and from second 4( pr ) − 4q 2 ≥ 0 (for real root)

⇒ b2x (a2x + 1) – 1(a2x + 1) = 0 ⇒ pr ≥ q 2 ... (ii)


⇒ (a2x + 1) (b2x – 1) = 0 From (i) and (ii), we get result q2 = pr
1 1 18. (d) The discriminant of a quadratic equation
⇒ x=− 2
,
a b2
ax2 + bx + c = 0 is given by b2 – 4ac.
M-82 Mathematics
a = 2, b = 2(p + 1) and c = p 26. (c) Given,
b2 – 4ac = [2(p + 1)]2 – 4(2p) = 4(p + 1)2 – 8p f(x) = x2 + ax + 2 and g(x) = x2 + 2x + a
= 4[(p + 1)2 – 2p] = 4[(p2 + 2p + 1) – 2p] = 4(p2 + 1) Let a be the common root of f(x) = 0 and g(x) = 0.

For any real value of p, 4(p2 + 1) will always be \ a2 + aa + 2 = 0 ...(i)


positive as p2 cannot be negative for real p. and a2 + 2a + a = 0 ...(ii)
Hence, the discriminant b2 – 4ac will always be positive. Using elimination method,
When the discriminant is greater than ‘0’ or is positive, a2 + aa = – 2
then the roots of a quadratic equation will be real. a2 + 2a = – a
and
19. (d) Let 1, 2 be the roots of equations (i), 2 and 4 be the – + +
roots of equation (ii). aa – 2a = – 2 + a
\ equations are x2 – 3x + 2 = 0 and x2 – 6x + 8 = 0. -2 + a
⇒ a(–2 + a) = –2 + a ⇒ a = =1
Comparing with x2 + bx + c = 0 and x2 + qx + r = 0, -2 + a
we get b = – 3, c = 2, q = – 6 and r = 8. Substitute value of a in (i) eqn., we get
Putting these values in the options, we find that option \ – (a + 2) = 1 ⇒ a + 2 = –1 ⇒ a = –3
(d) is satisfied.
Now f (x) + g(x) = 0
20. (d) The given equation is x2/3 + x1/3 – 2 = 0 \ x2 – 3x + 2 + x2 + 2x – 3 = 0 ⇒ 2x2 – x – 1 = 0
Put x1/3 = y, then y2 +y–2=0 1
So, sum of roots =
⇒ (y – 1) (y + 2) = 0 ⇒ y = 1 or y = – 2 2
⇒ x1/3 = 1 or x1/3 = – 2 27. (d) x2 – 6x – 2 = 0
∴ x = (1)3 or x = (–2)3 = – 8 a and b are the roots of the above equation.
Hence, the real roots of the given equations are 1, –8. So, a2 – 2 = 6a
21. (b) x (x + 1) + 1 = (x – 2) (x – 5) Similarly, b2 – 2 = 6b
⇒ x2 + x + 1 = x2 – 7x + 10 We can see that, a + b = 6 and ab = – 2
⇒ 8x – 9 = 0, which is not a quadratic equation. Given: an = an – bn

22. (d) If one root is α, then the other is


1
So,
10 10
a10 − 2a 8 α − β − 2 α − β
=
8 8
( )
1 c c
α 2a 9 2 α 9 − β9 ( )
∴ α. = product of roots = ⇒ 1= ⇒ a = c
α a a
=
α10 − β10 + αβ α8 − β8 ( )
23. (b) x +
1 10
=
x 3

x + 1 10
x
=
2

3
⇒ 3 x 2 − 10 x + 3 = 0 (
2 α 9 − β9 )
⇒ (x – 3) (3x – 1) = 0 ⇒ x = 3, x =
1
=
α10 − α 9β − αβ9 + β10 ( )
24. (b) ‘b2 = 4ac’
3
(
2 α −β 9 9
)


⇒ 4 (mp +
⇒ m2q2 +
nq)2
n2p2
=4 (m2
– 2mnpq = 0
+ n2) (p2 + q2)
α 9 ( α + β ) − β9 ( α − β ) (
( α + β ) α 9 − β9 )
⇒ (mq – np)2 = 0 ⇒ mq – np = 0.
=
(
2 α −β 9 9
) =
( 9
2 α −β 9
)
25. (a) α + β = b, αβ = c 6
= = 3 (Q a + b = 6)
Sum of roots of resulting equation = (α – 2) + (β – 2) 2
⇒ (α + β – 4 ) = b – 4 ; 28. (b) The given quadratic equation is, nx2 + 7 n x + n = 0
Product of roots resulting equation Now, the discriminant,
= (α – 2) (β – 2) = αβ – 2 (α + β ) + 4 D = 49n – 4n2 = n (49 – 4n)
= c – 2b + 4 D ≠ 0; \ ∀ n ∈ I+
Now, 2 = b – 4 ; 1 = c – 2b + 4 etc.
⇒ Roots of the quadratic equation are distinct.
Quadratic Equations M-83
For real roots D ≥ 0 18 – 3k (k + 6) – 5 (k + 6)2 = 0
49 18 – 3k2 – 18k – 5k2 – 180 – 60k = 0
⇒ n(49 – 4n) ≥ 0 ⇒ n ≤
4
–8k2 – 78k – 162 = 0
So, n ∈ {1, 2, 3, 4, ..., 12}
8k2 + 78k + 162 = 0
So, x have only finite value. 4k2 + 39x + 81 = 0
n 4k2 + 27k + 12k + 81 = 0
Product of roots = =1
n
k(4k + 27) + 3 (4k + 27) = 0
⇒ Products of root is necessarily integer.
(k + 3) (4k + 27) = 0
Hence, option (b) is correct.
27
29. (b) Let a, b be the roots of x2 – bx + 6 = 0 and a, g be the k = −3, −
4
roots of x2 – 6x + c = 0
3+ 5
x2 – bx + 6 = 0; α + β = b, α + γ = 6 31. (a) x =
2
x2 – 6x + c = 0; αβ = 6, αγ = c 3
3 Ê3+ 5ˆ 27 + 5 5 + 9 5(3 + 5)
β 3 fi x = Á ˜ = = 9+4 5
Given, = Ë 2 ¯ 8
γ 4
αβ 6 ⇒ y = x3 = 9 + 4 5
=
αγ c ⇒ One root is 9 + 4 5 \ other is root 9 – 4 5
β 6 3 6
= ⇒ = ∴ c=8 \ Sum of roots = 9 + 4 5 + 9 – 4 5 = 18
γ c 4 c
Product of roots = (9 + 4 5 ) (9 – 4 5 ) = 1
ab = 6; ag = 6
\ Required equation is: y2 – 18y + 1 = 0
HCF (α β, α γ ) = α
32. (c) D > 0 (for real roots)
HCF (6, 8) = 2
1 b3 − 8
a=2 ⇒ b2 − 4 × 2 × > 0 ⇒ >0
b b
30. (c) Let the common root be t 1.5
(b − 2)(b 2 + 2b + 4) 0 3
Then, the equation becomes ⇒ >0
b
2t2 + kt – 5 = 0 ...(i)
⇒ b ∈ ( −∞, 0) ∪ (2, ∞)
t2 – 3t – 4 = 0 ...(ii)
Clearly options A and B are wrong
Multiply equation (ii) by 2 and then subtract from
Let f(b) = b2 – 3b
equation (i)
range of f (b) when b ∈ (–∞, 0) ∪ (2, ∞)
2t2 + kt – 5 = 0
= (f(2), ∞) = (–2, ∞)
2t2 – 6t – 8 = 0
⇒ b2 – 3b > – 2 is correct
– + +
So, b ∈ (–∞, 0) ∪ (2, ∞) is subset of solution set of
(k + 6)t + 3 = 0 b2 – 3b + 2 > 0
3 1
t= −
Also D is wrong as b 2 + 2 ∈ (0, ∞)
k +6 b
Now, put the value of t in equation (i) 33. (c) (i) For equal roots,

 −3   −3 
2 D = 0 ⇒ b2 – 4ac = 0
2  +k −5 = 0
 k + 6   k + 6  16 – 8 (a + 5) = 0
18 −3 a+5=2
2
+ −5 = 0
k+6 a = –3
(k + 6)
M-84 Mathematics
(ii) For distinct real roots, For equal roots, discriminant, D = 0
D > 0 Þ b2 – 4ac > 0 ⇒ b2 – 4ac = 0 ⇒ ( − k )2 − 4(2) (k ) = 0
a2 + 12b(a + 4) > 0 ⇒ k2 – 8k = 0 ⇒ k = 0, 8
9 + 12b > 0
40. (c) Let α + 3 = x ∴ α = x – 3 (replace x by x – 3)
9
b>– So the required equation
12
(x – 3)2 – 5 (x –3) + 6 = 0
3
b> − ⇒ x2 – 6x + 9 – 5x + 15 + 6 = 0 ⇒ x2 – 11x + 30 = 0
4
(x2 – 11x + 30) × 2 = 0 ⇒ 2x2 – 22x + 60 = 0
34. (d)
The given quadratic equation is
x2 + (3 – l) x + 2 = l
41. (b) 42. (a)

Sum of roots = a + b = l – 3 43. (b) Let the roots be α and β.


Product of roots = ab = 2 – l α + β = 8 , |α – β| = 10
a2 + b2 = (a + b)2 – 2ab
(α – β)2 = 100 ⇒ (α + β)2 – 4αβ = 100 ⇒ αβ = –9
= (l – 3)2 – 2 (2– l) ∴ x2 – 8x – 9 = 0 ⇒ (x2 – 8x – 9) = 0
= l2 – 4l + 5 or – (– x2 + 8x + 9) = 0
= (l – 2)2 + 1 44. (d) Since, α and β are roots of x2 + 5x + a = 0
For least (a2 + b2), l = 2. ∴ α + β = – 5 and αβ = a
35. (a) ∵ α + β = 64, αβ = 256 Consider 2α + 5β = – 1
⇒ 2α + 5 (– 5 – α) = –1 ⇒ 2α – 25 – 5α = – 1
α3/8 β3/8 α+β 64 64
5/8
+ 5/8 = 5/8
= 8 5/8 = =2 ⇒ – 3α = 24 ⇒ α = – 8 and β = – 5 + 8 = 3
β α (αβ ) (2 ) 32

Also, αβ = (3) (– 8) = – 24 = a
36. (c) ( x + 2) ( x − 1) = x 2 − 2 x − 3
45. (c) Let α and β be the roots of x2 + px + 8 = 0
⇒ x2 − x + 2 x − 2 = x2 − 2 x − 3
⇒ α + β = – p and αβ = 8
−1
⇒ x − 2 = −2 x − 3 ⇒ 3 x = −1 ⇒ x = ⇒ α(– p – α) = 8 ⇒ – pα – α2 = 8
3
2
37. (b) 4 x − 3 x − 5 = 0 ⇒ α2 + pα + 8 = 0 ... (i)
3 5 Also, given α – β = 2
⇒ x2 − x− = 0
4 4 ∴ α + β = – p and α – β = 2 together gives
2 2 2α = 2 – p ⇒ p = 2 – 2α
3  3  3 5
⇒ x 2 − 2. .x +   −   − = 0 Put value of ‘p’ in equation (i), we get
8 8
  8
  4
α2 + (2 – 2α) α + 8 = 0 ⇒ α2 + 2α – 2α2 + 8 = 0
2 2
 3 3 5  3  83
⇒  x −  − − = 0 ⇒  x −  − =0 ⇒ – α2 + 2α + 8 = 0
 8  64 4  8  64
On solving this, we get, α = – 2, 4
Hence, the required constant that should be added and ∴ p = ± 6 (when α = – 2, 4)
3
subtracted is . 46. (b) Let the roots of the given equation be α and 3α.
64
Now, α + 3α = – p and α(3α) = 12
38. (c) 2 x 2 − 7 x + 6 = 2(2)2 − 7(2) + 6
⇒ 4α = – p and α2 = 4 ⇒ α = ± 2
= 8 − 14 + 6 = 14 − 14 = 0
Now, 4(2) = – p and 4(–2) = – p ⇒ p = ± 8
39. (d) Compare the quadratic equation 2x2 – kx + k = 0 with
the standard form of quadratic equation 47. (c) Given equation is
ax2 + bx + c = 0; a = 2, b = –k, c = k x2 – 3x + 2 = 0
Quadratic Equations M-85
⇒ x2 – 2x – x + 2 = 0 ⇒ x(x – 2) – 1 (x – 2) = 0 Assertion : (2x – 1)2 – 4x2 + 5 = 0 ⇒ – 4x + 6 = 0
⇒ (x – 1) (x – 2) = 0 ⇒ x = 1, 2 Reason : 2x2 – 6x = 0 ⇒ 2x(x – 3) = 0
48. (c) Since x = 2 and x = 3 are roots of given equation ⇒ x = 0 and x = 3.

∴ 3(2)2 – 2p(2) + 2q = 0 60. (a) Assertion : 9x2 + 3k x + 4 = 0


⇒ 12 – 4p + 2q = 0 ⇒ – 2p + q = – 6 ... (i) ⇒ D = b2 – 4ac = (3k)2 – 4(9) (4) = 9k2 – 144
and 3(3)2 – 2p(3) + 2q = 0 For equal roots : D = 0 ⇒ 9k2 = 144

⇒ 27 – 6p + 2q = 0 ⇒ – 6p + 2q = – 27 ... (ii) 12
⇒ k = ± ⇒ k=±4
3
On solving (i) and (ii), we get
61. (c) Reason is false.
15
p = and q = – 6 + 15 = 9
2 Assertion : 4x2 – 12x + 9 = 0
49. (a) Speed of Ajay's car = (x + 5) km/h. ⇒ D = b2 – 4ac = (– 12)2 – 4(4) (9) = 144 – 144 = 0
⇒ Roots are repeated.
Distance = Speed × Time = 2(x + 5) km
400 400 62. (c) Assertion is correct. Reason is incorrect.
50. (c) – =4
x x+5 63. (a) Assertion and Reason both are correct and Reason is
⇒ x2 + 5x – 500 = 0 correct explanation.
51. (a) (x + 25)(x – 20) = 0 Assertion : 2x2– 3x + 5 = 0
⇒ x = 20 km/hour – b – (–3) 3 c 5
⇒ α+β = = = and αβ = =
52. (d) Speed of Ajay = x + 5 = 25 km/h. a 2 2 a 2
400 64. (A) → s; (B) → p; (C) → q; (D) → r
Time = = 16 hours.
25 (A) 6x2 + x – 12 = 0
53. (c) (20 – x) km/hr 6x2 + 9x – 8x – 12 = 0
54. (a) Speed = Distance/Time 3x (2x + 3) – 4 (2x + 3) = 0
(3x – 4)(2x + 3x) = 0
15 15
55. (c) – =1 4 −3
20 – x 20 + x x= ,
⇒ x2 + 30x – 400 = 0 3 2
(B) 8x2 + 16x – 192 = 0
56. (b) (x – 10)(x + 40) = 0
8x2 + 48x – 32x – 192 = 0
⇒ x = 10 km/hour 8x (x + 6) – 32(x + 6) = 0
57. (c) Speed in downstream = 20 + 10 = 30 km/h x = 4, – 6
Distance 15 (C) x2 – 45x + 324 = 0
Time = = = 30 minutes
Speed 30 x2 – 36x – 9x + 324 = 0
58. (b) Assertion : Given equation x2 – bx + c = 0 x (x – 36) – 9 (x – 36) = 0
2x (x – 3) + 1 (x – 3) = 0
Let α, β be two consecutive roots such that | α – β | = 1
x = 9, 36.
⇒ (α + β)2 – 4αβ = 1 ⇒ b2 – 4c = 1
(D) 2x2 – 5x – 3 = 0
Reason : Given equation : 2x2 – 6x + x – 3 = 0
4abc x2 + (b2 – 4ac) x – b = 0 −1
x = ,3
D= (b2 – 4ac)2 + 16ab2c 2
D= (b2 + 4ac)2 >0 65. (A) → q; (B) → r; (C) → s; (D) → p
Hence, roots are real and unequal. 66. (A) → (r,); (B) → (p,); (C) → (q) ; (D) → (s)

59. (b) Assertion and Reason both are true statements. But 67. ≠ 0
Reason is not the correct explanation.
M-86 Mathematics
68. > 0 1
76. (b − q )
69. 5 cm, 12 cm. 2
70. b2 < 4ac 77. two 78. b2 < 4ac
71. 7 and –9 79. False 80. True
72. x2 – bx + ca = 0 81. False 82. True

b 2 − 2ac 83. True 84. True


73.
c2 85. True 86. False
87. True 88. True
74. x2 – 8x + 15 = 0
75. –1
10 Acids, Bases and
Arithmetic
Salts
Progressions
1 1− p 1− 2 p
Multiple Choice Questions (MCQs) 9. The common difference of the A.P. , , , ........
p p p
is
DIRECTIONS: This section contains multiple choice questions. 1 1
Each question has 4 choices (a), (b), (c) and (d) out of which (a) 1 (b) (c) –1 (d) −
p p
only one is correct.
10. The nth term of the A.P. a, 3a, 5a, ......., is
1. In an A.P. if a = 5, an = 81 and Sn = 860, then n is (a) na (b) (2n – 1)a
(a) 10 (b) 15 (c) 20 (d) 25 (c) (2n + 1)a (d) 2na
2.
What is the value of k if (k + 2), (4k – 6) and (3k – 2) are 11. If the sum of the series 2 + 5 + 8 + 11 ........... is 60100,
three consecutive terms of an A.P.? then the number of terms are
(a) k = –3 (b) k = 2 (c) k = –2 (d) k = 3 (a) 100 (b) 200
3. The first term of an A.P. is 5 and its 100th term is –292. (c) 150 (d) 250
The 50th term of this A.P. will be
12. What is the common difference of four terms in A.P. such
(a) 142 (b) –142 (c) 130 (d) –140
that the ratio of the product of the first and fourth term to
4.
If a, b, c are in A.P., then the value of (a + 2b – c) (2b + c – a) that of the second and third term is 2 : 3 and the sum of
(c + a – b) will be all four terms is 20 ?
(a) 4abc (b) 2abc (a) 3 (b) 1 (c) 4 (d) 2
(c) abc (d) None of these 13. There are 60 terms in an A.P. of which the first term is 8
5. Sum of n terms of the series and the last term is 185. The 31st term is
2 + 8 + 18 + 32 + .... is (a) 56 (b) 94 (c) 85 (d) 98

n ( n + 1) 14. There are four arithmetic means between 2 and –18. The
(a) (b) 2n( (n + 1) means are
2
(a) –4, –7, –10, –13 (b) 1, –4, –7, –10
n ( n + 1) (c) –2, –5, –9, –13 (d) –2, –6, –10, –14
(c) (d) 1
2 15. If the first, second and the last terms of an A.P. are a, b, c
respectively, then the sum is
6. If eight times the8th term of an A.P. is equal to 12 times
the 12th term of the A.P. then its 20th term will be (a + b) (a + c − 2b) (b + c) (a + b − 2c)
(a) (b)
(a) –1 (b) 1 (c) 0 (d) 2 2 (b − a ) 2 (b − a )

7. The 10th term of an AP is 20 and the 19th term is 101. (a + c) (b + c − 2a )


(c) (d) None of these
Then, the third term is 2 (b − a )
(a) – 43 (b) – 61 (c) – 52 (d) 1
16. The sum of 11 terms of an A.P. whose middle term is 30,
8. Given that the sum of the first ‘n’ terms of an arithmetic is
progression is 2n2 + 3n, find the 12th term. (a) 320 (b) 330
(a) 72 (b) 36 (c) 625 (d) 56 (c) 340 (d) 350
M-88 Mathematics
17. The first and last term of an A.P. are a and  respectively. (nr – 1)nr n (nr + 1)
If S is the sum of all the terms of the A.P. and the common (c) (d)
2 2
2 − a 2 28. First term of an arithmetic progression is 2. If the sum of
difference is , then k is equal to
k − ( + a ) its first five terms is equal to one-fourth of the sum of the
(a) S (b) 2S next five terms, then the sum of its first 30 terms is
(c) 3S (d) None of these (a) 2670 (b) 2610 (c) –2520 (d) –2550
18. If four numbers in A.P. are such that their sum is 50 and 29. The odd natural numbers have been divided in groups as
the greatest number is 4 times the least, then the numbers (1, 3) ; (5,7, 9, 11) ; (13, 15, 17, 19, 21, 23), .....
are
(a) 5, 10, 15, 20 (b) 4, 10, 16, 22 Then the sum of numbers in the 10th group is
(c) 3, 7, 11, 15 (d) None of these (a) 4000 (b) 4003 (c) 4007 (d) 4008
30. Suppose the sum of the first m terms of an arithmetic
19. Let Tr be the rth term of an A.P. for r = 1, 2, 3, .... If
progression is n and the sum of its first n terms is m,
1
for some positive integers m, n we have, Tm = and where m ≠ n. Then, the sum of the first (m + n) terms of
1 n the arithmetic progression is
Tn = , then Tmn equals
m (a) 1 – mn (b) mn – 5
1 1 1 (c) – (m + n) (d) m+n
(a) (b) + (c) 1 (d) 0
mn m n 31. Let an, n ≥ 1, be an arithmetic progression with first term
2 and common difference 4. Let Mn be the average of the
20. If the sum of the first 2n terms of 2, 5, 8, ....... is equal to first n terms. 10
the sum of the first n terms of 57, 59, 61......., then n is Then the sum is  M n
equal to n =1

(a) 10 (b) 12 (c) 11 (d) 13 (a) 110 (b) 335 (c) 770 (d) 1100
21. The number of terms of the series 5, 7, 9, .... that must be 32. Which of the following represents an A.P. ?
taken in order to have the sum 1020 is (a) 0.2, 0.4, 0.6, .... (b) 29, 58, 116....
(a) 20 (b) 30 (c) 40 (d) 50 (c) 15, 45, 135, 405... (d) 3, 3.5, 4.5, 8.5 ....
22. If the nth term of an A.P. is 4n + 1, then the common 33. If tn = 6n + 5, then tn+1 =
difference is : (a) 6(n + 1) + 17 (b) 6(n – 1) + 11
(a) 3 (b) 4 (c) 5 (d) 6 (c) 6n + 11 (d) 6n – 11
23. If a, b, c, d, e, f are in A.P., then e – c is equal to: 34. Summation of n terms of an A.P. is
(a) 2(c – a) (b) 2(d – c) n n
(c) 2(f – d) (d) (d – c)
(a) ( 2a + l ) (b)  2a + ( n − 1) d 
2 2
24. The number of common terms of the two sequences
17, 21, 25, ....., 417 and 16, 21, 26, ........, 466 is
a rn −1
(c)
(
) (d)
(
a 1− rn )
(a) 19 (b) 20 (c) 21 (d) 91
( r − 1) (1 − r )
25. The number of two digit numbers which are divisible by 35. Sn = 54 + 51 + 48 + ........ n terms = 513. Least value of n is
3 is (a) 18 (b) 19
(a) 33 (b) 31 (c) 30 (d) 29 (c) 15 (d) None of these
26. If the nth term of an A.P. is given by an = 5n – 3, then the 36. If the nth term of an A.P. be (2n – 1), then the sum of its
sum of first 10 terms is first n terms will be
(a) 225 (b) 245 (c) 255 (d) 270 (a) n2 – 1 (b) (n – 1)2
2
(c) (n – 1) – (2n – 1) (d) n2
27. If S1, S2, S3, ......., Sr are the sum of first n terms of r
arithmetic progressions respectively. Whose first terms b+c−a c+a−b a+b−c
37. If , , are in A.P., then which of
are 1, 2, 3, ......... and whose common differences are 1, a b c
3, 5, ........ respectively, then the value of S1 + S2 + S3 + the following is in A.P.?
...... Sr is (a) a, b, c (b) a2, b2, c2
(nr – 1 )(nr +1 ) (nr + 1)nr 1 1 1
(a) (b) (c) , , (d) a3, b3, c3
2 2 a b c
Arithmetic Progressions M-89
46. If nth term of an AP is given by
an = 2n + 3 then common difference of an AP is
(a) 2 (b) 3 (c) 5 (d) 1
DIRECTIONS : Study the given Case/Passage and answer 47. The value of x, for which 2x, x+ 10, 3x + 2 are three
the following questions. consecutive terms of an AP
Case/Passage-I (a) 6 (b) –6 (c) 18 (d) –18
India is competitive manufacturing location due to the low cost
of manpower and strong technical and engineering capabilities
contributing to higher quality production runs. The production of
Assertion & Reason
TV sets in a factory increases uniformly by a fixed number every
year. It produced 16000 sets in 6th year and 22600 in 9th year. DIRECTIONS : Each of these questions contains an Assertion
[From CBSE Question Bank 2021] followed by Reason. Read them carefully and answer the
question on the basis of following options. You have to select
the one that best describes the two statements.

(a) If both Assertion and Reason are correct and Reason is


the correct explanation of Assertion.
(b) If both Assertion and Reason are correct, but Reason is
not the correct explanation of Assertion.
(c) If Assertion is correct but Reason is incorrect.
(d) If Assertion is incorrect but Reason is correct.
Based on the above information, answer the following questions:
38. Find the production during first year. 48. Assertion : Let the positive numbers a, b, c be in A.P.,
39. Find the production during 8th year. 1 1 1
then , , are also in A.P.
40. Find the production during first 3 years. bc ac ab
41. In which year, the production is Rs 29,200. Reason : If each term of an A.P. is divided by abc, then
42. Find the difference of the production during 7th year and 4th
the resulting sequence is also in A.P.
year.
Case/Passage-II 49. Assertion : The sum of the series with the nth term,
Your friend Veer wants to participate in a 200m race. He can tn = (9 – 5n) is (465), when no. of terms n = 15.
currently run that distance in 51 seconds and with each day of Reason : Given series is in A.P. and sum of n terms of an
practice it takes him 2 seconds less.He wants to do in 31 seconds . n
[From CBSE Question Bank 2021] A.P. is Sn =  2a + ( n − 1) d  .
2
50. Assertion : Sum of first 10 terms of the arithmetic
progression – 0.5, – 1.0, – 1.5, ............................ is 27.5.
Reason : Sum of n terms of an A.P. is given as
n
Sn = [2a + (n − 1)d ] where a = first term, d = common
2
difference.
51. Assertion : Sum of first hundred even natural numbers
divisible by 5 is 500.
Reason : Sum of first n-terms of an A.P. is given by
43. Which of the following terms are in AP for the given n
Sn = [a + ] where  = last term.
situation 2
(a) 51,53,55…. (b) 51, 49, 47…. 52. Assertion : If nth term of an A.P. is 7 – 4n, then its common
(c) –51, –53, –55…. (d) 51, 55, 59… difference is –4.
44. What is the minimum number of days he needs to Reason : Common difference of an A.P. is given by
practice till his goal isachieved d = an + 1 – an.
(a) 10 (b) 12 (c) 11 (d) 9 53. Assertion : If Sn is the sum of the first n terms of an A.P.,
45. Which of the following term is not in the AP of the above then its nth term an is given by an = Sn – Sn – 1 .
given situation Reason : The 10th term of the A.P.
(a) 41 (b) 30 (c) 37 (d) 39 5, 8, 11, 14, ................... is 35.
M-90 Mathematics
54. Assertion : Arithmetic between 8 and 12 is 10. 61. Sum of 1 + 3 + 5 + .... + 1999 is ......... .
Reason : Arithmetic between two numbers ‘a’ and ‘b’ is 62. The sum of 8 A.Ms between 3 and 15 is ................... .
a+b
given as . 63. The sum of n terms of an A.P. is 4n2 – n. The common
2
difference = .................. .

Match the Following 64. The difference of corresponding terms of two A.P’s will
be .................... .

DIRECTIONS : Each question contains statements given in two 65. Sum of all the integers between 100 and 1000 which are
columns which have to be matched. Statements (A, B, C, D) in divisible by 7 is ................... .
column-I have to be matched with statements (p, q, r, s) in column-II.
True / False
55. Column -I Column-II
(A.P.) (Common Difference)
DIRECTIONS : Read the following statements and write your
3 5 answer as true or false.
(A) 1, , 2, ,....... (p) – 4
2 2
1 5 9 13 66. In an AP with first term a and common difference
(B) , , , ,....... (q) 0.2 d, the n th term (or the general term) is given by
3 3 3 3
(C) 1.8, 2.0, 2.2, 2.4 (r) 4/3 an = a + (n – 1)d.
(D) 0, – 4, –8, –12 (s) 1/2 67. If  is the last term of the finite AP, say the nth term, then
the sum of all terms of the AP is given by :
56. Column-I Column-II
(A.P.) (nth term) S = n (a + )
(A) 119, 136, 153, 170 ...... (p) 13 – 3n 2
68. The balance money ( in `) after paying 5% of the total
(B) 7, 11, 15, 19, ....... (q) 9 – 5n loan of ` 1000 every month is 950, 900, 850, 800, . . . 50.
(C) 4, –1, –6, –11, ...... (r) 3 + 4n represented A.P.
(D) 10, 7, 4, 3, ...... (s) 17n + 102 69. 2, 4, 8, 16, ............. is not an A.P.
70. 10th term of A.P. 2, 7, 12, ......... is 45.
Fill in the Blanks
71. 301 is a term of A.P. 5, 11, 17, 23, ............. .

DIRECTIONS : Complete the following statements with an 72. The general form of an A.P. is a, a + d, a + 2d, a + 3d,
appropriate word / term to be filled in the blank space(s). .............

57. 4, 10, 16, 22, ........, .......... . 73. In an Arithmetic progression, the first term is denoted by
58. 1, –1, –3, – 5, ....... , ......... . ‘a’ and ‘d ’ is called the common difference.
59. 11th term from last term of an A.P. 10, 7, 4......... , – 62, is 74. If an+1 – an = same for all ‘n’, then the given numbers
......... . form an A.P.
60. In a flower bed, there are 23 rose plants in the first row, 21 75. If Sn of A.P. is 3n2 + 2n, then the first term of A.P. is 3.
in the second, 19 in the third, and so on. There are 5 rose
plants in the last row. Number of rows in the flower bed
is ............... .
Arithmetic Progressions M-91

ANSWER KEY & SOLUTIONS


n ⇒ 8a + 56d = 12a + 132d
1. (c) Sn = (a + an )
2 ⇒ 8a – 12a + 56d – 132d = 0
n ⇒ –4a – 76d = 0
⇒ 860 = (5 + 81)
2 ⇒ a + 19d = 0 …(i)
n = 860 ÷ 43 = 20
∴ t20 = a + 19d = 0 using (i)
2. (d) (k + 2), (4k – 6) and (3k – 2) are in A.P.
∴ t20 = 0
⇒ 4k – 6 – k – 2 = 3k – 2 – 4k + 6
7. (a) Given that,
⇒ 3k – 8 = –k + 4 ⇒ 3k + k = 4 + 8
t10 = a + 9d = 20 ....(i)
12
⇒ 4k = 12 ⇒ k = =3 and t19 = a + 18d = 101 ....(ii)
4
By solving equations (i) and (ii), we get
3. (b) a = 5, t100 = –292
a = –61, d = 9
t100 = 5 + (100 – 1)d
t3 = a + 2d = –61 + 2 × 9 = – 43
[using tn = a + (n – 1)d]
8. (a) Sn = 2n2 + 3n
⇒ –292 = 5 + 99d
an = Sn – Sn – 1
⇒ –292 – 5 = 99d
a12 = S12 – S11
−297
⇒ d = ⇒ d = −3 = 2(12)2 + 3(12) – (2(11)2 + 3(11))
99
= 288 + 36 – (242 + 33) = 288 + 36 – 242 – 33
∴ t50 = 5 + (50 – 1) (–3) = 5 + (–147)
= 46 + 3 = 49 = 72
= 5 – 147 ⇒ t50 = –142
1− p 1 1− p −1 − p
4. (a) Let a, b, c are in A.P. 9. (c) d = − = = = –1
p p p p
∴ b – a = c – b ⇒ 2b = a + c
10. (b) an = a + (n – 1)d = a +(n – 1)2a
So the given expression becomes [Q d = 3a – a = 2a]
(a + a + c − c)(a + c + c − a )(2b − b) = a + 2an – 2a = 2an – a = (2n – 1)a
11. (b)
= (2a ) (2c) (b) = 4abc
12. (d) Take the four terms as a – 3x, a – x, a + x, a + 3x
5. (c) Here, a1 = 2, a 2 = 8 = 2 2 The sum = 4a = 20 ⇒ a = 5
∴ d = 2 2 − 2 = 2, a = 2 Also, 3 (a2 – (3x)2) = 2 (a2 – x2)

n ⇒ x = 1
Sn =  2a + ( n − 1) d 
2 However, the common difference is 2x and not x
n n ( n + 1) ∴ When x = 1, d = 2x = 2
= 2 × 2 + ( n − 1) 2  =
2 2 13. (d) Let d be the common difference

6. (c) t8 = a + 7d, t12 = a + 11d then 60th term = 8 + 59d = 185

According to question, 8t8 = 12t12 (given) ⇒ 59d = 177

⇒ 8(a + 7d) = 12(a + 11d) ⇒ d = 3


⇒ 31st term = 8 + 30 × 3 = 98.
M-92 Mathematics
14. (d) Let the means be X 1 , X 2 , X 3 , X 4 and the common 25. (c) Two digit numbers which are divisible by 3 are 12,
difference be b; then 2, X 1 , X 2 , X 3 , X 4 , –18 are 15, 18,..., 99;
in A.P.; So, 99 = 12+ (n – 1) × 3.
⇒ –18 = 2 + 5b 26. (b) Putting n = 1, 10, we get a = 2, l = 47.
⇒ 5b = – 20 10
\ S10 = ( 2 + 47 ) = 5 × 49 = 245.
⇒ b = – 4 2
Hence, X1 = 2 + b = 2 + (–4) = – 2; n
27. (b) S1 =
[2(1) + (n − 1)(1)]
2
X2 = 2 + 2b = 2 – 8 = –6 n
S2 = [2(2) + (n − 1)(3)]
X3 = 2 + 3b = 2 – 12 = –10;
 2
X4 = 2 + 4b = 2 – 16 = – 14 n
S3 = [2(3) + (n − 1)(5)]
The required means are –2, –6, –10, –14. 2
15. (c) ..................................................
..................................................
16. (b) ..................................................

n 2S n
17. (b) We have, S = ( a + ) ⇒ = n …(i) Sr = [2(r ) + (n − 1)(2r − 1)]
2 a+ 2
Adding S1, S2, S3, ......., Sr, we have
−a −a
Also,  = a + (n − 1) d ⇒ d = = n r (r + 1) r 
n −1 2S S1 + S2 + ... + Sr = (2) + (n − 1) [1 + 2r − 1]
−1 2 2 2 
a+
n
2 − a 2 =  r (r + 1) + (n − 1)r 2 
= \ k = 2S 2  
2 S − ( + a )
nr nr
= [ r + 1 + nr − r ] = [ nr + 1]
18. (a) 2 2

19. (c) 28. (d) a = 2, d = d,

2n n According to question,
20. (c) Given, {2.2 + (2n − 1)3} = {2.57 + (n − 1)2}
2 2 1
S5 = ( S10 – S5 ) , 4S5 = S10 – S5, 5S5 = S10
4
or 2 (6n + 1) = 112 + 2n
or 10n = 110 ∴ n = 11 5  10
5  {2 × 2 + (5 – 1)d } = [2 × 2 + (10 –1)d ]
21. (b)  2  2

22. (b)  n 
 Sn = ( 2a + (n – 1) d 
23. (b) Let x be the common difference of the A.P.  2 
a, b, c, d, e, f. 5 10
⇒ 5 × (4 + 4d ) [4 + 9d ]
∴ e = a + (5 – 1)x 2 2
[∵ an = a + (n – 1)d] ⇒ 20 + 20d = 8 + 18d
⇒ e = a + 4x ...(i) ⇒ d = – 6
and c = a + 2x ...(ii) 30 30
S30 = [2 × 2 + (30 – 1) (–6)] = [4 + 29 × (–6)]
∴ Using equations (i) and (ii), we get 2 2
e – c = a + 4x – a – 2x 30 –5100
= × (–170) = = – 2550
⇒ e – c = 2x = 2(d – c). 2 2
24. (b) Common terms will be 21, 41, 61, ........ 29. (a) Since, the general term of sum of odd natural
21 + (n – 1) 20 ≤ 417 number in the group is = n (2n)2 = 4n × n2 = 4n3
Hence, the required sum of numbers in the 10th group =
⇒ n ≤ 20.8 ⇒ n = 20
4 × 103 = 4000
Arithmetic Progressions M-93
30. (c) Given, Sm = n and Sn = m Dividing each term by (a + b + c),
m a+b+c a+b+c a+b+c
Sm = [2a + (m − 1)d ] = n ...(i) , , are in A.P.
2 a (a + b + c) b (a + b + c) c (a + b + c)
n 1 1 1
Sn = [2a + (n − 1)d ] = m ...(ii) , , are in A.P.
2 a b c

On subtracting Eq. (ii) from Eq. (i), we get 38. Given that
a6 = a + 5d = 16000 ..... (i)
(m − n ) d a9 = a + 8d = 22600 ..... (ii)
2a + [m(m − 1) − n(n − 1)] = n − m
2 2 – – –
d –3d = – 6600 ⇒ d = 2200
(m – n)a + (m – n)(m + n –1) = – (m – n) ⇒ a = 5000
2
\ Production during first year = 5000
⇒ 2a + (m + n –1) d = –2 [m ≠ n]
39. Production during 8th year is (a + 7d) = 5000 + 2(2200)
m+n
\ Sm + n = [(2a + (m + n − 1)d ]
2 = 20400
m+n 40. Production during first 3 year = 5000 + 7200 + 9400
= ( − 2) = −(m + n)
2
= 21600
31. (a) a1 = 2, d = 4
41. 5000 + (n – 1) 2200 = 29200 ⇒ n = 12th year
n
[ 2a1 + (n − 1)d ] 42. Difference = (a + 6d) – (a + 3d) = 3d = 6600
Mn = 2 = 2(n + 1) − 2 = 2n
n 43. (b)
10 10 44. (c) an = 51 – (n – 1)2 = 31 ⇒ n = 11
\ ∑ M n = 2 ∑ n = 110 45. (b) an = 51 – (n – 1)2 = 30 ⇒ n = 11.5 (not possible)
n =1 n =1
32. (a) Since there is a common difference option (a), 46. (a) d = a2 – a1 = [2(2) +3] – [2(1) + 3] = 2
d = 0.4 – 0.2 = 0.6 – 0.4 = 0.2 47. (a) 2(x + 10) = 2x + 3x + 2 ⇒ x = 6
33. (c) Put n + 1 in place of n in Tn = 6n + 5 48. (a) 49. (d)
34. (b)
n 50. (a) Both are correct. Reason is the correct reasoning for
35. (a) Sn = 513; [2(54) + (n – 1)(– 3)] = 513 assertion.
2
10
⇒ n(108 – 3n + 3) = 1026 Assertion : S10 = [ 2(−0.5) + (10 − 1) (−0.5)]
⇒ n2 – 37n + 342 = 0 2
⇒ n2 – 19n – 18n + 342 = 0 = 5 [−1 − 4.5] = 5(−5.5) = 27.5
⇒ n(n – 19) – 18(n – 19) = 0 51. (d) Assertion is incorrect.
⇒ (n – 18) (n – 19) = 0 ⇒ n = 18 or n = 19
Assertion : Even natural numbers divisible by 5 are
36. (d) t1 = 2(1) – 1 = 1
10, 20, 30, 40, ..........
t2 = 2(2) – 1 = 3, t3 = 2(3) – 1 = 5 and so on.
They form an A.P. with a = 10, d = 10
\ t1 + t2 + t3 +…+ tn = 1 + 3 + 5 +…[2(n) – 1]
100
n n
= [2 + (n – 1)2] = (2 + 2n – 2) = n2
S100 = [ 2(10) + 99(10)] = 50500
2
2 2
Reason is correct.
b+c−a c+a−b a+b−c 52. (a) Both are correct. Reason is the correct explanation.
37. (c) , , are in A.P.
a b c
Assertion : an = 7 – 4n
Adding 2 to each term
d = an + 1 – an= 7 – 4 (n + 1) – (7 – 4n)
b+c−a c+a−b a+b−c
+2, +2, + 2 are in A.P. = 7 – 4n – 4 – 7 + 4n = – 4.
a b c
a+b+c a+b+c a+b+c 53. (c) Assertion is correct. Reason is incorrect.
, , are in A.P.
a b c a10 = a + 9d = 5 + 9(3) = 5 + 27 = 32.
M-94 Mathematics
54. (a) 
Both are correct and Reason is the correct
  3 + 15  
explanation for the Assertion. 62. 72 8   etc.
  2  
55. (A) → (s); (B) → (r); (C) → (q); (D) → (p)
63. 11 [S2 = 4(2)2 – 2 ⇒ 14
3 1
(A) Common difference = d = −1 = S1 = 4(1)2 – 1 ⇒ 3 etc.]
2 2
5 1 4 64. another A.P.
d= − =
(B)
3 3 3 65. 70336 [Hint : S = 105 + 112 + ... 994 and 105 + (n – 1)7
d = 2 – 1.8 = 0.2
(C) = 994 ⇒ 105 + 7n – 7 = 994 ⇒ n = 128 etc.]
d = – 4 – 0 = – 4.
(D) 66. True
56. (A) → (s); (B) → (r); (C) → (q); (D) → (p) 67. True
13 – 3n = 13 – 3(1) = 10 68. True
9 – 5n = 9 – 5(1) = 4 69. True
3 + 4n = 3 + 4(1) = 7 70. False
17n + 102 = 17(1) + 102 = 119 71. False
57. 28, 34 72. True
58. –7, –9 73. True
59. –32 74. True
60. n = 10 75. False
First term = a = a1 = 3 (1)2 + 2(1) = 5
61. 1000 [ 2(1) + (1000 − 1)2]
2
11 Acids, Bases and
Circles
Salts
(a)
∠AQB = 70°, ∠AMB = 110°
Multiple Choice Questions (MCQs) (b)
∠AQB = 110°, ∠AMB = 70°
(c)
∠AQB = 100°, ∠AMB = 50°
DIRECTIONS : This section contains multiple choice
(d)
∠AQB = 60°∠AMB = 40°
questions. Each question has 4 choices (a), (b), (c) and (d) out
of which only one is correct. 5. A tangent CQ touches a circle
A
with centre O at P. Diameter AB
1. Two circles with centres O Q
is produced to meet the tangent O
R
and P, and radii 8 cm and at C. If ∠ACP = a° and
T
4 cm touch each other S ∠BPC = b°, the relation
B

externally. Find the length O P connecting a and b is


C
P
Q
of their common tangent
(a) a° + b° = 180° (b) a° + 2b° = 90°
QR.
(c) a° – b° = 60° (d) 2a° + b° = 100°
(a) 8 cm (b) 7 cm (c) 8 2 cm (d) 7 3 cm
6. PQ is a tangent drawn from a point P to a circle with
2. Two chords AB and CD
A
5 centre O and QOR is a diameter of the circle such that
of a circle intersect each B ∠POR = 120°, then ∠OPQ is
other at P outside the
3
(a) 60° (b) 45° (c) 30° (d) 90°
circle. If AB = 5 cm, C x D 2
P

BP = 3 cm and PD = 2 cm, 7. If a regular hexagon is inscribed in a circle of radius r,


find CD. then its perimeter is
(a) 4 cm (b) 5 cm (c) 8 cm (d) 10 cm (a) 3r (b) 6r (c) 9r (d) 12r

3. A ball is in the rest position 8. AB and CD are two chords of a circle intersecting at the
against a step PQ. If PQ = 10 point P outside the circle. If PA = 12 cm, CD = 7cm and
cm and QR = 15 cm, then find PC = 15 cm, then AB is equal to
diameter of the ball. (a) 15.5 cm (b) 4 cm
P
(a) 16 cm (c) 8 cm (d) 10 cm
(b) 32.5 cm Q R 9. In the figure below (not to scale), AB = CD and AB and
(c) 28 cm CD are produced to meet at the point p.
(d) 42 cm A

4. In the given figure, PA and A B


P
PB are two tangents to the Q

circle with centre O. If O D

∠APB = 40°, find ∠AQB M C


and ∠AMB. 40° If ∠BAC = 70°, then ∠P is
P
B
(a) 30° (b) 40° (c) 45° (d) 50°
M-96 Mathematics
10. In the adjoining figure, TP and TQ are the two tangents to (c) only parallel chords are of same length.
a circle with centre O. If ∠POQ = 110°, then ∠PTQ is (d) only perpendicular chords are of same length.
P
T
15. Number of tangents to a circle which are parallel to a
secant, is
O (a) 3 (b) 2 (c) 1 (d) infinite
Q
16. AB and CD are two common tangents to circles which
touch each other at a point C. If D lies on AB such that
(a) 60° (b) 70° (c) 80° (d) 90°
CD = 4 cm, then AB is
11. In the diagram below, if l and m are two tangents and AB (a) 12 cm (b) 8 cm (c) 4 cm (d) 6 cm
is a chord making an angle of 60° with the tangent l, then
17. In the figure, ∆APB is formed by three tangents to the
the angle between l and m is
m
circle with centre O. If ∠APB = 40°, then the measure of
∠BOA is

C B
R
B
O
60°
40°
l P
A P A T
(a) 50° (b) 55° (c) 60° (d) 70°
(a) 45° (b) 30° (c) 60° (d) 90°
18. Three circles with radii R1, R2 and r touch each other
12. In the diagram, O is the centre of the circle and D, E and F externally as shown in the adjoining figure. If PQ is their
are mid points of AB, BO and OA respectively. If ∠DEF common tangent and R1 > R2, then which of the following
= 30°, then ∠ACB is relations is correct?

C (a) R1 – R2 = r 
P
(b) R1 + R2 = 2r
R1 R
1 1 1 Q
O (c) + =
F E R1 R 2 r R2
1 1 1
A B (d) + =
D R1 R2 r
19. Two circles, both of radii a touch each other and each of them
(a) 30° (b) 60° (c) 90° (d) 120º touches internally a circle of radius 2a. Then the radius of the
circle which touches all the three circles is
13. In the below diagram, O is the centre of the circle, AC is
the diameter and if ∠APB = 120°, then ∠BQC is 1 2 3
(a) a (b) a (c) a (d) a
2 3 4
C 20. In the figure, O is the centre of the circle and OA = CD,
O
then ∠CPD is
Q
O
A  B
A
B
P
C D
(a) 30° (b) 150° (c) 90° (d) 120°

14. In two concentric circles, if chords are drawn in the outer
P

circle which touch the inner circle, then (a) 45° (b) 30° (c) 70° (d) 60°
(a) all chords are of different lengths. 21. In figure ABCD is a cyclic quadrilateral and ∠ADC = 80°,
(b) all chords are of same length. ∠ACD = 50°, then ∠CBD is:
Circles M-97
(b) In a circle, the perpendicular from the centre to a chord
C B bisects the chord.
(c) The point common to a circle and its tangent is called
the point of contact.
O (d) Adjacent angles of a cyclic quadrilateral are
D A supplementary.
29. Which of the following statement(s) is / are not correct ?
(a) 60° (b) 130° (c) 50° (d) 40° (a) The length of tangent from an external point P on
circle with centre O is always less than OP.
22. In the given figure, AC is the diameter of the circle.
(b) The tangent to the circumcircle of an isosceles triangle
ED || AC, ∠CBE = 65°, then ∠DEC is ABC at A, in which AB = AC, is parallel to BC.
B
(c) If angle between two tangents drawn from a point
P to a circle of radius ‘a’ and centre ‘O’is 90°, then
A C OP = a 2.
O
(d) None of these
E D 30. Which of the following statement(s) is/are correct?
(a) If a chord AB subtends an angle of 60° at the centre
(a) 35° (b) 25° (c) 65° (d) 30°
of a circle, then angle between the tangents at A and
23. Let ABCD be a square of side length 1, and G a circle passing B is also 60°.
through B and C, and touching AD. The radius of G is (b) The length of tangent from an external point on a circle
1 is always greater than the radius of the circle.
3 1 5
(a) (b) (c) (d) (c) If a number of circle touch a given line segment PQ at
8 2 2 8
a point A, then their centres lie on the perpendicular
24. Three circles of radii 1, 2 and 3 units respectively touch bisector of PQ.
each other externally in the plane. The circumradius of the (d) None of these
triangle formed by joining the centers of the circles is
31. Which of the following statement(s) is/are incorrect?
(a) 1.5 (b) 2 (c) 2.5 (d) 3
(a) Angle between the tangent line and the radius at the
25. Circles A, B and C are externally D point of contact is 90°.
tangent to each other and internally (b) A circle can have two parallel tangents atmost.
tangent to circle D. Circles A and B A (c) The distance between two parallel tangents drawn to
are congruent. Circle C has radius C a circle is equal to twice of radius.
1 unit and passes through the centre B (d) A line intersecting a circle in two points is called a chord.
of circle D. Then are radius of circle
32. A tangent PQ at a point P of a circle of radius 5 cm meets a
B is ________ units.
line through the centre O at a point Q so that OQ = 12 cm.
3 6 8 9
(a) (b) (c) (d) Length PQ is :
7 9 9 8
(a) 12 cm (b) 13 cm
26. The length of tangent drawn from a point Q to a circle
(c) 8.5 cm (d) 119 cm
is 24 cm and distance of Q from the centre of circle is
25 cm. The radius of circle is 33. In fig. if TP and TQ are the two tangents to a circle with
(a) 7 cm (b) 12 cm centre O so that ∠POQ = 110°, then ∠PTQ is equal to
(c) 15 cm (d) 24.5 cm T

27. Which of the following is a cyclic quadrilateral? P


(a) Rhombus (b) Rectangle
(c) Parallelogram (d) Trapezium 110°
Q
O
28. Which of the following is/are not correct?
(a) A secant is a line that intersects a circle in two distinct
points. (a) 60° (b) 70° (c) 80° (d) 90°
M-98 Mathematics
34. If tangents PA and PB from a point P to a circle with centre 39. Find ∠RSQ
O are inclined to each other at an angle of 80°, then ∠POA (a) 60 (b) 75
is equal to (c) 100 (d) 30
(a) 50° (b) 60° (c) 70° (d) 80°
40. Find ∠ORP
35. If angle between two radii of a circle is 130°, the angle (a) 90 (b) 70
between the tangents at the ends of the radii is :
(c) 100 (d) 60
(a) 90° (b) 50° (c) 70° (d) 40°
Case/Passage-II
36. In fig. the pair of tangents AP and AQ drawn from an Varun has been selected by his School to design logo for Sports
external point A to a circle with centre O are perpendicular Day T-shirts for students and staff . The logo design is as given
to each other and length of each tangent is 5 cm. in the figure and he is working on the fonts and different colours
according to the theme.
Then, the radius of the circle is
P
m
5c
A
O

In given figure, a circle with centre O is inscribed in a ΔABC,


such that it touches the sides AB, BC and CA at points D, E and
DIRECTIONS : Study the given Case/Passage and answer the
F respectively. The lengths of sides AB, BC and CA are 12 cm,
following questions.
8 cm and 10 cm respectively. [From CBSE Question Bank 2021]
Case/Passage-I C

A Ferris wheel (or a big wheel in the United Kingdom) is an F E


amusement ride consisting of a rotating upright wheel with
multiple passenger-carrying components (commonly referred
to as passenger cars, cabins, tubs, capsules, gondolas, or pods)
attached to the rim in such a way that as the wheel turns, they A D B

are kept upright, usually by gravity. 41. Find the length of AD


After taking a ride in Ferris wheel, Aarti came out from the (a) 7 (b) 8
crowd and was observing her friends who were enjoying the
ride . She was curious about the different angles and measures (c) 5 (d) 9
that the wheel will form. She forms the figure as given below. 42. Find the Length of BE
[From CBSE Question Bank 2021]
R (a) 8 (b) 5
S
(c) 2 (d) 9
S R 43. Find the length of CF
O (a) 9 (b) 5
30°
P (c) 2 (d) 3
Q 30°
P
Q 44. If radius of the circle is 4cm, Find the area of ∆OAB
37. In the given figure find ∠ROQ (a) 20 (b) 36
(a) 60 (b) 100 (c) 24 (d) 48
(c) 150 (d) 90 45. Find area of ∆ABC
38. Find ∠RQP (a) 50 (b) 60
(a) 75 (b) 60
(c) 100 (d) 90
(c) 30 (d) 90
Circles M-99
50. If two tangents PA and PB are drawn to a circle with
Assertion & Reason centre O from an external point P (figure), then match
the column.
DIRECTIONS : Each of these questions contains an Assertion
followed by Reason. Read them carefully and answer the
question on the basis of following options. You have to select A
the one that best describes the two statements.

(a) If both Assertion and Reason are correct and Reason is P  O


Y
the correct explanation of Assertion.
(b) If both Assertion and Reason are correct, but Reason is B
not the correct explanation of Assertion.
(c) If Assertion is correct but Reason is incorrect.
Column-I Column-II
(d) If Assertion is incorrect but Reason is correct.
(A) ∠ PAB (p) 90°
46. Assertion: If in a circle, the radius of the circle is 3 cm (B) ∠ OAP (q) q/2
and distance of a point from the centre of a circle is 5 cm, θ
then length of the tangent will be 4 cm. (C) ∠ OAB (r) 90° −
2
Reason:(hypotenuse)2 = (base)2 + (height)2 (D) ∠ AOB (s) 180° – θ

47. Assertion: If in a cyclic quadrilateral, one angle is 40°, 51. If an isosceles ∆ABC in which AB = AC = 6 cm is inscribed
then the opposite angle is 140° in a circle of radius 9 cm, then
A
Reason: Sum of opposite angles in a cyclic quadrilateral 6 cm 6 cm
P
is equal to 360° B C

48. Assertion: If length of a tangent from an external point 9 cm 9 cm


O
to a circle is 8 cm, then length of the other tangent from
the same point is 8 cm.
Reason: length of the tangents drawn from an external
Column-I Column-II
point to a circle are equal.
AP
(A) (p)
8 2
Match the Following CP
(B) (q) 4 2
OB
(C) (r) 2
DIRECTIONS : Each question contains statements given in
(D) Area of ∆ABC (s) 9
two columns which have to be matched. Statements (A, B, C, D)
in Column-I have to be matched with statements (p, q, r, s) in 52. A circle is inscribed in a A
Column-II. ∆ABC having sides
AB = 8 cm, BC = 10 cm F D
49. If AB is a chord of length 6 cm A and CA = 12 cm as shown
of a circle of radius 5cm, the in figure. Observe the
X O C B
tangents at A and B intersect Y diagram and match the E
at a point X (figure), then B
columns.
match the columns. Column-I Column-II
(A) AD (p) 15
Column-I Column-II
(A) AY (p) 4 cm (B) BE (q) 7 cm
(C) CF (r) 3 cm
(B) OY (q) 3.75 cm
(D) AD + BE + CF (s) 5 cm
(C) XA (r) 5 cm
(D) OA (s) 3 cm
M-100 Mathematics
53. Column-I Column-II
63. In given Fig., the length PB = .................. cm.
Definition Term
(A) A line segment which join (p) Secant
any two points on a circle. 5 cm
A
O
(B) A line which intersect the (q) Tangent
3 cm
circle in two points. P
(C) A line that intersects the circle (r) Chord B
at only one point.
True / False
Fill in the Blanks
DIRECTIONS : Read the following statements and write your
DIRECTIONS : Complete the following statements with an answer as true or false.
appropriate word / term to be filled in the blank space(s).
64. The tangent to a circle is a special case of the secant.
54. A tangent to a circle touches it at ............... point (s). 65. The perpendicular at the point of contact to the tangent to
a circle does not pass through the centre.
55. A line intersecting a circle at two points is called a ...........
66. A circle can have at the most two parallel tangents.
56. A circle can have .............. parallel tangents at the most.
67. If P is a point on a circle with centre C, then the line drawn
57. The common point of a tangent to a circle and the circle through P and perpendicular to CP is the tangent to the
is called .................. . circle at the point P.
58. There is no tangent to a circle passing through a point lying 68. The centre of the circle lies on the bisector of the angle
............ the circle. between the two tangents.

59. The tangent to a circle is .............. to the radius through 69. A tangent to a circle is a line that intersects the circle at
the point of contact. only one point.
70. Two equal chords of a circle are always parallel.
60. There are exactly two tangents to a circle passing through
a point lying ........... the circle. 71. A line drawn from the centre of a circle to a chord always
bisects it.
61. The lengths of the two tangents from an external point to
72. Line joining the centers of two intersecting circles always
a circle are ............. .
bisect their common chord.
62. The tangents drawn at the ends of a diameter of a circle 73. In a circle, two chords PQ and RS bisect each other. Then
are .................. . PRQS is a rectangle.
Circles M-101

ANSWER KEY & SOLUTIONS


1. (c) Join O to P and Q. Join P to R. Draw SP ⊥ OQ. 6. (c) Since, PQ is tangent from a point P
Now SP = QR, as they are opposite sides of rectangle PRQS.
OP = 8 cm + 4 cm = 12 cm; OS = 8 cm – 4 cm = 4 cm Q
In right triangle POS,
O
2 2 = 2 2
SP = OP − OS 12 − 4 = 8 2 cm
120°
∴ QR = 8 2 cm P
R
2. (d) Since, two chords AB and CD of the circle are
∴ ∠OQP = 90° ⇒ ∠POR = 120°
intersecting at P, when produced.
∴ ∠POR + ∠POQ = 180° [linear pair]
∴ PA. PB = PC.PD
∠POQ = 180° – 120° = 60°
[Each = (length of the tangent from P)2] In ∆OPQ
⇒ (AB + PB). (PB) = (PD + DC) ⋅ PD ⇒ ∠OPQ + ∠OQP +∠ POQ = 180°
⇒ (5 + 3) (3) = (2 + x) 2 [Q Sum of angles of ∆ = 180°]
⇒ 24 = (2 + x) (2) ⇒ 12 = 2 + x ∴ ∠OPQ + 90° + 60° = 180° ⇒ ∠OPQ = 30°
⇒ x = 10 ⇒ CD = 10 cm 7. (b) Side of the regular hexagon inscribed in a circle of
3. (b) In right ∆OSP,  radius r is also r, the perimeter is 6r.
OP2 = PS2 + OS2 r
O
8. (a)
r2 = 225 + (r – 10)2 P S
2 2
10 15 10 9. (b) Exterior angle of a cyclic quadrilateral is equal to
⇒ r = 225 + r – 20r + 100
Q 15 R its interior opposite angle.
⇒ 20 r = 325 ⇒ 2r = 32.5
Hence, diameter = 32.5 cm. ∠BAC = ∠DCA and proceed.
4. (a) Since, OA ⊥ PA and OB ⊥ PB, 10. (b) [Hint. OP ⊥ PT, OQ ⊥ QT.
∴ In quadrilateral AOBP, In quad. OPTQ, ∠POQ + ∠OPT + ∠PTQ + ∠OQT = 360°
⇒ 40° + 90° + 90° + ∠AOB = 360° ⇒ 110° + 90° + ∠PTQ + 90° = 360°
⇒ ∠AOB = 140°
⇒ ∠ PTQ = 70°]
1 1
Also, ∠AQB = of ∠AOB = 70° and ∠AMB = 11. (c) Tangents drawn to a circle from an external point
2 2
are equal.
1 1
of reflex ∠AOB = × (360° – 140°) = × 220 = 110° 12. (b) (i) ADEF is a parallelogram.
2 2
[Q The angle subtended by an arc at the centre is double (ii) ∠FAD = 30° and ∠OAD = ∠OBA
the angle subtended by the arc at any point on the (angles opposite to equal sides)
remaining part of the cirlce.]
13. (b) (i) APBC is a cyclic quadrilateral.
5. (b) Given, ∠BPC = b° and ∠ACP = a°.
Also, ∠OPA = ∠OAP = b° (Angles in an isosceles (ii) ∠ABC is an angle in a semi circle.
triangle OAP, angle in alternate segment.) (iii) ABQC is a cyclic quadrilateral.
∠CPO = 90° ∴ ∠CPA = 90° + b° 14. (b) All chords are of same length.
In ∆ ACP, ∠ACP = 180° – [(b° + 90°) + b°]
15. (b) Only two tangents are parallel to a secant.
⇒ a° + 2b° = 90°
M-102 Mathematics
16. (b) AD = CD A D B In ∆ABP,
and BD = CD C ∠APB = 180° − ( x + y ) = 60°
∴ AB = AD + BD = CD + CD 21. (c) In cyclic quadrilateral ABCD,
= 2CD = 2 × 4 = 8 cm ∠ADC = 80° and ∠ABC = 100°
17. (d) We redraw the figure. (Q opposite angles of a cyclic quadrilateral are
L supplementary)
B
y
y
O Now, ∠ACD = ∠DBA = 50°
xx
20° (angles in same segment of a circle are equal).
P 20°
A S \ ∠CBD = ∠ABC – ∠ABD = 50°
In ∆OPS, using Pythagoras theorem, ∠POS = 70° 22. (b) ∠ABC = ∠ABE + ∠CBE = 90°
and In ∆POL, ∠POL = 70° (angle in a semi circle is a right angle)
B
From figure, 2x + 2y = 140°
∠BOA = x + y = 70° 65°

18. (d) Here, PQ is the common tangent to the three circles A  C


2 2 O
So, PR = (R1 + r) − (R1 − r) = 4R1r ...(i)
E D
RQ = 4R 2 r ....(ii) PQ = 4R1R 2 ....(iii)
We know that PQ = PR + RQ ⇒ ∠ABE = 90° – 65° = 25°
⇒ 4R1R 2 = 4R1r + 4R 2 r (QFrom (i), (ii) and (iii)) ⇒ ∠ACE = ∠ABE = 25°
(∵ angles in same segment of a circle are equal)
1 1 1
⇒ R1R 2 = R1r + R 2 r ∴ = + ⇒ ∠ACE = ∠DEC = 25°
r R2 R1
[alternate interior angles are equal]
19. (b)
23. (d) ABCD is a square of sides
A AB = BC = CD = AD = 1 unit
r
2a
A circle G passing through B and C and touching AD at N,
a where BC is chord of circle.
C
D
a O a B a a
1
r

Since, in figure, DAOB is a right angled triangle r


2
N M
O 1–r
\ OA2 = AB2 + OB2
⇒ (a + r)2 = (2a – r)2 + a2 A B

⇒ a2 + 2ar + r2
+ = 4a2 r2 – 4ar + a2
\ OM bisects the chord BC
2 a
⇒ 6ar = 4a2 ⇒ r = 1 1
3 \ CM = MB = BC =  (∵ BC = 1)
2 2
20. (d) In the given figure, ODC is
⇒ OM = MN – ON = 1 – r
equilateral triangle
A x
O
B In DOMC, OC2 = OM2 + CM2
⇒ ∠ODC = ∠OCD = 60° y
2
Ê 1ˆ
⇒ r2 = (1 – r)2 + Á ˜
x y
Now, in quadrilateral ABCD C
60° 60°
Ë 2¯
D
⇒ x + x + 60° + y + y + 60° = 360°
1 5
⇒ x + y = 120° ⇒ r2 = 1 – 2r + r2 + ⇒r=
P 4 8
Circles M-103
24. (c) Since, radii of circles are 1, 2 and 3 units. 26. (a) Here, O is the centre of circle.
∴ Side of ∆ABC are AB = 5, BC = 3, AC = 4

C
1
1
2
3
B
3
2
Let r be the radius of circle r = (25) 2 − (24) 2
A
= 625 − 576 = 49
= 7 cm
∴ ∆ABC is formed a right angled triangle, where AB is
27. (b) 28. (d)
hypotenuse of triangle.
29. (d) All the three statements are correct.
Since, circumradius of a right angled triangle is the half
of the hypotenuse. 30. (d) All the three statements are false.
1 1 31. (d)
∴ Circumradius = × AB = × 5 = 2.5
2 2
32. (d) O is the centre of the circle. The radius of the circle
25. (c) In ∆MAN is 5 cm.
(2 – r)2 = x2 + r2
⇒ 4 + r2 – 4r = x2 + r2 O

4(1 – r) = x2 ⇒ 4 – 4r = x2 12 c m 5 cm
Q
2
4− x
⇒ r= P
4
PQ is tangent to the circle at P. Then,
OP = 5 cm and ∠OPQ = 90°.
r B
1
1M
r
x N We are given that OQ = 12 cm.
C
1 2–r r
r A
By Pythagoras Theorem, we have
PQ2 = OQ2 – OP2 = (12)2 – (5)2 = 144 – 25 = 119
⇒ PQ = 119 cm .
In ∆CAN, (1 + x)2 + r2 = (1 + r)2
33. (b) In figure, TPOQ is a quadrilateral.
1 + x2 + 2x + r2 = 1 + r2 = 2r
Here, ∠OPT = ∠OQT = 90° ⇒ ∠PTQ + ∠POQ = 180°
⇒ x2 +2x = 2r ⇒ x2 = 2r – 2x
⇒ ∠PTQ + ∠110° = 180° ⇒ ∠PTQ = 70°.
 4 − x2 
⇒ x2 = 2  − 2x 34. (a) In figure
 4 
P
2
4− x
⇒ x2 = − 2x
°
40 40°
2
⇒ 2x2 = 4 – x2 – 4x ⇒ 3x2 + 4x – 4 = 0
A
B

⇒ 3x2 + 6x – 2x – 4 = 0 ⇒ 3x(x + 2) – 2(x + 2) = 0 O

2
⇒ x = , x = −2.
3
∆OAP ≅ ∆OBP (SSS congruence)
2
 2 4
  4− ⇒ ∠POA = ∠POB = ∠AOB ... (i)
r = 4−
3
= 9 ⇒ 36 − 4 = 32 ⇒ 8 .
4 4 36 36 9
Also, ∠AOB + ∠APB = 180°
M-104 Mathematics
⇒ ∠AOB + 80° = 180° B

⇒ ∠AOB = 100° ... (ii) 3cm

Then, from (i) and (ii), A


5cm O

1
∠POA = × 100° = 50°.
2
47. (c) Angle + 40° = 180°
35. (b)
Angle = 180° – 40° = 140°.
36. (c)
48. (a)
37. (c) ∵ ∠ROQ + ∠RPQ = 180°
⇒ ∠ROQ = 180° − 30° = 150° 49. (A) → (s); (B) → (p); (C) → (q); (D) → (r)
38. (a) ∵ ∠RQP = ∠QRP 50. (A) → (r); (B) → (p); (C) → (q); (D) → (s)
and ∠RQP + ∠QRP + ∠QPR = 180° 51. (A) → (r); (B) → (q); (C) → (s); (D) → (p)
⇒ 2∠RQP = 180° − 30° ⇒ ∠RQP = 75° OP ⊥ BC, Let AP = x cm and PB = CP = y cm
1 1
39. (b) ∠RSQ = ∠ROQ = × 150° = 75° On applying Pythagoras in ∆APB and ∆OPB, we have
2 2
36 = y2 + x2 and 81 = (9 – x)2 + y2
40. (a) 90° [∵ OR ⊥ RP]
On solving these, we get, x = 2 cm and y = 4 2 cm.
Sol. (41-45)
C 1 1
Area of ∆ABC = (BC × AP) = × 8 2 × 2 = 8 2 cm2.
10-x 2 2
10-x
52. (A) → (s); (B) → (r); (C) → (q); (D) → (p)
F E
O AD = AF = x cm, BD = BE = y cm, CE = CF = z cm
x 12-x
(∵ tangents drawn from an exterior point to a circle are
B equal in length).
A x D 12-x
AB = 8 cm ⇒ AD + BD = 8 ⇒ x + y = 8  ... (i)
BC = 10 – x + 12 – x = 8 Similarly, BE + CE = 10
⇒ x = 7.
⇒ y + z = 10 ... (ii)
41. (a) AD = 7 cm
z + x = 12 
and ... (iii)
42. (b) BE = 12 – x = 12 – 7 = 5 cm
Adding equations (i) + (ii) + (iii),
43. (d) CF = 10 – x = 10 – 7 = 3 cm
⇒ x + y + z = 15  ... (iv)
1
44. (c) Ar DOAB = × AB × OD Thus, on solving (i), (ii), (iii) and (iv)
2
we get, AD = x cm = 5 cm
1
= × 12 × 4 = 24 cm 2 BE = y cm = 3 cm
2
45. (b) Ar DABC = Ar DAOB + Ar DOBC + Ar DAOC CF = z cm = 7 cm

1 1 53. (A) → (r); (B) → (p); (C) → (q)


= 24 + × 8 × 4 + × 10 × 4 = 60 cm 2
2 2 54. One
46. (a) (OA)2 = (AB)2 + (OB)2 55. Secant
AB = 25 − 9 = 4 cm. 56. Two
Both Assertion and Reason are correct. 57. Point of contact
Also, Reason is the correct explanation of 58. inside
the Assertion.
Circles M-105
59. perpendicular 64. True 65.  False 66.  True 67. True
60. outside 68. True 69.  True 70.  False 71. False
61. equal 72. True 73. True
62. Parallel
63. 4 cm
PB = AP = 52 − 32 (Q OP ^ AB)

= 25 − 9 = 4 cm
12 Acids, Bases and
Constructions
Salts
equal distance points are marked on the ray AX such that
Multiple Choice Questions (MCQs) the minimum number of these point is
(a) 2 (b) 5
DIRECTIONS : This section contains multiple choice (c) 4 (d) 7
questions. Each question has 4 choices (a), (b), (c) and (d) out 6. The sides of a triangle (in cm) are given below. In which
of which only one is correct. case, the construction of triangle is not possible.
(a) 8, 7, 3 (b) 8, 6, 4
1. To draw tangent from an exterior point ‘P’ to a circle using
(c) 8, 4, 4 (d) 7, 6, 5
the centre ‘O’ of the circle, first we
7. Given a triangle with side AB = 8 cm. To get a line segment
(a) join P to O.
3
(b) draw tangent from P to the circle. AB' = of AB, it is required to divide the line segment
4
(c) draw secant from P which intersects the circle at two
points. AB in the ratio
(d) draw perpendicular at P to PO. (a) 3 : 4 (b) 4 : 3
(c) 1 : 3 (d) 3 : 1
2. To divide a line segment AB of any length in the ratio
8. To draw a pair of tangents to a circle which are inclined
2 : 5, first draw a ray AX (or BX) making an acute angle
to each other at an angle of 70°, it is required to draw
with AB. Then, draw 7 arcs intersecting the ray at X1,
tangents at end points of those two radii of the circle, the
X2, X3, X4, X5, X6 and X7 such that AX1 = X1X2 =
angle between them should be
X2X3 = X3X4 = X4X5 = X5X6 = X6X7. Then join X7B,
(a) 110° (b) 20°
then after
(c) 90° (d) 120°
(a) join X2B
9.
To divide a line segment AB in the ratio p : q (p, q are
(b) draw X2B′ parallel to X7 B, which intersects AB at B′.
positive integers), draw a ray AX so that ∠BAX is an acute
(c) join X5B angle and then mark points on ray AX at equal distances
(d) draw perpendicular from X2 to AB. such that the minimum number of the points is
3. To draw a pair of tangents to a circle which are inclined to (a) greater of p and q (b) p+q
each other at an angle of 45° it is required to draw tangents (c) p + q – 1 (d) pq
at end points of those two radii of the circle, the angle 10. Draw a pair of tangents to a circle which are inclined to
between them should be- each other at an angle of 35°, it is required to draw tangents
(a) 60° (b) 90° (c) 45° (d) 135° at the end points of those two radii of the circle, the angle
4. Which is true? between which is
In order to divide a line segment AB = 6 cm, in the radio (a) 105° (b) 70°
3 : 5, we draw a ray AX making an acute angle ∠BAX. (c) 140° (d) 145°
Along AX mark off.... points A1, A2, A3 etc. such that 11. To divide a line segment AB in the ratio 4 : 7, a ray AX
AA1 = A1A2 = etc., The number of points is is drawn first such that ∠BAX is an acute angle and then
(a) 3 (b) 5 (c) 8 (d) 2 points A1, A2, A3, ..... are located at equal distance on the
ray AX and the point B is joined to
5. To divide a line segment AB in the ratio 2 : 5, first a ray
(a) A12 (b) A11 (c) A10 (d) A9
AX is drawn, so that ∠BAX is an acute angle and then at
Constructions M-107

Match the Following

DIRECTIONS : Given below question contains statements given in two columns which have to be matched. Statements
(A, B, C, D,....) in Column-I have to be matched with statements (p, q, r, s,....) in Column-II.

12. To draw tangents from an exterior point P to a circle, drawn using a bangle, each step of continue diagram in random order is
given in column I and how to draw each steps of continue diagram is written in different random order in column II match
the items in two columns.
     Column-I       Column-II

(A)      (p) Draw a circle using a bangle

(B) (q) Draw perpendicular bisector of RB, which intersects RB


at O.
P A B

(C) (r) Draw a secant PAB intersecting the circle at A and B.

O A
R B
P
T

(D) (s) Produce AP to R, such that PA = PR.

O A
R B
P

(E) O A (t) Draw PQ perpendicular to RB at P which intersects the


R B
P semi-circle drawn with centre O and radius OR at Q.
M-108 Mathematics
Q

(F) (u) Draw ray PS and PT.

O A
R B
P
T

(G) (v) With centre O and radius OB (or OR), draw a semi-circle.
A
R B
P

(H) (w) With centre P and radius PQ draw arcs intersecting the
circle drawn using the bangle at S and T.
O A
R B
P

Fill in the Blanks True / False

DIRECTIONS : Complete the following statements with an DIRECTIONS : Read the following statements and write your
appropriate word / term to be filled in the blank space(s). answer as true or false.

13. Two circles are called concentric if both have ............ 16. We can not draw the tangent to a circle at a given point on
centre and ............ radii. it, when the centre of the circle is not known.
14. To construct a triangle we must know atleast its ............ 17. We can draw the tangents to a circle from a point outside
parts. it (external point), only when its centre is known.
15. To construct the tangents to a circle from a external point
18. Construction of a triangle is not possible if AB + BC < AC.
if distance of point from centre of circle is ............ radius
of circle. 19. To draw the perpendicular bisector of line segment PQ,
1
we open the compass more than PQ.
2
Constructions M-109

ANSWER KEY & SOLUTIONS


1. (a) Join P to O 6. (c) We know that, in a triangle sum of two sides of
2. (b) draw X2B′ parallel to X7 B, which intersects AB at triangle is greater than the third side. Here, the sides
B′. of triangle given in option (c) does not satisfy this
condition. So, with these sides the construction of a
3. (d) Here PA and PB are two tangents inclined at an angle
45º triangle is not possible.
Then, x + 90º + 90° + 45º = 360º 7. (d) 8. (a) 9. (b)
x + 225º = 360º
x = 360° – 225° 10. (d) 11. (b)
x = 135º
12. (A) → (p); (B) → (r); (C) → (u); (D) → (t); (E) → (q);
A
(F) → (w); (G) → (s); (H) → (v)
13. Same, different
O x 45º P
14. Three
15. More than
B
16. False
4. (c) 17. False
5. (d) We know that, to divide a line segment AB in the 18. True
ratio m : n, first draw a ray AX which makes an acute
19. True
∠BAX then, marked m + n points at equal distance.
Here, m = 2, n = 5
∴  Minimum number of these points = 2 + 5 = 7
Some
13 Acids, Bases
Applications
Salts
Trigonometry
and
of

4. A 15m long ladder placed vertically along a wall broke


Multiple Choice Questions (MCQs) in such a way that its top touches the ground making an
angle of 60° with it. At what height from the ground did
DIRECTIONS : This section contains multiple choice the ladder break?
questions. Each question has 4 choices (a), (b), (c) and (d) out (a) 7.34 m (b) 8 m
of which only one is correct.
(c) 9.46 m (d) 6.96 m
1. From a point on the ground, the angles of elevation of the 5. A tree 6 m tall casts a 4 cm long shadow. At the same time,
bottom and the top of a transmission tower fixed at the top a flag pole casts a shadow 50 m long. How long is the flag
pole?
of a 20 m high building are 45° and 60° respectively. Find
the height of the tower. (a) 75 m (b) 100 m
(c) 150 m (d) 50 m
(a) 
20 3 –1 m (b)
20 3m 6. The height of a tower is h and the angle of elevation of the
(c) 20 m (d) 40 m top of the twoer is α. On moving a distance h/2 towards
the tower, the angle of elevation becomes β. What is the
tan θ
2. From the figure, find the value of and it is given value of cotα – cotβ?
tan φ
that ‘D’ be the mid point of BC. 1 2
(a) (b)
A 2 3
(c) 1 (d) 2
 7. A bridge across a river makes an angle of 45° with the

river bank. If the length of the bridge across the river is
150 m, width of the river is
(a)
75 2m (b) 150 m

75
B D C (c) m (d) 75 m
2
3 1 8. A professor standing on one end of a football field observes
(a) (b)
4 2 the elevation of the top of a flood light tower at an angle
1 4 of α. He then walks a distance equal to twice the height
(c) (d) of the tower and finds that the elevation of the top is now
2 7
at an angle of 90°– α. What is the value of tanα ?
3. The angles of elevation of the top of a tower, as seen
from two points A and B situated in the same line and at 1 − 2
(a) (b) 2 −1
distance x and y respectively. from the foot of the tower,
(c) 2 (d) 1
are complementary. Find the height of the tower.
(a) x + y (b) xy 9. An electrician has a repair an electric fault on a pole of
height 5 cm. He has to reach a point 1.3 m below the top
xy
(c) (d) x+ y of the pole to undertake the repair work. What should
Some Applications of Trigonometry M-111
be the length of the ladder that he should use which, 16. In the given figure, the positions of the observer and the
when inclined at an angle of 60° to the horizontal, would object are mentioned, the angle of depression is
enable him to reach the required position? Also, how Observer
far from the foot of the pole should he place the foot of
the ladder?
[Take 3 = 1.73].
(a) 4.28 m, 2.14 m
(b) 4.28 m, 2.83 m
(c) 4.6 7m, 2.14 m Object

(d) 3.67 m, 2.14 m (a) 30° (b) 90°


10. An aeroplane when flying at a height of 3125 m from (c) 60° (d) 45°
the ground passes vertically below another plane at an 17. A person walks towards a tower. Initially when he starts,
instant when the angles of elevation of the two planes angle of elevation of the top of tower is 30°. On travelling
from the same point on the ground are 30° and 60° 20 metres towards the tower, the angle changes to 60°. How
respectively.Find the distance between the two planes much more has he to travel to reach the tower?
at that instant.
(a)
10 3 metres (b) 10 metres
(a) 3920 m (b) 6250 m
10
(c) 5125 m (d) 4444 m (c) 20 metres (d) metres
3
11. In the adjoining figure, the length of BC is
18. ABC is a field in the form of an equilateral triangle. Two
(a)
2 3 cm
C vertical poles of heights 45m and 20m are erected at A
m and B respectively. The angles of elevation of the tops of
(b)
3 3 cm 6c the two poles from C are complementary to each other.
There is a point D on AB such that from it, the angles of
(c)
4 3 cm elevation of the tops of the two poles are equal.Then AD
30°
(d) 3 cm A B is equal to –
5 10
12. If the angle of depression of an object from a 75 m high (a)
17 m (b)
20 m
tower is 30°, then the distance of the object from the tower 12 13
is 5 10
(c)
20 m (d)
17 m
(a)
25 3 m (b)
50 3 m 13 12
(c)
75 3 m (d) 150 m 19. A vertical pole of height 10 metres stands at one corner of
a rectangular field. The angle of elevation of its top from
13. The angles of elevation of the top of a tower from two the farthest corner is 30º, while that from another corner
points at distances m and n metres are complementary. If is 60º. The area (in m2) of rectangular field is
the two points and the base of the tower are on the same
200 2 (b) 400
straight line, then the height of the tower is (a)
3 3
(a) mn (b) mn
m 200 2 400 2
(c) (d) None of these (c) (d)
n 3 3
14. If the height and length of the shadow of a man are the 20. From the top of a building of height ‘h’ meter, the angle
same, then the angle of elevation of the sun is of elevation of the top of the tower is ‘a’ and angle of
(a) 45° (b) 60° depression of the foot of the tower is ‘b’. The height of
the tower is
(c) 90° (d) 120°
h + (tan α + tan β) h(tan α + tan β)
15. The height of a tree, if it casts a shadow 15 m long on the (a) (b)
level of ground, when the angle of elevation of the sun is tan β tan β
45°, is
h tan( α + β) h + tan( α + β)
(a) 10 m (b) 14 m (c) (d)
tan β tan β
(c) 8 m (d) 15 m
M-112 Mathematics
21. Which of the following is/are incorrect? 23. What is the angle of elevation if they are standing at a
on A distance of 42m away from the monument?
ati
bse
rv (a) 30° (b) 45°
o
e of (c) 60° (d) 0°
Lin  Horizontal line
O 24. They want to see the tower at an angle of 60°. So, they
C
Li
ne  want to know the distance where they should stand and
of hence find the distance.
ob
ser
va (a) 25.24 m (b) 20.12 m
tio
n B (c) 42 m (d) 24.64 m
(a)
θ1 is the angle of elevation. 25. If the altitude of the Sun is at 60°, then the height of
the vertical tower that will cast a shadow of length
(b)
θ2 is the angle of depression. 20 m is
(c) The angle of elevation or depression is always measured (a) 20√3 m (b) 20/√3 m
from horizontal line through the point of observation. (c) 15/√3 m (d) 15√3 m
(d)
θ1 and θ2 are always equal. 26. The ratio of the length of a rod and its shadow is 1:1 . The
angle of elevation of the Sun is
22. Which of the following statements are incorrect?
(a) 30° (b) 45°
(a) Line of sight is the line drawn from the eyes of the (c) 60° (d) 90°
observer to a point in the object where the person is
27. The angle formed by the line of sight with the horizontal
viewing. when the object viewd is below the horizontal level is
(b) Angle of elevation is the angle formed by the line of (a) corresponding angle
sight with horizontal through the eyes of observer (b) angle of elevation
when the object is above the horizontal level. (c) angle of depression
(c) Angle of depression is the angle formed by the line (d) complete angle
of sight with the horizontal when the object is below
the horizontal level. Assertion & Reason
(d) None of these
DIRECTIONS : Each of these questions contains an Assertion
followed by Reason. Read them carefully and answer the
question on the basis of following options. You have to select
DIRECTIONS : Study the given Case/Passage and answer the the one that best describes the two statements.
following questions. (a) If both Assertion and Reason are correct and Reason is
the correct explanation of Assertion.
Case/Passage-I
(b) If both Assertion and Reason are correct, but Reason is
A group of students of class X visited India Gate on an not the correct explanation of Assertion.
education trip. The teacher and students had interest in history (c) If Assertion is correct but Reason is incorrect.
as well. The teacher narrated that India Gate, official name (d) If Assertion is incorrect but Reason is correct.
Delhi Memorial, originally called All-India War Memorial,
28. Assertion :
monumental sandstone arch in New Delhi, dedicated to the
troops of British India who died in wars fought between In the figure,
A
1914 and 1919.The teacher also said that India Gate, which is
located at the eastern end of the Rajpath (formerly called the
Kingsway), is about 138 feet (42 metres) in height.
30°
B C

if BC = 20 m, then
height AB is 11.56 m.
AB perpendicular
Reason : tan θ = = ,
BC base
where θ is the angle ∠ACB.
[From CBSE Question Bank 2021]
Some Applications of Trigonometry M-113
29. Assertion : If the length of shadow of a vertical pole is
equal to its height, then the angle of elevation of the sun Fill in the Blanks
is 45°.
Reason : According to pythagoras theorem, h2 = l2 + b2, DIRECTIONS : Complete the following statements with an
where h = hypotenuse, l = length and b = base appropriate word / term to be filled in the blank space(s).

Match the Following 32. The ............... is the line drawn from the eye of an observer
to the point in the object viewed by the observer.
DIRECTIONS : Each question contains statements given in two
33. The ............. of an object viewed, is the angle formed by
columns which have to be matched. Statements (A, B, C, D) in
the line of sight with the horizontal when it is above the
column-I have to be matched with statements (p, q, r, s) in column-II.
horizontal level, i.e., the case when we raise our head to
30. From a window, h metres high above the ground, of a house look at the object.
in a street, the angles of elevation and depression of the top
and bottom of another house on the opposite side of the street 34. The ....................... of an object viewed, is the angle formed
are α and β respectively, then match the column. by the line of sight with the horizontal when it is below
E the horizontal level, i.e., the case when we lower our head
to look at the object.
35. In the adjoining figure, the positions of observer and object
α
D
β A are marked.
O
(Observer)
hm

β B
30°
C
P (Object) Horizontal
Column-I Column-II
(A) DB (p) h(1 + tana cotb) The angle of depression is ...............
(B) DE (q) h / sinb
(C) CE (r) h tan a cot b 36. The top of a building from a fixed point is observed at an
(D) AD (s) h cot b angle of elevation 60° and the distance from the foot of
31. Column-I Column-II the building to the point is 100 m. then the height of the
(A) A (p) 60º building is................

10 37. In fig. the angles of depressions from the observing


45°
positions O1 and O2 respectively of the object A are
B
BC = ?
C .................., ................... .
(B)
A (q) 10 O2 O1

60°
60°
B C
3
AB = ?
1
(C)
A
(r) 45°
5 A
40 B C


B
20
C True / False
=?
A
DIRECTIONS : Read the following statements and write your
(D) (s) 3
answer as true or false.

45°  A straight highway leads to the foot of a tower of height 50 m.


B D
C From the top of the tower, the angles of depression of two cars
2
10 standing on the highway are 30° and 60°. [ 3 = 1.73]
tan  = ?
M-114 Mathematics
Now, based on the above information, mark the given statements 38. Distance between the cars is 57.67 m.
as true or false.
39. First car is at a distance of 38.90 m from the tower.
A
40. Second car is at a distance of 86.50 m. from the tower.
41. Car at point C is at a distance of 200 m away from the top

50 m
of the tower.
30° 60°
C D B
Some Applications of Trigonometry M-115

ANSWER KEY & SOLUTIONS


1.
(a) Let AB be the building of height 20m and BC be the
transmission tower of height h metres.
AB 20
In ∆ OAB, tan 45° = ⇒1=
OA OA

⇒ OA = 20 m
In ∆OAC,

h + 20
tan 60° =
20
h
h + 20 In rt. ∆BDC, tan (90 – θ) =
⇒ 3 = y
20
h
⇒ h = 20 ( 3 –1 m ) ⇒ cot θ =
y
 ... (ii)

2. (b) BD = DC given as D is the mid point of BC. Multiplying (i) by (ii), we get
A h h
tan θ × cot θ = ×
x y

 1 h2
⇒ tan θ × =
tan θ xy

2
⇒ 1 = h ⇒ h 2 = xy
C xy
B D
⇒ h = xy
AC
In ∆ADC, cot θ = ... (i) 4. (d)
CD
AC
In ∆ABC, cot φ = ... (ii)
BC
AC
cot θ CD
∴ =
cot φ AC
BC
AC
= CD [∵ 2CD = BC] Let the broken part of the ladder, AC = x metres
AC
2CD ∴ length of ladder = AC + AB = 15 m
cot θ 2
⇒ = ⇒ x + AB = 15 ⇒ AB = (15 – x)m
cot φ 1
In ∆ ABC,
tan θ 1
⇒ =
tan φ 2 AB 15 − x 3 15 − x
sin 60° = = ⇒ =
3. (b) Let DC be the tower of height ‘h’ metres. AC x 2 x

In rt. ∆ACD, tan θ =


h
… (i)
⇒ 3 x = 30 − 2 x ⇒ x ( )
3 + 2 = 30
x
M-116 Mathematics

⇒ x=
30
=
(
30 2 − 3 ) From (i) and (ii)
tan2α + 2tanα – 1 = 0
( )
3 + 2 ( 2 )2 − 3 2
∴ tan α = 2 − 1

⇒ x = 30 ( 2 − 3 ) = 8.04 m
9. (a)
In figure, the electrician is required to reach the point
Hence, AB = 15 – x = (15 – 8.04)m = 6.96 m B on the pole AD.
5. (a) Let h be the length of the pole. A

6 h
By the given condition =
4 50
B
6 × 50
⇒ h = = 75m
4
60°
6. (a)
Here, In rt. ∆ABD,  A
D C
h
+ BC So, BD = AD – AB = (5 – 1.3)
BD
cot α = = 2 ....(i) = 3.7 m
AB h
also, In rt. ∆ABC,
hm BD 3.7 3
So, sin 60° = ⇒ =
BC BC BC BC 2
cot β = =
AB h 3.7 × 2
α β ∴ BC = = 4.28 m (Approx.)
Now, cot α – cot β 3
D C B
h m
h 2 i.e., the length of the ladder should be 4.28 m
+ BC BC
2 1
= − = DC 1
h h 2 Now, = cot 60° =
BD 3
7. (a)
A
3.7
i.e., DC = = 2.14m (Approx )
3
15 dge
0m
i

Therefore, he should place the foot of the ladder at a


Br

River River
distance of 2.14 m from the pole.
45°
C B
10. (b)
Let A1 and A2 be the position of the two aeroplanes.
and A1A2 = x m
x 1
= sin 45° = OM = y m
150 2

150 2 y
⇒ x= . = 75 2 m. = cot 30° = 3
2 2 3125

8. (b) In the following figure, ⇒ ⇒ y = 3 × 3125


A1

Let OB = x m.
A2
h
tan α = ...(i) 60° 3125 m
2h + x
30°
h O M
tan(90° − α ) = ⇒ x = tanα  ... (ii) y
x
Some Applications of Trigonometry M-117
y 1 C
Also, = cot 60° =
3125 + x 3


(3125) 3
=
1
3125 + x 3
⇒ 3125 + x = (3125) (3) ⇒ x = 6250 m.

BC
11. (d) Hint: sin 30° = A 15 m B
AC
1 BC In ∆ ABC,
⇒ = ⇒ BC = 3 cm.
2 6
BC
AB = tan 45°
12. (c) Hint: tan30° = BA
OB
⇒ BC = AB = 15 m.
A
30° 16. (c) ∠ XCA = ∠CAB = 60°

Hence, angle of depression = 60°


Tower 17. (b)
D
30°
B
O (object)
x
1 75
⇒ =
3 OB 30o 60o
A 20 m B y C
⇒ OB = 75 3 m
x
13. (a) In ∆DBC, tan 60o = ⇒  x = 3y  ... (i)
y
14. (a) Let AB be the height of a man and BC be the shadow
x
of a man. In ∆ΑDC, tan 30o =
A 20 + y

1 3y
⇒ = (Q From (i))
3 20 + y
⇒ y + 20 = 3y
⇒ 2y = 20 ⇒ y = 10 m
18. (b) R
A
C B

∴ AB = BC D
Q
In ∆ ABC,
B
C
AB AB Taking DARC and DBQC
tan θ = ⇒ = tan θ ⇒ tan θ = 1 ⇒ θ = 45°
BC AB
R Q
⇒ tan q = 1 ⇒ q = 45°
45 20
15. (d) Let BC be the tree of height h metre.
 90 – 
Let AB be the shadow of tree. A C B C
M-118 Mathematics
tan(90 – q) = cotq 20. (b) In figure,
20 AC y
=  (let AC = BC = AB = x m) tan a = and
BC 45 x
h
x2 = 900 ⇒ x = 30 tan b =
x
Now, in D ARD and BQD
tan α y
\ =
45 20 AD 9 tan β h
= = tan θ ⇒ =
AD BD BD 4
 tan α 
h + y = h 1 +
9  10   tan β 
AD = × 30 = 20   m
13  13 
21. (d)
19. (a) Let PQRS be the rectangular field with length l and 22. (d) All the statements given in option a, b and c are correct.
width b and RT be the vertical pole.
23. (b) A
T

42 m
60°
S R
C 42 m B
30°
Since h = 42
P Q d = 42
So q = 45°
RT 1 10
In ∆PRT, tan 30° = ⇒  = 42
PR 3 PR 24. (a) tan 60° =
x
PR = l 2 + b 2 = 10 3 42 42
x= = 3 = 14 3 = 24.24 m
3 3
l2 + b2 = 100 × 3 = 300 ... (i)
25. (a) A
10
In ∆SRT, tan 60° =
SR
10
⇒ 3= h
SR
10
SR = =l 60°
3 C 20 m B
100 h
⇒ l2 = ... (ii) tan 60° =
3 20
Use equation (ii) in (i), then h = 20 3 m
100 800 26. (b)
b2 = 300 − = A
3 3
100 800 80000
l2b2 = × =
3 3 9 x
200 2
⇒ lb = 45°
3 C B
x
Hence, the required area of rectangular field is
Since ratio of length of a rod: Shadow of rod = 1:1
200 2 .
3 so Angle = 45°
Some Applications of Trigonometry M-119
27. (a) Corresponding angle 32. line of sight
28. (a) Both the assertion and reason are correct, reason is 33. angle of elevation
the correct explanation of the assertion.
34. angle of depression
AB AB
tan30° = = 35. 30°
BC 20
1 20 36. 100 3 m
AB = × 20 = = 11.56 m.
3 1.73 37. [30°, 45°]
29. (b)
Both assertion and reason are correct, but reason is O2 O1
F E
not the correct explanation of the assertion.
60°
30. (A) → q; (B) → r; (C) → p; (D) → s
31. (A) → q ; (B) → s ; (C) → p ; (D) → r
AB 45°
(A) tan 45° = ⇒ BC = 10 A
BC B C
AB AB Depression angle at O1 = 90° – 60° = 30°
tan 60° =
(B) = ⇒ AB = 3 × 3 = 3
BC 3 Depression angle at O2 = 45° = ∠BAO2
20 1 38. True
(C)
cos θ = = = cos 60º ⇒ θ = 60º
40 2
AB 39. False
(D) In ∆ABC, tan 45° = ⇒ AB = 2, and
BC 40. True
AB 2 1 41. False
tan θ = = = .
BD 10 5
14 Acids,
Surface
Salts
Bases and
Areas
and Volumes
20
Multiple Choice Questions (MCQs) (a) 
π 10 cm
50
DIRECTIONS : This section contains multiple choice (b)
questions. Each question has 4 choices (a), (b), (c) and (d) out π
of which only one is correct. 25
(c)
π 2 cm
1. The volume of a cylinder is 448 p cm3 and height 7 cm. (d) 40 π
Find its lateral surface area and total surface area is
7. If h be the height and α the semi-vertical angle of a right
(a) 253 cm2 (b) 352 cm2
2
circular cone, then its volume is given by
(c) 532 cm (d) 325 cm2
1 3 1 2
2. The diameter of a garden roller is 1.4 m, and 2m long. (a) πh tan 2 α (b) πh tan 2 α
3 3
How much area will it cover in 5 revolutions.
1 1 3
(a) 44 m2 (b) 140 m2 (c) πh 2 tan 3 α (d) πh tan 3 α
2 3 3
(c) 440 m (d) 220 m2
8. Cubes A, B, C having edges of 18 cm, 24 cm and 30 cm
3. If a sphere and a cube have equal surface areas, then the respectively are melted and moulded into a new cube D.
ratio of the diameter of the sphere to the edge of the cube is Find the edge of the bigger cube D.
(a) 1 : 2 (b) 2 : 1 (a) 32 (b) 28
(c) 6: ≠
≠ : 6 (d) (c) 39 (d) 36
4. The internal and external diameter of a hollow 9. The height of a conical tent is 14 m and its floor area is 346.5
hemispherical vessel are 42 cm and 45.5 cm. respectively. m­­­2. The length of canvas, 11m wide, required for it is.
Find its capacity (volume) and also its outer curved (a) 490 m (b) 525 m
surface area. (c) 665 m (d) 860 m
(a) 5.27 litres, 3253.25 cm2
10. Three identical cones with base radius r are placed on
(b) 5.20 litres, 3253.25 cm2 their bases so that each is touching the other two. The
(c) 5.27 litres, 3200.18 cm2 radius of the circle drawn through their vertices is –
(a) smaller than r
(d) 5.27 litres, 3250.25 cm2 (b) equal to r
5. If h, c, v are respectively the height, the C.S.A and the (c) larger than r
(d) depends on the height of the cones
volume of a cone, find the value of 3πvh3 – c2h2 + 9v2
(a) 1 (b) 2 11. The diameter of hollow cone is equal to the diameter of a
(c) 0 (d) 3 spherical ball. If the ball is placed at the base of the cone,
what portion of the ball will be outside the cone?
6. The diagram shows the parts of a right cylinder. The
(a) 50% (b) less than 50%
volume of the cylinder, in cm3 is
(c) more than 50% (d) 100%
Surface Areas and Volumes M-121
12. A slab of ice 8 inches in length, 11 inches in breadth, and 21. In the adjoining figure, the bottom of the
2 inches thick was melted and resolidified in the form of glass has a hemispherical raised portion. If
a rod of 8 inches diameter. The length of such a rod, in the glass is filled with orange juice, the
inches, is nearest to quantity of juice which a person will

15cm
(a) 3 (b) 3.5 get is
(c) 4 (d) 4.5 (a) 135 π cm3
13. The base radii of a cone and a cylinder are equal. If their (b) 117 π cm3
curved surface areas are also equal, then the ratio of the (c) 99 π cm3
slant height of the cone to the height of the cylinder is
6cm
(d) 36 π cm3
(a) 2 : 1 (b) 1 : 2
(c) 1 : 3 (d) 3 : 1 22. Ratio of lateral surface areas of two cylinders with equal
height is
14. If the perimeter of one face of a cube is 20 cm, then its
(a) 1 : 2 (b) H : h
surface area is
(c) R : r (d) None of these
(a) 120 cm2 (b) 150 cm2
2
(c) 125 cm (d) 400 cm2 23. Ratio of volumes of two cylinders with equal height is
(a) H : h (b) R:r
15. A cube of side 12 cm is painted red on all the faces and
(c) R2 : r2 (d) None of these
then cut into smaller cubes, each of side 3 cm. What is the
total number of smaller cubes having none of their faces 24. Ratio of volumes of two cones with same radii is
painted? (a) h1 : h2 (b) s1 : s2
(a) 16 (b) 8 (c) r1 : r2 (d) None of these
(c) 12 (d) 24 25. Volume of a spherical shell is given by
16. If the diameter of the sphere is doubled, the surface (a) 4π (R2 – r2) (b) π (R3 – r3)
area of the resultant sphere becomes x times that of the 4
(c) 4π (R3 – r3) (d) π (R3 – r3)
original one, then x would be 3
(a) 2 (b) 3
(c) 4 (d) 8 26. The volume of a largest sphere that can be cut from
cylindrical log of wood of base radius 1m and height 4 m, is
17. If h be the height and α the semi-vertical angle of a right
16 8
circular cone, then its volume is given by (a) ≠ m3 (b) ≠ m3
3 3
1 1 2
(a) πh3 tan 2 α (b) πh tan 2 α 4 10
3 3 (c) ≠ m3 (d) ≠ m3
3 3
1 1
(c) πh 2 tan 3 α (d) πh3 tan 3 α
3 3 27. If four times the sum of the areas of two circular faces
of a cylinder of height 8 cm is equal to twice the curve
18. If the radius of the sphere is increased by 100%, the surface area, then diameter of the cylinder is
volume of the corresponding sphere is increased by (a) 4 cm (b) 8 cm
(a) 200% (b) 500% (c) 2 cm (d) 6 cm
(c) 700% (d) 800%
28. A rectangular sheet of paper 40 cm × 22 cm, is rolled to
19. A sphere is melted and half of the melted liquid is used form a hollow cylinder of height 40 cm. The radius of the
to form 11 identical cubes, whereas the remaining half is cylinder (in cm) is
used to form 7 identical smaller spheres. The ratio of the (a) 3.5 (b) 7
side of the cube to the radius of the new small sphere is (c) 80/7 (d) 5
1/ 3 1/ 3
4 8 29. A right circular cylinder has its height equal to two times
(a)   (b)  
3 3 its radius. It is inscribed in a right circular cone having
(c) (3)1/3 (d) 2 its diameter equal to 10 cm and height 12 cm, and the
axes of both the cylinder and the cone coincide. Then, the
20. If a solid of one shape is converted to another, then the
volume (in cm3) of the cylinder is approximately
volume of the new solid
(a) 107.5 (b) 118.6
(a) remains same (b) increases
(c) 127.5 (d) 128.7
(c) decreases (d) can’t say
M-122 Mathematics
30. If the ratio of volumes of two cubes is 27 : 64, then the 36. The number of solid cones with integer radius and integer
ratio of their surface area is: height each having its volume numerically equal to its
(a) 3 : 4 (b) 4 : 3 total surface area is
(c) 9 : 16 (d) 16 : 9 (a) 0 (b) 1
(c) 2 (d) infinite
31. Volumes of two spheres are in the ratio 125 : 64. The ratio
of their surface areas will be 37. A solid metallic cylinder of height 10 cm and diameter 14
(a) 5 : 4 (b) 25 : 16 cm is melted to make two cones in the proportion of their
(c) 16 : 25 (d) 125 : 64 volumes as 3 : 4, keeping the height 10 cm, what would
be the percentage increase in the flat surface area?
32. A solid metallic block of volume one cubic metre is (a) 9 (b) 16
melted and recast into the form of a rectangular bar of (c) 50 (d) 200
length 9 metres having a square base. If the weight of the
block is 90 kg and biggest cube is cut off from the bar, 38. Given three cubes with integer side lengths, if the sum of
then the weight of the cube is surface areas of three cubes is 498 sq. cm, then the sum
of the volumes of the cubes in all possible solutions is
1 2
(a)
6 kg (b) 5 kg (a) 731 (b) 495
3 3 (c) 1226 (d) None of these
2 1
(c)
4 kg (d) 3 kg 39. Which one of the following is/are incorrect ?
3 3
(a) Total surface area of cuboid is 2(lb + bh + hl)
33. Consider a cuboid all of whose edges are integers and (b) Total surface area of a cube is 4l2
whose base is a square. Suppose the sum of all its edges (c) Area of four walls = 2h (l + b)
is numerically equal to the sum of the areas of all its six (d) Area of four walls = Height × Perimeter of the room
faces. Then, the sum of all its edges is
40. A Circular Cylinder can not be separated into
(a) 12 (b) 18
(a) circular end at the bottom
(c) 24 (d) 36
(b) curved surface
34. Shyam wants to make a solid brick shape structure from (c) circular end at the top
400 wooden cubes of unit volume each. If the sides of (d) None of these
the solid brick have the ratio 1 : 2 : 3, then the maximum
41. Which one of the following is / are incorrect ?
number of cubes, which can be used, will be _______.
(a) Total surface area of cylinder is 2πr2 + 2πrh.
(a) 400 (b) 288
(b) Total surface area of a sphere is 4πr2.
(c) 300 (d) 384
(c) Total surface area of cone is πr2 + πrl.
35. Sealed bottle containing some water is made up of two (d) None of these
cylinders A and B of radius 1.5 cm and 3 cm respectively,
42. Which one of the following is/ are made up of
as shown in the figure. When the bottle is placed right up
combinations of two or more of the basic solids?
on a table, the height of water intit is 15 cm, but when
(a) Buildings (b) Funnel
placed upside down the height of water is 24 cm. The
(c) Monuments (d) Test-tube
height of the bottle is
43. Among the following, which one is/are correct?
(a) The slant height is the longest side of a pyramid.
(b) The section between the base and a plane parallel to
the base of a solid is known as frustum.
(c) All the surfaces of a cuboid are square.
15 cm (d) For a cylinder, the top, the bottom and the walls of
24 cm
the cylinder determine the total surface area.
44. If a marble of radius 2.1 cm is put into a cylindrical cup
(a) 25 cm (b) 26 cm full of water of radius 5cm and height 6 cm, then how
(c) 27 cm (d) 28 cm much water flows out of the cylindrical cup?
(a) 38.8 cm3 (b) 55.4 cm3
(c) 19.4 cm3 (d) 471.4 cm3
Surface Areas and Volumes M-123
45. A cubical ice-cream brick of edge 22 cm is to be distributed
among some children by filling ice-cream cones of radius
2 cm and height 7 cm upto its brim. How many children
will get the ice-cream cones? Area = 551 m2
(a) 163 (b) 263 (c) 363 (d) 463 r=7m
46. The volume of the largest right circular cone that can be
cut out from a cube of edge 4.2 cm is
(a) 9.7 cm3 (b) 77.6 cm3 49. The volume of cylindrical cup is
3
(c) 58.2 cm (d) 19.4 cm3 (a) 295.75cm3 (b) 7415.5cm3
3
(c) 384.88cm (d) 404.25cm3
47. If two solid hemispheres of same base radius r are joined 50. The volume of hemispherical cup is
together along their bases, then curved surface area of (a) 179.67cm3 (b) 89.83 cm3
this new solid is 3
(c) 172.25 cm (d) 210.60 cm3
(a) 4πr2 (b) 6πr2 (c) 3πr2 (d) 8πr2
51. Which container had more juice and by how much?
48. A right circular cylinder of radius r cm and height h cm (a) Hemispherical cup, 195 cm3
(h > 2r) just encloses a sphere of diameter (b) Cylindrical glass, 207 cm3
(a) r cm (b) 2r cm (c) h cm (d) 2h cm (c) Hemispherical cup, 280.85 cm3
(d) Cylindrical glass, 314.42 cm3
52. The height of the conical tent prepared to accommodate
four students is
DIRECTIONS : Study the given Case/Passage and answer (a) 18m (b) 10m
the following questions. (c) 24m (d) 14m
53. How much space on the ground is occupied by each
Case /Passage-I student in the conicaltent
Adventure camps are the perfect place for the children to (a) 54m2 (b) 38.5m2
practice decision making for themselves without parents and (c) 86m 2 (d) 24m2
teachers guiding their every move. Some students of a school
reached for adventure at Sakleshpur. At the camp, the waiters Case /Passage-II
served some students with a welcome drink in a cylindrical glass
and some students in a hemispherical cup whose dimensions are
shown below. After that they went fora jungle trek. The jungle
trek was enjoyable but tiring. As dusk fell, it was time to take
shelter. Each group of four students was given a canvas of area
551m2. Each group had to make a conical tent to accommodate
all the four students. Assuming that all the stitching and wasting
incurred while cutting, would amount to 1m2, the students put
the tents. The radius of the tent is 7 m.
[From CBSE Question Bank 2021]

[From CBSE Question Bank 2021]


The Great Stupa at Sanchi is one of the oldest stone structures
in India, and an important monument of Indian Architecture. It
was originally commissioned by the emperor Ashoka in the
3rd century BCE. Its nucleus was a simple hemispherical
brick structure built over the relics of the Buddha. .It is a perfect
example of combination of solid figures. A big hemispherical
22
dome with a cuboidal structure mounted on it.  Take π = 
 7
54. Calculate the volume of the hemispherical dome if the
height of the dome is21 m –
d = 7 cm
h = 10.5 cm (a) 19404 cu. m (b) 2000 cu .m
d = 7 cm
(c) 15000 cu. m (d) 19000 cu. m
M-124 Mathematics
55. The formula to find the Volume of Sphere is- 63. The total surface area of cone with hemispherical ice cream
2 3 4 3 is
(a) πr (b) πr (a) 858 cm2 (b) 885 cm2
3 3
(c) 588 cm2 (d) 855 cm2
(c) 4 pr2 (d) 2 pr2
56. The cloth require to cover the hemispherical dome if the
radius of its base is14m is Assertion & Reason
(a) 1222 sq.m
(b) 1232 sq.m DIRECTIONS : Each of these questions contains an Assertion
(c) 1200 sq.m followed by Reason. Read them carefully and answer the
(d) 1400 sq.m question on the basis of following options. You have to select
57. The total surface area of the combined figure i.e. the one that best describes the two statements.
hemispherical dome withradius 14m and cuboidal shaped
top with dimensions 8m × 6m × 4m is (a) If both Assertion and Reason are correct and Reason is
(a) 1200 sq. m (b) 1232 sq. m the correct explanation of Assertion.
(c) 1392 sq.m (d) 1932 sq. m (b) If both Assertion and Reason are correct, but Reason is
58. The volume of the cuboidal shaped top is with dimensions not the correct explanation of Assertion.
mentioned in question 4
(c) If Assertion is correct but Reason is incorrect.
(a) 182.45 m3 (b) 282.45 m3
(c) 292m 3 (d) 192m3 (d) If Assertion is incorrect but Reason is correct.
Case /Passage-III 64. Assertion: Total surface area of the cylinder having
radius of the base 14 cm and height 30 cm is 3872 cm2.
On a Sunday, your Parents took you to a fair. You could see
lot of toys displayed,and you wanted them to buy a RUBIK’s Reason: If r be the radius and h be the height of the
cube and strawberry ice-cream for you. Observe the figures and cylinder, then total surface area = (2πrh + 2πr2 ).
answer the questions-: [From CBSE Question Bank 2021]
65. Assertion: If the height of a cone is 24 cm and diameter of
the base is 14 cm, then the slant height of the cone is 15 cm.
Reason: If r be the radius and h the slant height of the
cone, then slant height = h2 + r 2 .

66. Assertion: If the radius of a cone is halved and volume is


not changed, then height remains same.
Reason: If the radius of a cone is halved and volume is
not changed then height must become four times of the
59. The length of the diagonal if each edge measures 6cm is original height.
(a) 3√3 (b) 3√6 67. Assertion: If a ball is in the shape of a sphere has a
(c) √12 (d) 6√3
surface area of 221.76 cm2, then its diameter is 8.4 cm.
60. Volume of the solid figure if the length of the edge is 7cm
Reason: If the radius of the sphere be r, then surface
is-
(a) 256 cm3 (b) 196 cm3 1 S
area, S = 4πr2, i. e. r = .
(c) 343 cm3 (d) 434 cm3 2 π

61. What is the curved surface area of hemisphere (ice cream) 68. Assertion: No. of spherical balls that can be made out of
if the base radiusis 7cm? a solid cube of lead whose edge is 44 cm, each ball being
(a) 309 cm2 (b) 308 cm2 4 cm. in diameter, is 2541
(c) 803 cm 2 (d) 903 cm2
Volume of one ball
62. Slant height of a cone if the radius is 7cm and the height Reason : Number of balls = .
volume of lead
is 24 cm
(a) 26cm (b) 25 cm
(c) 52 cm (d) 62cm
Surface Areas and Volumes M-125
Column-I Column-II
Match the Following
(A) Area of bottom of cylinder (p) 10.56
(B) Outer curved surface area (q) 1.54
DIRECTIONS : Each question contains statements given in
two columns which have to be matched. Statements (A, B, C, (C) Curved area of conical cavity (r) 5.5
D)in Column-I have to be matched with statements (p, q, r, s) (D) Total surface area (s) 17.6
in Column-II. 72. Column-I (Object) Column-II (Shape)
69. For figure shown, match the column (A) Dice (p) Right circular cone
(B) Road rollers (q) Sphere
Hemisphere (C) Ice-cream cone (r) Cylinder
(D) Volleyball (s) Cube
3.5cm

5cm. A Fill in the Blanks

DIRECTIONS : Complete the following statements with an


appropriate word / term to be filled in the blank space(s).

73. The volume of a cube with diagonal d is .............


Fig. : Top (Lattu)
74. A cube is a special type of ..............
Column -I Column-II
75. If the heights of two cylinders are equal and their radii
(A) Curved area of hemisphere (p) 3.25
are in the ratio of 7 : 5, then the ratio of their volumes is
(B) Height of cone (q) 77/4 .................
(C) Slant height of cone (r) 3.7
76. The volume of a solid is the measurement of the portion
(D) Surface area of top (s) 39.6 of the ............ occupied by it.
70. For a wooden article was made by scooping out a 77. If the volume of a cube is 64 cm3, then its surface area is
hemisphere from each end of a solid cylinder, as shown ....................
in Fig. If the height of the cylinder is 10 cm, and its base
is of radius 3.5 cm, match the column. 78. If the volume and the surface area of a solid sphere are
numerically equal, then its radius is .........

79. The length of the diagonal of a cube that can be inscribed


in a sphere of radius 7.5 cm is ............

True / False

DIRECTIONS : Read the following statements and write your


Column-I Column-II answer as true or false.
(A) Volume of cylinder (p) 616/3
80. Volume of the solid is measured in cubic units.
(B) Volume of scoops (q) 374
81. Area is the length of the boundary of a closed figure.
(C) Total surface area (r) 122.5 π
82. Area is the total surface covered by a closed figure.
(D) Volume of the article (s) 171.5/3 π
71. From a solid cylinder of height 2.4 cm and diameter πd 3
83. The volume of sphere of diameter is .
1.4cm a conical cavity of the same height and same 6
diameter is hollowed out then match the column.
M-126 Mathematics
84. The total surface area of a solid cylinder of radius r and 87. If the curved surface of a right circular cylinder is
height h is 2π r (h + r). 80
1760 cm2 and its radius is 21 cm, then its height is
85. If a right circular cone and a cylinder have equal circles 3
cm.
as their base and have equal heights, then the ratio of
their volumes is 2 : 3 50 2
88. If the total surface area of a cube is m , then its side
86. If the base area and the volume of a cone are numerically is (5/3) m. 3
equal, then its height is 3 units.
Surface Areas and Volumes M-127

ANSWER KEY & SOLUTIONS


1. (b) Volume of cylinder = πr2h 2
1  1 
448π = πr2 × 7 ⇒ 3π  πr 2 h  h3 − (πr )2 h 2 + 9  πr 2 h 
3  3 
448 448
⇒ r2 = ⇒ r= 1 
⇒ 3π  πr 2 h  h3 − π 2 r 2 (r 2 + h 2 )
7   7 3 

⇒ r = 64 ⇒ r = 8 cm 1
h 2 + 9 × π 2 r 4 h 2  (∵  = h2 + r 2 )
∴ L. S. A or C. S. A = 2ph 9

22 ⇒ p2r2h4 – p2r4h2 – p2r2h4 + p2r4h2 = 0 = R.H.S.


= 2 × × 8 × 7 = 352 cm2
7 6. (b) Height of cylinder, h = 2 cm
1.4 Circumference of base of cylinder = 10 cm
2. (a) r = = 0.7 m and h = 2 m
2
5
Area covered   = C.S.A × number of revolutions ⇒ 2πr = 10  ⇒  r =
π
= 2ph × 5 = 10prh
25 50
Volume of the cylinder = πr 2 h = π × 2 × 2 =
 22  2 π π
⇒ 10   (0.7)(2) = 44 m
 7 
7. (a) In right ∆AOB A
3. (d) Let the diameter of the sphere be d units and the edge
OB
of the cube be a units, then = tan α ⇒ OB = h tan α α
OA h
2 2
d  2 d 6 d 6 ⇒ r = h tan α
4π   = 6a   ⇒  =   ⇒  = . B
2
  a 2 π a π
O r
1 2
Volume of cone = πr h
45.5 42 3
4. (a) =R = 22.75 cm ,=
r = 21 cm
2 2
1 1
Capacity (volume) = π(h tan α) 2 h = πh3 tan 2 α
3 3
2 2 22
= π (R 3 − r 3 ) = × × [(22.75)3 − (21)3 ] 8. (d) Volume of A = 183 = 5832 cm3
3 3 7
Volume of B = 243 = 13824 cm3
= 2 × 22 × 2513.543 = 5266.46 cm3
Volume of C = 27000 cm3
3 7
5266.46 Total volume A, B and C = 46656 cm3
or = lts. = 5.27 litres
1000 Now,volumeofnewcubeD=sumofvolumesofA,BandC.
Outer surface area Let a be the edge of D.
22 2 Then a3 = 46656  ⇒  a = 36 cm.
= 2πR2 = 2 × × (22.75) = 3253.25 cm2
7
 3465 7 
2 2 9. (b) pr2 =346.5  ⇒  r2 =  × 
5. (c) We know that, l = r +h ,  10 22 
2
441  21 
v =
1 2
πr h , c = prl = =     ⇒  r = 21 m
3 4  2 2
441 2 1225 35
L.H.S. 3πvh3 – c2h2 + 9v2 l = r 2 + h2 = + (14 )= = m
4 4 2
M-128 Mathematics
Area of canvas 1 2 1
24. (a) ≠r1 h1 : ≠r22 h2
 22 21 35  3 3
= πrl =  × ×  m 2 = 577.5m 2
 7 2 2 1 2 1
⇒ ≠r1 h1 : ≠r12 h2 (Q r1 = r2)
Area 577.50 3 3
Length of canvas = = m = 525 m
Width 1.1 ⇒ h1 : h2
10. (c) The centres of the bases of the cones form a triangle 25. (d) Volume of spherical shell
of side 2r. The circumference of the circle will be
4 3 4 3 4
identical to a circle drawn through the vertices of the = πR − πr = π( R3 − r 3 )
3 3 3
cones and thus, it will have a radius of 2 / 3 times
r, which is greater than r. 4 3 4
26. (c) Volume of sphere = ≠r = ≠ (1)3
11. (c) Though, it is given that diameter of the 3 3
cone is equal to the diameter of the 27. (b) 28. (a)
spherical ball. But the ball will not fit into 29. (c) Let h be the height and r be the radius of the cylinder.
the cone because of its slant shape.
Hence, more than 50% of the portion of Given h = 2r
the ball will be outside the cone. Since, ∆PSC ~ ∆AOC
12. (b) Volume of the given ice cuboid = 8 × 11 × 2 = 176 ∴ By similarity of two triangles, we have
Let the length of the required rod be l.
PS SC PC h 5 − r PC
= = ⇒ = =
82 AO OC AC 12 5 AC
\ π = 176   \  l = 3.5 inches
4 h 5− r 2r 5 − r
⇒ = ⇒ = ⇒ 10r = 60 – 12r
 2 12 5 12 5
13. (a) πr = 2πrh  ⇒  =
h 1 A

20
14. (b) Edge of cube = cm = 5 cm, Q P 12
4 h
r
surface area = 6×52 cm2 = 150 cm2 B
R O S
C
15. (b)
5
16. (c)
17. (a) 30 60
⇒ r = and h =
11 11
18. (c) When the radius is increased by 100%, the
corresponding volume becomes 800% and thus 22  900  60
Volume of cylinder = πr2h =   ≈ 127.50
increase is 700%. 7  121  11
19. (b) As per the given conditions,
30. (c) Let a1 and a2 be the sides of two cubes and S1 andS2
4 1/ 3 be the surface area of cubes
11a = 7 × × π × r 3
3 a 8
∴ = 
3 r 3 a13 27 a 3 S1 6a12 9
∴ = ⇒ 1 = , = 2 =
20. (a) a23 64 a2 4 S2 6a2 16

21. (b) [Hint. Quantity of juice 4 3


πr
V1 125 3 1 125 r 5
31. (b) = ; = ⇒ 1 = ;
 2 2 3 V2 64 4 πr 3 64 r2 4
=  π × 3 × 15 − π × 3  cm3] 2
 3  3
2
22. (c) 2πRh : 2πrh = R : r SA1 4πr12  r1  25
= =  = = 25 : 16.
23. (c) πR2h : πr2h = R2 : r2 SA2 4πr22  r2  16

Surface Areas and Volumes M-129
32. (d) Volume of metallic block = 1 m3...(1) 35. (c)
Let the side of the square base be x m r

So, volume of the rectangular bar = x2 × 9 ...(2) A h

Volume of metallic block = Volume of rectangular


bar 15 24
B H
1 1
So, 9 x 2 = 1 ⇒ x 2 = ⇒x= m
9 3
1
Side of biggest cube possible = m
2r
3
3 Let the height of cylinder A be h and height of
1 cyilnder B be H. Since, in both the situation, the
So, weight of the cube = weight of block ×  
3 volume of the water in the bottle remains same, then
1 10 1 π(2r)2H + πr2(15 – H) = πr2h + π(2r)2(24 – h)
= 90 × = kg = 3 kg
27 3 3
πr2 [4H + (15 – H)] = πr2[h + 4(24 – h)]
33. (c) Since, all the edges of cuboid are integers and base 4H + 15 – H = h + 4 (24 – h)
is square.
3H + 15 = h + 96 – 4h
Let the length, breadth and height of cuboid be x, x, y.
3H + 15 = 96 – 3h
Sum of all edges of cuboid = 4x + 4x + 4y
H + 5 = 32 – h
Sum of area of all faces = 2x2 + 2xy + 2xy
H + h = 27 cm
Given,
36. (b) Let height and radius of cone is h and r respectively
Sum of all edges of cuboid = Sum of area of all faces such that h, r ∈ I
∴ 4x + 4x + 4y = 2(x2 + xy + xy) Q Volume of cone = Surface area of cone
⇒ x2 + 2xy – 4x – 2y = 0
1 2
∴ πr h = πrl + πr 2
⇒ x2 + 2x (y – 2) – 2y = 0 3

−2( y − 2) ± 4( y − 2)2 + 4(2 y ) ⇒


1 2
πr h = πr h 2 + r 2 + πr 2
⇒ x = 3
2
1
⇒ x = y − 2 ± y 2 − 2 y + 4 ⇒ rh = h 2 + r 2 + r [Q r ≠ 0]
3
Then x is integer, when y = 2 ∴ y = 2, x = 2 2 2
⇒ rh − 3r = 3 h + r
Hence, the required sum of edges of cuboid
⇒ r2h2 + 9r2 – 6hr2 = 9h2 + 9r2
= 8x + 4y = 16 + 8 = 24
6r 2
34. (d) Let sides of the solid brick be x, 2x and 3x. ⇒ h2(r2 – 9) = 6hr2  ⇒  h =
r2 − 9
Volume = Number of bricks = 6x3
 r2 
6x3 < 400 ⇒ h = 6  54
 r2 − 9  ⇒ h = 6+
2
400   r −9
x3 <
6
h and r are integers
2
x3 < 66 Here (r2 – 9) should be factor of 54.
3
Number less than 66 which is perfect cube is 64 than ∴ r2 – 9 = 1, 2, 3, 6, 9, 18, 27, 54.
volume = 6 × 64 = 384 i.e., no. of cube = 384. ⇒ r2 = 10, 11, 12, 15, 18, 27, 36, 63.
M-130 Mathematics
∴ r = 6 (only possible value)
πr 2
53. (b) Space occupied by each student = = 38.5 m 2
54 4
∴ h = 6 + = 6 + 2 = 8 ∴ r = 6, h = 8 54. (a) Volume of hemispherical dome
36 − 9
2 2 22
= πr 3 = × × 21 × 21 × 21 = 19404 cu. m
37. (c) Let r1and r2 be the radii of two cones. 3 3 7
Volume of cylinder = Sum of volume of two cones 4
55. (b) πr 3
3
p 56. (b) Cloth required to cover hemispherical done = curved
p × 72 × 10 = × 10(r12 + r22 )
3 surface area of hemisphere = 2pr2 = 1232 sq. m
57. (c) Surface area of combined figure = 2pr2 + 2(l + b)h
⇒ r 12 + r22 = 49 × 3 ...(i)
= 1232 + 2(6 + 4)8 = 1392 sq. m
\ Percentage increase 58. (d) 192 m3

pr12 + pr22 - 2p ¥ 7 2 r 2 + r22 - 2 ¥ 49 59. (d) Length of diagonal = 3l 2 = 6 3


= ¥ 100 = 1 ¥ 100
2 ¥ p ¥ 72 2 ¥ 49 60. (c) Volume of cube = (side)3 = 343 cm3
61. (b) Curved surface area of hemisphere = 2pr2 = 308 cm2
(49 ¥ 3 - 49 ¥ 2) 62. (b) Slant height = r 2 + h 2 = 25 cm
= ¥ 100  [From (i)]
49 ¥ 2 63. (a) 858 cm2
= 50% 64. (a) Assertion and Reason both are correct and reason is
the correct explanation of the assertion .
38. (c) It is given that 6(x2 + y2 + z2) = 498, where x, y and
z are sides of cubes. Total surface area = 2πrh + 2πr2 = 2πr(h + r)
x2 + y2 + z2 = 83, for x, y, z to be integer 22
= 2× ×14 (30+14) = 88 (44)
7
x = 49 , y = 25 , z = 9 ⇒  x = 7, y = 5, z = 3
= 3872 cm2
Now, sum of volumes= 73 + 53 + 33
65. (d) Assertion is incorrect here, but reason is correct.
⇒  343 + 125 + 27 = 495
For x, y, z to be integer Slant height = (14 / 2)2 + (24)2 = 49 + 576

= 625 = 25 cm
x = 81 , y = 1,z= 1 ⇒  x = 9, y = 1, z = 1
66. (d) Assertion is incorrect and reason is true.
Sum of volumes = 93 + 13 + 13 = 729 + 1 + 1 = 731
V1 (1 / 3)πr 2 h1 4h
So, total sum of volume of cubes is (495 + 731) 1226. = = 1
39. (b) 40. (d) 41. (d) 42. (b) 43. (c) V2 (1 / 3)π(r / 2)2 h2 h2

44. (a) 45. (c) 46. (d) 47. (a) 48. (b)
V1 = V2
As
49. (d) Volume of cylindrical cup = pr2h
22 7 7 ∴ h2 = 4h1
× × × 10.5 = 404.25 cm3
7 2 2 67. (a) Both assertion and reason are correct and reason is
50. (b) Volume of hemispherical cup the correct explanation of the assertion.
3 68. (c) Assertion is correct but reason is not correct.
2 3 3 22  7 
= πr = × ×   = 89.83 cm3
3 2 7  2 69. (A) → (q); (B) → (p); (C) → (r); (D) → (s)
51. (d) Cylindrical glass, 314.42 cm3
70. (A) → (r); (B) → (s); (C) → (q) ; (D) → (p)
52. (c) Curved surface area of cone = 551 ⇒ prl = 441
22 71. (A) → (q); (B) → (p); (C) → (r); (D) → (s)
⇒ × 7 × l = 551
7
72. (A) → (s); (B) → (r); (C) → (p); (D) → (q)
⇒ l = 25.045
d3
2 2
73. cb. units.
3 3
∴ h = l − r = 24 m
Surface Areas and Volumes M-131
74. cuboid 82. True
75. 49 : 25 83. True
76. space 84. True
77. 96 cm2 85. False
78. 3 units 86. True
79. 15 cm 87. False
80. True 88. True
81. False
15 Acids, Bases and
Statistics
Salts
4. The mean weight of a class of 35 students is 45kg.
Multiple Choice Questions (MCQs) If the weight of a teacher be included, the mean
weight increases by 500 grams. Find the weight of
DIRECTIONS : This section contains multiple choice the teacher.
questions. Each question has 4 choices (a), (b), (c) and (d) out (a) 63 kg (b) 61 kg
of which only one is correct.
(c) 64 kg (d) 70 kg
1. For the following grouped frequency distribution 5. In the following distribution
Class 3–6 6–9 9–12 12–15 15–18 18–21 21–24
Monthly income range (in `) Number of families
Frequency 2 5 10 23 21 12 3
Income more than ` 10,000 100
Consider the following. Income more than ` 13,000 85
(I) Lower limit of model class is 12 Income more than ` 16,000 69
(II) Frequency of the class preceding the model class = 10 Income more than ` 19,000 50
(III) Mode = 14.6 Income more than ` 22,000 33
(IV) Width of the model class is 4 Income more than ` 25,000 15
Which of the following is correct? the number of families having income range (in `)
(a) I and II (b) II and III 16000 – 19000 is
(c) I, II and III (d) All of these (a) 15 (b) 16
(c) 17 (d) 19
2. The marks in science of 80 students of class X are given
below. Find the mode of these marks obtained by the 6. Find the mean of the following frequency distribution.
students in science. Class Interval 0–10 10–20 20–30 30–40 40–50
Marks 0–10 10–20 20–30 30–40 40–50 Frequency 8 12 10 11 9
Frequency 3 5 16 12 13
(a) 25. 3 (b) 25.2
Marks 50–60 60–70 70–80 80–90 90–100
Frequency 20 5 4 1 1 (c) 24 (d) 25.5
7. Calculate the mean of the following frequency distribution:
(a) 53.18 (b) 53.25
C. I. 0–80 80–160 160–240 240–320 320–400
(c) 52.25 (d) 53
Frequency 22 35 44 25 24
3. There are three sections A, B and C in class X with 25,
(a) 195.5 (b) 198.8
40 and 35 students respectively. The average marks
obtained by section A, B and C are 70%, 65% and (c) 196.8 (d) 195
50% respectively. Find the average marks of entire 8. If the mean of first ‘n’ odd natural numbers is ‘n’ itself,
class X. what is the value of ‘n’?
(a) 59% (b) 56% (a) 2 (b) 3
(c) 63% (d) 61% (c) 1 (d) Any natural number
Statistics M-133
9. What is the arithmetic mean of 20 fours, 40 fives, 30 sixes 17. In a frequency distribution, the mid value of a class is 10
and 10 tens? and the width of the class is 6. The lower limit of the class
(a) 50 (b) 25 is
(c) 5.6 (d) 33 (a) 6 (b) 7 (c) 8 (d) 12

10. The mean of discrete observations y1, y2, .........., yn is given 18. If the mean of the observations x, x + 3, x + 5, x + 7 and
by x + 10 is 9, then the mean of the last three observations is
1 2
n n
(a)
10 (b) 10
∑y i ∑y i 3 3
(a) i =1
(b) i =1
n 1 2
n (c)
11 (d) 11
∑i
i =1
3 3
19. The mean of fifteen different natural numbers is 13. The
n n
maximum value for the second largest of these numbers
∑y f i i ∑y f i i
is
(c) i =1
(d) i =1
n
n (a) 46 (b) 51 (c) 52 (d) 53
∑f i
i =1 20. For the data (2, 9, x + 6, 2x + 3, 5, 10, 5) if mean is 7, then
11. If the mean of the numbers 27 + x, 31 + x, 89 + x, 107 + x, mode is
156 + x is 82, then the mean of 130 + x, 126 + x, 68 + x, (a) 3 (b) 5 (c) 9 (d) 10
50 + x, 1 + x is 21. The mean of three positive numbers is 10 more than the
(a) 75 (b) 157 smallest of the numbers and 15 less than the largest of the
(c) 82 (d) 80 three. If the median of the three numbers is 5, then the
mean of squares of the numbers is
12. If the class-intervals are 10 – 19, 20 – 29, 30 – 39, ..............,
then the upper limit of the first class-interval is 2 2
(a)
108 (b) 116
(a) 19.5 (b) 19 3 3
(c) 20 (d) None of these 1 2
(c) 208 (d) 216
13. The numbers 3, 5, 7 and 9 have their respective frequencies 3 3
x – 2, x + 2 , x – 3 and x + 3. If the arithmetic mean is 6.5, 22. Positive integers from 1 to 21 are arranged in 3 groups
then the value of x is of 7 integers each, in some particular order. Then the
(a) 3 (b) 4 highest possible mean of the medians of these 3 groups
is ______
(c) 5 (d) 6
(a) 16 (b) 12.5
14. The median of a set of 9 distinct observations is 20.5. If
(c) 11 (d) 14
each of the largest 4 observation of the set is increased by
2, then the median of the new set 23. The mean of 5 observation is 15. If the mean of first three
(a) is increased by 2 observations is 14 and last three observations is 17, then
the third observation is :
(b) is decreased by 2
(a) 18 (b) 19 (c) 17 (d) 20
(c) is two times the original median
24. The mean of a group of eleven consecutive natural numbers
(d) remains the same as that of the original set
is m. What will be the percentage change in the mean when
15. A set of numbers consists of three 4’s, five 5’s, six 6’s, next six consecutive natural numbers are included in the
eight 8’s and seven 10’s. The mode of this set of numbers group ?
is m m 300
(a) 6 (b) 7 (a) m% (b) % (c) % (d) %
3 300 m
(c) 8 (d) 10 25. A 100 mark examination was administered to a class of
16. The mean of a set of 20 observation is 19.3. The mean is 50 students. Despite only integer marks being given, the
reduced by 0.5 when a new observation is added to the set. average score of the class was 47.5. Then, the maximum
The new observation is number of students who could get marks more than the
class average is
(a) 19.8 (b) 8.8
(a) 25 (b) 35 (c) 45 (d) 49
(c) 9.5 (d) 30.8
M-134 Mathematics
26. Let x1, x2 , ..., x11 be 11 distinct positive integers. If we 29. The upper limit of modal class is
replace the largest of these integers by the median of the (a) 15 (b) 25
other 10 integers, then
(c) 35 (d) 45
(a) the median remains the same
30. The mean of the given data is
(b) the mean increases
(a) 26.2 (b) 32.4
(c) the median decreases
(c) 33.5 (d) 35.4
(d) the mean remains the same
Refer to table 2
27. The mean of 15 observations written in some order is
50. If the mean of first eight observations and last eight 31. The mode of the given data is
observations are 48 and 53 respectively, then the eighth (a) 41.4 (b) 48.2
observation is (c) 55.3 (d) 64.6
(a) 35 (b) 80 (c) 72 (d) 58
32. The median of the given data is
(a) 32.7 (b) 40.2
(c) 42.3 (d) 48.6

DIRECTIONS : Study the given Case/Passage and answer the Case/Passage-II


following questions. COVID-19 Pandemic
Electricity energy consumption is the form of energy
Case/Passage-I
consumption that uses electric energy. Global electricity
COVID-19 Pandemic consumption continues to increase faster than world population,
The COVID-19 pandemic, also known as coronavirus pandemic, leading to an increase in the average amount of electricity
is an ongoing pandemic of coronavirus disease caused by the consumed per person (per capita electricity consumption).
transmission of severe acute respiratory syndrome coronavirus 2 [From CBSE Question Bank 2021]
(SARS-CoV-2) among humans. Teriff : LT. Residential
[From CBSE Question Bank 2021]
Type of Supply : Single Phase
Meter Reading Date : 31-11-13
Previous Reading Date : 31-10-13
Bill Number : 384750
Connected Load : 3 kW
The following tables shows the age distribution of case admitted Meter Reading : 65700
during a day in two different hospitals Previous Meter Reading : 65500
Table 1 Units consumed : 269

Age A survey is conducted for 56 families of a Colony A. The


5-15 15-25 25-35 35-45 45-55 55-65 following tables gives the weekly consumption of electricity
(in years)
of these families.
No. of
6 11 21 23 14 5 Weekly
cases
consumption 0-10 10-20 20-30 30-40 40-50 50-60
Table 2 (in units)
Age No. of
(in years) 5-15 15-25 25-35 35-45 45-55 55-65 16 12 18 6 4 0
families
No. of The similar survey is conducted for 80 families of Colony B
8 16 10 42 24 12
cases and the data is recorded as below:
Refer to table 1 Weekly
consumption 0-10 10-20 20-30 30-40 40-50 50-60
28. The average age for which maximum cases occurred is (in units)
(a) 32.24 (b) 34.36 No. of
0 5 10 20 40 5
(c) 36.82 (d) 42.24 families
Statistics M-135
Refer to data received from Colony A
Match the Following
33. The median weekly consumption is
(a) 12 units (b) 16 units
DIRECTIONS : Each question contains statements given in
(c) 20 units (d) None of these two columns which have to be matched. Statements (A, B, C, D)
34. The mean weekly consumption is in column-I have to be matched with statements (p, q, r, s) in
column-II.
(a) 19.64 units (b) 22.5 units
(c) 26 units (d) None of these 41. The table shows a frequency distribution of the life time
35. The modal class of the above data is I of 400 radio tubes tested at a company.

(a) 0-10 (b) 10-20 Life time Number of Life time Number
(c) 20-30 (d) 30-40 (hours) tubes (hours) of tubes
Refer to data received from Colony B 300 – 399 14 800 – 899 62
400 – 499 46 900 – 999 48
36. The modal weekly consumption is
500 – 599 58 1000 – 1099 22
(a) 38.2 units (b) 43.6 units
600 – 699 76 1100 – 1199 6
(c) 26 units (d) 32 units
700 – 799 68
37. The mean weekly consumption is
Column-II gives data for description given in Column-I,
(a) 15.65 units (b) 32.8 units
match them correctly.
(c) 38.75 units (d) 48 units
Column-I Column-II
Assertion & Reason (A) Upper limit of the fifth class (p) 100
(B) Lower limit of the eighth class (q) 949.5
DIRECTIONS : Each of these questions contains an Assertion
(C) Class marks of the seventh class (r) 1000
followed by Reason. Read them carefully and answer the
question on the basis of following options. You have to select (D) Class interval size (s) 799
the one that best describes the two statements. 42. Following is the distribution of heights of students in a
(a) If both Assertion and Reason are correct and Reason is class and the total number of students is 50.
the correct explanation of Assertion.
Height (in cm) Frequency Cumulative
(b) If both Assertion and Reason are correct, but Reason is
frequency
not the correct explanation of Assertion.
(c) If Assertion is correct but Reason is incorrect. 150 – 155 12 a
(d) If Assertion is incorrect but Reason is correct. 155 – 160 b 25
38. Assertion : The arithmetic mean of the following given 160 – 165 10 c
frequency distribution table is 13.81. 165 – 170 d 43
x 4 7 10 13 16 19 170 – 175 e 48
f 7 10 15 20 25 30 175 – 180 2 f

Reason : x =
∑ fi xi . With the help of information given match the columns.
∑ fi
Column-I Column-II
39. Assertion : If the number of runs scored by 11 players of
a cricket team of India are 5, 19, 42, 11, 50, 30, 21, 0, 52, (Unknown) (Values)
36, 27 then median is 30. (A) a= (p) 35
th
 n +1 (B) b= (q) 5
Reason : Median =   value, if n is odd.
 2  (C) c= (r) 13
40. Assertion : If the value of mode and mean is 60 and 66 (D) d= (s) 50
respectively, then the value of median is 64. (E) e= (t) 12
1 (F) f= (u) 8
Reason : Median = (mode + 2 mean).
2
M-136 Mathematics
43. 46. For the given frequency distribution, match the Column-I
Column-I Column-I with Column-II.
(A) The arithmetic th Class 30–35 35–40 40–45 45–50 50–55 55–60 60–65
n +1
mean of a set of (p) Value of the   Frequency 14 16 18 23 18 8 3
observations is observation.  2 
h = width of the class interval
obtained by
c = cumulative frequency
(B) The value of 1 f = frequency of the class interval to which median belongs
the observation (q) × Value of
2 l1 = lower limit of the median class interval
having maximum
frequency is called  n  th  n  th  Column-I Column-II
  +  + 1 
  2   f
(A) (p) 45.4
 2 
observations. c
(B) (q) 45
(C) If n is odd, then (r) Median l1
(C) (r) 48
median is equal to (D) median (s) 23
(D) If n is even, then (s) Mode
median is equal to
Fill in the Blanks
(E) – divides the (t) Dividing the sum of the
arranged series values of observations by
(in ascending or the number of observations DIRECTIONS : Complete the following statements with an
descending order) parts. appropriate word / term to be filled in the blank space(s).
into two equal parts. 47. Median divides the total frequency into .......... equal parts.
44. 48. The algebraic sum of the deviations from arithmetic
Column-I Column-II mean is always ...............
(A) Mean of first five prime numbers is (p) 12 49. The class mark of a class is 25 and if the upper limit of
(B) Mean of all factors of 24 is (q) 7.5 that class is 40, then its lower limit is ...................
(C) Mean of first six multiples of 4 is (r) 5.6 50. The mid-value of 20-30 is .................
(D) If mean of x – 5y, x – 3y, x – y, (s) 14 51. The sum of 12 observations is 600, then their mean is .........
x + y , x + 3y and x + 5y is 12, 52. In the class interval 35-46, the lower limit is ....... and
then x is upper limit is .............
45. For the following marks distribution of 5 students in 53. A class interval of a data has 15 as the lower limit and 25
an examination, match Column-I with the data given in as the size, then the class mark is .............
Column- II. 54. 0–10, 10–20, 20–30 ............ so on are the classes, the
lower boundary of the class 20–30 is ...............
Class interval 0–10 10–20 20–30 30–40
55. The mid-point of a class interval is called its ................
No. of students 1 3 0 1
56. Facts or figures, collected with a definite purpose, are
Here, called ..................
xk = lower limit of the modal class interval 57. Value of the middle-most observation (s) is called ............
h = width of the class interval 58. The ................. is the most frequently occurring observation.
fk = frequency of the modal class 59. 3 median = mode + ............. mean
fk–1 = frequency of the class preceding the modal class 60. To find the mode of a grouped data, the size of the classes
fk+1 = frequency of the class succeeding the modal class is ..................
Column-I Column-II n
61. On an ogive, point A (say), whose Y- co-ordinate is
2
xk
(A) (p) 3
(half of the total observation), has its X-coordinate equal
fk
(B) (q) 10 to .................. of the data.
fk–1
(C) (r) 0 62. Two ogives, for the same data intersect at the point P.
h
(D) (d) 1 Then, Y-coordinate of P represents .................. .
Statistics M-137
66. The modal value is the value of the variate which divides
True / False the total frequency into two equal parts.
67. The mean of x, y, z is y, then x + z = 2y
DIRECTIONS : Read the following statements and write your
68. 2(Median – Mean) = Mode – Mean.
answer as true or false.
69. Mean may or may not be the appropriate measure of
63. The median for grouped data is formed by using the central tendency.
n  70. If 16 observations are arranged in ascending order, then
− cf
 
formula, Median = l +  2  ×h (8th observation + 9th observation )
f median is
  2
71. Median of 15, 28, 72, 56, 44, 32, 31, 43 and 51 is 43.
Upper class limit + Lower class limit
64. Class mark = 72. Mode of 2, 3, 4, 5, 0, 1, 3, 3, 4, 3 is 3.
2
65. The median of grouped data with unequal class sizes 73. Mean of 41, 39, 48, 52, 46, 62, 54, 40, 96, 52, 98, 49, 42,
cannot be calculated. 52, 60 is 55.4
M-138 Mathematics

ANSWER KEY & SOLUTIONS


1. (c) ( 25 × 70) + ( 40 × 65) + ( 35 × 50)
=
Class 3–6 6–9 9–12 12–15 15–18 18–21 21–24 25 + 40 + 35
1750 + 2600 + 1750 6100
fi 2 5 10 23 21 12 3 = = = 61%
100 100
Here the maximum frequency is 23 and the 4. (a) 
Let the mean weight of a class of 35 students
corresponding class is 12-15 be x 1 and that of both students and a teacher be
x2
We have l = 12, h = 13, f = 23 , f1 = 10

and f2 = 21 Then x 1 = 45 kg and


f − f1
Mode = l + ×h
x 2 = 45 +
500
= 45 + 0.5 = 45.5kg
2f − f1 − f 2
1000
23 − 10
⇒ Mode = 12 + ×3 Σx1 Σx
46 − 10 − 21 x 1 = , x2 = 2
n1 n1
39
⇒ Mode = 12 + = 14.6
15 Σx1 Σx
⇒ 45 = , 45.5 = 2
35 36
2. (a) The class (50-60) has maximum frequency i.e. 20
therefore this is the modal class. ⇒ Σx1 = 1575 kg, Σx2 = 1638 kg

Lower Limit of the modal class = l = 50 ⇒ Total weight = weight of students + weight of
teacher
Class-Interval = h = 10
∴ Weight of teacher = Total weight – weight of
Frequency of the modal class = fi = 20
students
Frequency of the class preceding the modal class
∴ Weight of the teacher = Σx2 – Σx1
= f0 = 13
= 1638 – 1575 = 63 kg
Frequency of the class succeeding the modal class
= f2 = 5 5. (d) 
Clearly, the number of families having income
range (in `)
 f1 − f 0 
Mode, N 0 = l +  ×h
 2f1 − f 0 – f 2  16000 – 19000 = 69 – 50 = 19.
6. (b)
 20 − 13 
= 50 + 
 2 × 20 − 13 − 5  C.I. xi fi fixi

7 0 – 10 5 8 40
= 50 + × 10 = 50 + 3.18 = 53.18
22 10 – 20 15 12 180
3. (d) n1 = 25 x1 = 70% 20 – 30 25 10 250
n 2 = 40 x 2 = 65%
30 – 40 35 11 385
n 3 = 35 x 3 = 50%
40 – 50 45 9 405
n x1 + n 2 x 2 + n 3 x 3
x= 1 50 1260
n1 + n 2 + n 3

Statistics M-139
Σ fx 1260 ∴ Required mean is,
We have x =  = = 25.2 130 + x + 126 + x + 68 + x + 50 + x + 1 + x
Σf 50 x=
375 + 5 x 375 + 0 5 375
x= = = = 75
7. (c) Let a = 200 5 5 5

C.I. fi xi di = xi – 200 fidi 12. (a) 13. (c)


th
0 – 80 22 40 – 160 – 3520  9 +1 th
14. (d) Since n = 9, then middle term =   = 5 term.
 2 
80 – 160 35 120 – 80 – 2800
Now, last four observations are increased by 2.
160 – 240 44 200 0 0 Q The median is 5th observation, which is remains


240 – 320 25 280 +80 +2000 unchanged.

\ There will be no change in median.


320 – 400 24 360 + 160 + 3840
15. (c) 
Mode of the data is 8 as it is repeated maximum
150 – 480
number of times.
5
1 1 16. (b)
x = a +
n ∑ fidi   = 200 + 150 ( – 480)
i =1 17. (b) Let x be the upper limit and y be the lower limit.
= 200 – 3.2 = 196.8 Since, the mid value of the class is 10.

8. (d) First find the sum of first ‘n’ odd natural numbers x+ y
∴ = 10 ⇒ x + y = 20 .....(i)
2
1 = 12
and x – y = 6 (width of the class = 6)  .....(ii)
1 + 3 = 4 = 22
By solving equations (i) and (ii), we get y = 7.
1 + 3 + 5 = 9 = 32
Hence, lower limit of the class is 7.
1 + 3 + 5 + ....+ (2n –1) = n2.
Sum of all the observations
2
18. (c) We know, Mean =
Sum of observations n Total no. of observation
Mean = = =n
n n x + x + 3 + x + 5 + x + 7 + x + 10
⇒ Mean =
It is given that the mean is ‘n’ itself. 5
5 x + 25
∴ n=n 9 = ⇒ x=4
5
Thus, ‘n’ is any natural number. So, mean of last three observations i.e; 14, 11 and 9
20 ( 4) + 40 ( 5) + 30 ( 6) + 10 (10) is
9. (c) x=
20 + 40 + 30 + 10 14 + 11 + 9 34 1
= = 11
80 + 200 + 180 + 100 560 3 3 3
= = = 5.6
100 100
x1 + ... + x15
19. (b) Given, = 13
10. (a) 15

11. (a) Given, x1 + x2 + x3 + ... + x15 = 15 × 13 = 195


(27 + x) + (31 + x) + (89 + x) + (107 + x) + (156 + x) In order to set the second largest and largest, first
82 =
5 thirteen natural numbers are
⇒ 82 × 5 = 410 + 57 ⇒ 410 – 410 = 5x ⇒ x = 0 1, 2, 3, 4, 5, 6, 7, 8, 9, 10, 11, 12, 13
M-140 Mathematics
13 × 14 23. (a) 3rd observation = (14 × 3) + (3 × 17) – (5 × 15) = 18
So, x14 + x15 = 195 –
2 24. (d) 
Since, the mean of a group of eleven consecutive
⇒ x14 + x15 = 195 – 91 = 104 natural numbers is m, then
x + x + 1 + ... + x + 10
Now, from the options, we have =m
11
x14 = 51 and x15 = 53
11x + 55 = 11 m; x + 5 = m; x = m – 5
Now, second largest is 51.
Let n be the mean when next six consecutive natural

2 + 9 + x + 6 + 2 x + 3 + 5 + 10 + 5
20. (c) Mean = =7 numbers are included in the group then
7
x + x + 1 + ..... + x + 16
=n;
3 x + 40 = 49 ⇒ x = 3 17
16 × 17
x + 6 = 9 ⇒ 2 x + 3 = 9 17 x + = 17 n
2
Data (2, 9, 9, 9, 5, 10, 5) and Mode = 9 17x + 8 × 17 = 17 n
21. (d) Let the three numbers be a, b and c such that a > b > c m – 5 + 8 = n
According to the given condition ⇒  n = m + 3
a+b+c (∴ x = m – 5)
= c + 10 = a – 15 = k
3 Hence, required percentage change in the mean

⇒ c = k – 10 n−m m +3− m 300


= × 100 = × 100 = %
m m m
Also, a = k + 15; b = 5 (QMedian = 5)
25. (d) Total number of students = 50
We know that, a + b + c = 3k Average marks of student = 47.5
⇒ k + 15 + 5 + k – 10 = 3k ∴ Total marks of students = 50 × 47.5 = 2375
⇒ 10 = k Now, the student get integer marks
Hence, for maximum number of students who
Thus, a = 25, b = 5, c = 0
could get marks more than 47.5,
∴ Mean of squares of the numbers we will divide total marks by 48.
2 2 2
25 + 5 + 0 650 2 2375
= = = 216 ∴ ≈ 49
3 3 3 48

22. (d) We need to maximize the median in each group to
26. (c) Let x1 < x2 < ... < x11
maximize the average of all median.
x5 + x6
Highest possible median is 18 at there should be 3 Median of x1, x2 ,..., x10 =
2
numbers higher than it in a group of 7.
x5 + x6
So we have Now replace x11 by
 and then arrange in
2
order
1 2 3 18 19 20 21
So, new set of numbers are
Similarly, 4 5 6 14 15 16 17
x1, x2, x3, x4, x5, x5 + x6 , x6, x7, x8, x9, x10,
7 8 9 10 11 12 13 2

Medians are, 18, 14 and 10. x5 + x6


Hence, median is < x6
2
18 + 14 + 10 42
Mean = = = 14
3 3 ⇒ median decreases
Statistics M-141
27. (d) Let x1, x2, x3, ..., x15 be the no. of observations 36. (b) Modal weekly consumption

x1 + x2 + x3 + x4 + ... +x8 = 384  f1 − f 0 


=l+  h = 43.6 units
 2f1 − f 0 − f 2 
x8 + x9 + x10 + ... + x15 = 424 37. (c) Mean weekly consumption
Σfi x i 3100
x1 + x2 + x3 + ... + x15 = 750 = =
Σf i 80 = 38.75 units
x1 + x2 + x3 + x4 + x5 + x6 + x7 + x8 ...+ x15 = 750 38. (a) Both assertion and reason are true, reason is the
correct explanation of the assertion.
384 – x8 + 424 = 750
39. (d) Arranging the terms in ascending order,
Hence, eighth observation is 58
0, 5, 11, 19, 21, 27, 30, 36, 42, 50, 52
 f1 − f 0  th
28. (c) Mode = l +  h = 36.82  11 + 1
 2f1 − f 0 − f 2  median value =  = 6th value = 27
 2 
29. (d) Modal class = The class with maximum frequency
1 1
40. (c) Median = (mode + 2 mean) = ( 60 + 2 × 66 ) = 64
= 35 – 45 3 3

\ Upper limit of modal class = 45 41. (A) → (s); (B) → (r); (C) → (q); (D) → (p)
Σf i x i 2830
30. (d) Mean = = = 35.4 42. (A) → (t); (B) → (r); (C) → (p); (D) → (u); (E) → (q);
Σf i 80
(F) → (s)
 f1 − f 0 
31. (a) Mode = l +  h = 41.4
 2f1 − f 0 − f 2  As we know, cumulative frequency of an interval is equal
to the sum of frequency of that interval and of previous
n 
− Cf intervals.
32. (b) Median = l +  2  × h = 40.2
 f 
  ∴ a = 12

a + b = 25 ⇒ b = 13 (∵ a = 12)
Weekly No. of Cumulative Class fixi
Now, 25 + 10 = c ⇒ c = 35
consumption families fi frequency marks
cf xi c + d = 43 ⇒ d = 8 (∵ c = 35)
1-10 16 16 5 8 43 + e = 48 ⇒ e = 5
10-20 12 28 15 180
and, f = 48 + 2 = 50
20-30 18 46 25 450
30-40 6 52 35 210 43. (A) → (t); (B) → (s); (C) → (p); (D) → (q); (E) → (r)
40-50 4 56 45 180
44. (A) → (r); (B) → (q); (C) → (s); (D) → (p)
50-60 0 0 55 0
Sfi = 56 Sfidi = 110 45. (A) → (q); (B) → (s); (C) → (t); (D) → (q)

46. (A) → (s); (B) → (u); (C) → (q); (D) → (p)


n 
− cf
33. (c) Median = l +  2  h = 20 units 47. two
 f 
 
48. zero
–) = Σfi x i = 19.64 units
34. (a) Mean (x 49. 10
Σf i

35. (c) Modal class = the class with maximum frequency 50. 25

= 20 – 30 51. 50
M-142 Mathematics
52. 35, 46 53. 27.5 63. True 64. True

54. 20 55. class-mark 65. False 66.


False

56. data 57. median 67. True 68.


False

58. mode 59. 2 69. True 70. True

60. uniform 61. Median 71. True 72. True

62. cumulative frequency of the median class. 73. True

You might also like